[obm-l] Re: [obm-l] ajuda em exercício olímpico

2012-10-19 Por tôpico Ralph Teixeira
Oi, Bruno.

Tem uma teoria toda pronta para estas equações a "diferenças finitas"...
Mas neste caso particular, não precisa ir tão longe. Eu sugiro a seguinte
linha:

**Tente escrever os primeiros poucos termos da sequencia para tentar
enxergar algum padrão**

(Mais exatamente, escreva pelo menos até x7 em função de a e b e veja o que
acontece!)

Abraço,
 Ralph

2012/10/19 bruno rodrigues 

>  Dados a e b inteiros, defina a sequência x
> n para n = 0; 1; ... tal que x0 = a, x1 = b e xn+2 = xn+1 - xn. Que
> relação a e b devem satisfazer para que a2009 = 2009².
>
> Alguém pode me ajudar na linha de raciocínio desse exercício? , não sei
> como resolvê-lo.
> Muito grato,
> Bruno
>


[obm-l] Re: [obm-l] Re: [obm-l] ajuda em exercício de trigonometria

2012-10-10 Por tôpico Ralph Teixeira
Botei no computador. As soluções de f(x,y)=sen²(x)+sen²(y)-sen(x+y)=0 para
-0.3

> 2012/10/9 bruno rodrigues :
> > Determine todos os ângulos x e y agudos tais que:
> >
> >  sen²(x)+sen²(y)=sen(x+y)
> >
> > Alguém poderia me ajudar a descobrir a resposta?
> Cara, isso é difícil... Se você tiver coragem:
> 1/ expanda sen(x+y) = sen(x)cos(y) + sen(y)cos(x)
> 2/ eleve ao quadrado
> 3/ substitua todos os cos^2 por 1 - sen^2
> 4/ vai sobrar um termo sem quadrados
> 5/ isole o cara, eleve de novo, substitua de novo
> 6/ Isso vai dar uma equação do 4º grau em sen^2(y) em função de sen^2(x)
>
> Não sei se tem jeito muito mais fácil não... o pior é que há pelo
> menos duas soluções diferentes para y com x=0 (y=0 e y=pi/2), e eu não
> vejo muito bem como elas se comportam.
> --
> Bernardo Freitas Paulo da Costa
>
> =
> Instruções para entrar na lista, sair da lista e usar a lista em
> http://www.mat.puc-rio.br/~obmlistas/obm-l.html
> =
>
<>

Re: [obm-l] Geometria(ajuda)

2012-10-08 Por tôpico Ralph Teixeira
>
> > 2) São dados dois quadrados em um mesmo plano,de lados 2cm e 1cm.Se o
> centro
> > do quadrado de menor lado coincide com um dos vértices do
> > quadrado de maior lado,determine as possíveis áreas da porção do plano
> comum
> > aos dois quadrados.
>

Sejam ABCD o quadrado de lado 2, e PQRS o quadrado de lado 1 com centro em
A e com P dentro de ABCD. Enfim, seja AXPY a interseção dos dois quadrados.
Suponho que todos eles estão rotulados no sentido anti-horário.

Em primeiro lugar, note que SX=PY. De fato, uma rotação em torno de A de 90
graus leva SX em PY (para ver porque X vai em Y, note que SP e AB vão para
PQ e AD).

Agora é simples. Escreva PY=x. Então:

A(APY)=1/2.PY.PA.sin(APY)=1/2.x.sqrt(2)/2.sqrt(2)/2
A(APX)=1/2.PX.PA.sin(APX)=1/2.(1-x).sqrt(2)/2.sqrt(2)/2

Somando, dá 1/4.

Abraço,
Ralph


[obm-l] Re: [obm-l] Análise Combinatória

2012-09-16 Por tôpico Ralph Teixeira
Certamente nao eh a segunda resposta... :)

Digo, para arrumar as nacionalidades, voce tem 3 opcoes para o primeiro, 2
para o segundo, etc., para um total de 3.2^8=768 possibilidades.

Mas isto estah errado, eh claro -- muitas dessas escolhas sao impossiveis,
como por exemplo RBRBRBRUR, que teria 5 russos -- nao vale.

Entao estou dizendo que sao MENOS que 768 possibilidades para a ordenacao
das nacionalidades. Portanto, sao menos que 768.3!.3!.3! filas (permutando
os individuos dentro de cada nacionalidade).

Nao estou resolvendo o problema, mas sei que a resposta eh (bem!) menos que
768.6^3=165888. Faltou exclusao na inclusao-exclusao. :) :) :)

Abraco,
   Ralph

P.S.: Vou resolver o problema de um jeito computacional feio. Faco isso
para mostrar que aas vezes vale a pena botar um pouco de algebra, fazer
tudo ficar mecanico, e mandar brasa!
R(a,b,c)=numero de filas COMECANDO COM UM RUSSO que tem a russos, b
bielorussos e c ucranianos, contando soh nacionalidades, sem ter
nacionalidades consecutivas
B(a,b,c)=numero de filas COMECANDO COM UM BIELO etc etc
U(a,b,c)=comecando com UCRANIANO
Por outro lado, por simetria,
R(a,b,c)=R(a,c,b)=B(b,a,c)=B(b,c,a)=U(c,a,b)=U(c,b,a), certo?

Entao, uma fila comecando por R tem que continuar com B ou com U, usando um
russo a menos:
R(a,b,c)=B(a-1,b,c)+U(a-1,b,c)=R(b,a-1,c)+R(c,b,a-1)
Esta recorrencia nao eh das piores se os numeros forem pequenos! Com
coragem, isto mata o problema:
R(3,3,3)=R(3,2,3)+R(3,3,2)=2R(3,3,2)=2.(R(3,2,2)+R(2,3,2))=
=2.(2.R(2,2,2)+R(3,1,2)+R(2,3,1))=2.(4.R(2,2,1)+R(1,2,2)+R(2,1,2)+R(3,1,1)+R(1,3,1))=
=2.(R(3,1,1)+5.R(2,2,1)+R(1,3,1)+R(1,2,2))

Agora que soh tem 5 fulanos na fila, acho que jah dah para calcular cada um
pensando direto:
R(3,1,1)=2 porque soh tem 3 lugares para por os russos nas 5 posicoes.
Entao eh RURBR ou RBRUR.
R(2,2,1)=R(2,1,1)+R(1,2,1)=4+1=5 (4=permutacoes de RBU sem comecar por R;
1=RBUB).
R(1,3,1)=0 (haveria dois B consecutivos!)
R(1,2,2)=2 (RBUBU ou RUBUB, soh)
Entao R(3,3,3)=2.(2+25+2)=58

O que queremos eh R(3,3,3)+U(3,3,3)+B(3,3,3)=3.58=174

Minto, o que REALMENTE queremos eh isso vezes 3!.3!.3!. Eh, concordo com
a primeira resposta.

2012/9/16 Osmundo Bragança 

>  Caros colegas solicito ajuda na resolução do seguinte problema:
>
> Três russos, três biolerussos e três ucranianos vão ser organizados em uma
> fila.
>
> Determine quantas filas existem que não contêm dois conterrâneos em
> posição consecutiva.
>
> Dois colegas apresentaram resolução, um encontrou, para resposta,
> 174x3!x3!x3!=37.584,
>
> outro colega chegou a:283 824 (via o Princípio da Inclusão-Exclusão)
>
> Qualquer ajuda será muito útil.
>
> Obrigado.
>
> Osmundo Bragança
>


[obm-l] Re: [obm-l] Re: [obm-l] Re: [obm-l] empacotamento máximo de esferas em um espaço tridimensional

2012-09-14 Por tôpico Ralph Teixeira
Confesso que nao sei qual eles usam na feira -- eh FCC?

Mas tem algo sobre FCC ser mais estavel mecanicamente -- bom, pelo menos eh
isso que eu acho que entendi daqui:

http://www.google.com/url?sa=t&rct=j&q=&esrc=s&frm=1&source=web&cd=7&cad=rja&ved=0CEQQFjAG&url=http%3A%2F%2Fwww.tcd.ie%2FPhysics%2FFoams%2Fdocs_public%2FLe_Monde_translation.pdf&ei=VeJTUPiDCaHw0gGYt4CIDQ&usg=AFQjCNFdodaaHQgIY3ZXr18Qs9qKEMjGLg&sig2=rP7uzeJonWtU58hT_4Xdgg

A proposito, tem uma pagina Wikipedia que explica bem a diferenca entre FCC
e HC aqui: http://en.wikipedia.org/wiki/Close-packing_of_equal_spheres.

Abraco,
Ralph

2012/9/14 Rafael Antunes de Andrade 

> Por que que as laranjas são empilhadas em estruturas CFC e não HC?
> Qual é a vantagem de uma sobre a outra já que as duas tem o mesmo fator de
> empacotamento?
>
> Em 10 de setembro de 2012 15:30, Ralph Teixeira escreveu:
>
>> Bom, o jeito ótimo não é exatamente o jeito que o pessoal empilha
>> laranjas na feira? Tipo:
>>
>> http://mathworld.wolfram.com/SpherePacking.html
>> http://mathworld.wolfram.com/HexagonalClosePacking.html
>>
>> http://pt.dreamstime.com/fotos-de-stock-pir%C3%A2mide-das-laranjas-image26292373
>>
>> Abraço,
>> Ralph
>>
>> 2012/9/10 Rafael Antunes de Andrade 
>>
>>> Como eu posso calcular o jeito mais eficiente de se empacotar esferas
>>> idênticas em um espaço tridimensional?
>>> Já pesquisei sobre a Conjectura de Kepler e vi que ela ainda não está
>>> completamente provada. Porém, eu preciso encontrar alguma solução algébrica
>>> ou então algum argumento geométrico que me leve à uma solução aproximada.
>>> Alguém pode me dar uma dica?
>>>
>>> Obrigado,
>>> Rafael
>>>
>>
>>
>


Re: [obm-l] Ajuda em Polinomios

2012-09-12 Por tôpico Ralph Teixeira
Vou fazer usando uns canhoes:

Lema: se R(x) eh um polinomio (nao nulo) com grau menor que Q(x), entao
R(x)/Q(x) nao pode ser inteiro para infinitos valores de x.

Prova:como lim(|x|->+Inf) R(x)/Q(x)=0, existe um certo N0 a partir do qual
|R(x)/Q(x)| < 1 (isto eh, se |x|>N0 teriamos |R(x)/Q(x)|<1). Mas naquela
lista de infinitos valores de x haveria infinitos com modulo maior que N0,
entao R(x)=0 para todos eles, o que eh absurdo (o numero de raizes de R(x)
eh finito).

---///---

Agora, ao problema original: dividindo P(x) por Q(x), fica
P(x)=Q(x)H(x)+R(x), isto eh, P(x)/Q(x)=H(x)+R(x)/Q(x) onde o grau de R(x)
eh menor do que o de Q(x) e os coeficientes de H(x) sao racionais. Pegue o
mmc de todos os denominadores dos coeficientes de H, digamos, M, e
multiplique a coisa toda por M.

MP(x)/Q(x)=MH(x)+MR(x)/Q(x)

Note que, se x eh inteiro, entao MH tambem eh (pois os coeficientes de MH
agora sao inteiros). Assim, se houvesse infinitos valores inteiros de x que
fizessem P/Q ser inteiro, teriamos MR/Q inteiro tambem. Como o grau de MR
eh menor que o grau de Q, usando o lema, temos que R eh identicamente nulo.

Abraco,
  Ralph
2012/9/12 Heitor Bueno Ponchio Xavier 

> Não consigo fazer a seguinte questão:
> Mostre que se P(x) e Q(x) são polinômios de coeficientes inteiros tais que
> P(x)/Q(x) é inteiro para infinitos valores inteiros de x então Q(x) divide
> P(x).
>


[obm-l] Re: [obm-l] Geometria(Construção(2))

2012-09-11 Por tôpico Ralph Teixeira
1) Considere o círculo de diâmetro P1P2. Ele contém o vértice A do
quadrado... Mas, melhor ainda, pense na diagonal AC! Como ela é a bissetriz
do ângulo P1AP2, então ela passa pelo ponto D1, médio do arco P1P2 daquele
círculo (veja figura anexa, viva Geogebra!), que é DETERMINADO A PARTIR DE
P1 e P2!

Analogamente, você pode encontrar D3, médio do arco P3P4 do círculo de
diâmetro P3P4. Como D1D3 será a diagonal do quadrado, você pode intersectar
esta diagonal com os círculos para achar A e C.

Agora repita para B e D, e acabou!

(Se alguém quiser o Geogebra da construção, mando por E-mail -- acho que a
lista não aceitaria o anexo)

2) Este é bem mais simples:
i) Desenhe o círculo inscrito, marque nele um arco PQ de tamanho 180-2A.
ii) Trace as tangentes ao círculo por P e Q, intersecte-as, este é o ponto
A (note que PÂQ=Â, de fato).
iii) Agora é só marcar b (a partir de A) em cima da reta AQ para achar o
ponto C...
iv)... e traçar a tangente ao círculo por C para achar o ponto B (sobre AP).

Para que a construção funcione, precisamos que Q esteja entre A e C, isto
é, que b>r.sin(Â/2)

Abraço,
   Ralph

P.S.: Note que há um outro arco P1P2! O que aconteceria se a gente
escolhesse D1 como médio desse OUTRO arco, assim como D3, D2 e D4? :) :) :)

2012/9/10 marcone augusto araújo borges 

>  1)Os pontos P1,P2,P3,P4 pertencem aos lados consecutivos de um quadrado
> ABCD.Construa com régua e compasso o quadrado.Justifique sua construção.
>
>
>  .P1
>
>.P2
>
>
>
> .P3
>
>
>  .P4
>
>  2) Construa o triangulo ABC conhecendo o angulo A,o lado b e o raio r do
> círculo inscrito.Justifique.
>
<>

[obm-l] Re: [obm-l] empacotamento máximo de esferas em um espaço tridimensional

2012-09-10 Por tôpico Ralph Teixeira
Bom, o jeito ótimo não é exatamente o jeito que o pessoal empilha laranjas
na feira? Tipo:

http://mathworld.wolfram.com/SpherePacking.html
http://mathworld.wolfram.com/HexagonalClosePacking.html
http://pt.dreamstime.com/fotos-de-stock-pir%C3%A2mide-das-laranjas-image26292373

Abraço,
Ralph

2012/9/10 Rafael Antunes de Andrade 

> Como eu posso calcular o jeito mais eficiente de se empacotar esferas
> idênticas em um espaço tridimensional?
> Já pesquisei sobre a Conjectura de Kepler e vi que ela ainda não está
> completamente provada. Porém, eu preciso encontrar alguma solução algébrica
> ou então algum argumento geométrico que me leve à uma solução aproximada.
> Alguém pode me dar uma dica?
>
> Obrigado,
> Rafael
>


Re: [obm-l] ab=1

2012-09-03 Por tôpico Ralph Teixeira
Ah, demorei para sacar este, mas agora que acabou é uma linha!

Note que a(b+ba-1)=1 também. Como b é o ÚNICO inverso à direita de a... :)

Abraço,
  Ralph

2012/9/3 Samuel Wainer 

>  Esse exercício parece ser fácil, mas está me complicando.
>
> Seja R um anel associativo com 1. Seja a em R e suponha que existe um
> único b em R tal que ab=1. Provar que ba=1.
>
> Consigo fazer um bem parecido, se ab=1 e bc=1 implica que a=c, ou seja o
> inverso será igual pela direita e esquerda. Mas esse ali de cima não sai.
> Alguém tem alguma ideia? :)
>


Re: [obm-l] Questao de probabilidade interessante

2012-09-02 Por tôpico Ralph Teixeira
O problema me parece bem dificil! Estamos supondo que dois eucaliptos nao
podem se intersectar? Entao a distribuicao de probabilidade dos valores de
x serah uma doideira quase impossivel de calcular... Note-se que as
coordenadas x_1, x_2, ..., x_{10} NAO SAO independentes -- e mesmo que
fossem, ainda seria um problema BEM interessante!

Depois mostra para a gente a solucao apresentada -- estou curioso.

Abraco,
Ralph
2012/9/2 João Maldonado 

>  Na lista de probabilidade  que o meu professor passou, uma questao em
> especial me chamou a atencao:
>
> Um terreno de 500m(eixo x)x200m (eixo z) comprimento tem 100.000 de
> eucaliptos plantados aleatoriamente. Considere um eucalipto como sendo um
> cilindro fino de altura h extremamente grande e raio R=10cm. para um
> observador passando na rua paralelamente ao eixo x, qual a probabilidade
> dele enxergar o outro lado do terreno, ou seja, observar uma brexa (buraco)
> sem eucaliptos, com pelo menos 20cm de largura? Considere que o observador
> so enxerga retas luminosas paralelas ao eixo z.
>
> Como faco isso?
>
> []`s
> Joao
>


Re: [obm-l] ajuda (faltou dizer que:)

2012-08-30 Por tôpico Ralph Teixeira
E para completar a solucao, vamos encontrar alguma funcao f que satisfaca
aquela relacao (caso f nao existisse, a resposta seria "f nao existe, entao
f(1) tambem nao").

Tomando y=1, vem:

f(x).f(1)-f(x)=x+1/x
Como f(1), se existir, eh 2, temos:
f(x)=x+1/x

Agora eh soh verificar se esta funcao de fato serve:

f(x).f(y)-f(xy)=(x+1/x)(y+1/y)-(xy+1/xy)=x/y+y/x.

Beleza!

Abraco,
   Ralph
2012/8/30 LEANDRO L RECOVA 

>  Fazendo x=y=1,
>
> f(1)^2 - f(1) -2 =0.
>
> Equacao do 2o grau.
>
> Delta = 1 -4(-2) = 9
>
> f(1) = (1 + 3)/2 ou
>
> f(1) = (1-3)/2. Essa ultima esta descartada.
>
> Entao, f(1)=2.
>
> Regards,
>
>  --
> From: mat.mo...@gmail.com
> Date: Thu, 30 Aug 2012 07:56:05 -0300
> Subject: [obm-l] ajuda (faltou dizer que:)
> To: obm-l@mat.puc-rio.br
>
>
>
> Será que alguém poderia me ajudar na seguinte questão:
>
> Seja f: R*+ - R*+ uma função tal que
>
> f(x).f(y)-f(x.y)=x/y+y/x, então f(1)=?
>
>


[obm-l] Re: [obm-l] RES: [obm-l] Fwd: [obm-l] Solução única

2012-08-29 Por tôpico Ralph Teixeira
30 p/ mim, 20 p/ voce, e nao se fala mais nisso. ;) ;) ;)
2012/8/29 

> Nananinanão!!!
>
> ** **
>
> Eu resolvi primeiro! Mandei ver Catalan e pronto! Reconheço que é tiro de
> canhão pra matar mosca, mas as 50 pratas são minhas!
>
> ** **
>
> Ralph, não me leve a mal, é que eu tô precisando dessa grana, pô!
>
> ** **
>
> *Albert Bouskela*
>
> bousk...@gmail.com
>
> ** **
>
> *De:* owner-ob...@mat.puc-rio.br [mailto:owner-ob...@mat.puc-rio.br] *Em
> nome de *Ralph Teixeira
> *Enviada em:* quarta-feira, 29 de agosto de 2012 10:34
> *Para:* obm-l@mat.puc-rio.br
> *Assunto:* [obm-l] Fwd: [obm-l] Solução única
>
> ** **
>
> Nao sei porque esta mensagem nao apareceu na lista Estou tentando de
> novo, para ver se ganho os R$50.
>
> -- Forwarded message --
> From: *Ralph Teixeira* 
> Date: 2012/8/28
> Subject: Re: [obm-l] Solução única
> To: obm-l@mat.puc-rio.br
>
> 
>
> Hmmm Veja se voce conhece este fato:
>
>  
>
> FATO: f(x)=(1+r/x)^x eh uma funcao crescente (para x>=1) que tende para
> e^r quando x->+Inf. Ou seja, (1+r/x)^x < e^r sempre que x>=1.
>
>  
>
> Agora sim!
>
>  
>
> i) Nao ha solucao com b>a>=3. De fato, escrevendo b=a+r, vem:
>
>  
>
> a^b-b^a = a^a.(a^r-(1+r/a)^a) > a^a.(a^r-e^r) > a^a.(a-e) > 3^3.(0.2) > 1*
> ***
>
>  
>
> onde usei que r>=1 para sumir com r (note que
> (a^r-e^r)=(a-e)(a^(r-1)+a^(r-2)+...+e^(r-1)) e este termo imenso eh >=1), e
> que a>=3 e que e=2.781828... no finzinho. 
>
>  
>
> ii) Nao ha solucao com 3<=b
>  
>
> a^b-b^a = b^b.((1+r/b)^b-b^r) < b^b.(e^r-b^r) < 0 pois b>e e r>=1.
>
>  
>
> Como obviamente tambem nao ha solucao com a=b, as unicas possiveis
> solucoes tem pelo menos um dos numeros menores do que 3. Agora eh soh
> analisar os casos a=1, a=2, b=1 e b=2:
>
>  
>
> -- Se a=1 entao 1-b=1, nao presta.
>
> -- Se a=2, entao 2^b-b^2=1, isto eh, 2^b=b^2+1. Entao b eh impar, digamos,
> b=2k+1, entao 2^b=4k^2+4k+2 nao eh divisivel por 4, entao b=1. De fato
> (a,b)=(2,1) serve.
>
> -- Se b=1, entao a-1=1, isto eh, a=2, o que jah vimos acima.
>
> -- Se b=2, entao a^2-2^a=1, isto eh, 2^a=a^2-1=(a+1)(a-1). Entao ambos a-1
> e a+1 tem que ser potencias de 2 (cuja diferenca eh 2!), ou seja, a-1=2 e
> a+1=4. Assim, a=3.
>
>  
>
> Cade meus 50 reais? ;)
>
>  
>
> Abraco,
>
>   Ralph
>
>  
>
> 2012/8/28 João Maldonado 
>
> Meu amigo me passou um desafio anteontem, falou que se eu resolvesse até
> ontem a meia-noite, ele me dava 50 reais.
> Acontece que por mais insistente que eu tenha sido não saiu muita coisa :)
>
> A aposta já acabou e ele também não sabe a resolução, e eu quero muito
> saber como se resolve isso!
> Se alguém puder me dar uma ajuda eu agradeço
>
> Prove que a^b - b^a = 1  admite única e exclusivamente a solução (3, 2),
> para a e b naturais maiores de 0.
>
>
> []'s
> João
>  
>
> ** **
>
> ** **
>


[obm-l] Fwd: [obm-l] Solução única

2012-08-29 Por tôpico Ralph Teixeira
Nao sei porque esta mensagem nao apareceu na lista Estou tentando de
novo, para ver se ganho os R$50.

-- Forwarded message --
From: Ralph Teixeira 
Date: 2012/8/28
Subject: Re: [obm-l] Solução única
To: obm-l@mat.puc-rio.br


Hmmm Veja se voce conhece este fato:

FATO: f(x)=(1+r/x)^x eh uma funcao crescente (para x>=1) que tende para e^r
quando x->+Inf. Ou seja, (1+r/x)^x < e^r sempre que x>=1.

Agora sim!

i) Nao ha solucao com b>a>=3. De fato, escrevendo b=a+r, vem:

a^b-b^a = a^a.(a^r-(1+r/a)^a) > a^a.(a^r-e^r) > a^a.(a-e) > 3^3.(0.2) > 1

onde usei que r>=1 para sumir com r (note que
(a^r-e^r)=(a-e)(a^(r-1)+a^(r-2)+...+e^(r-1)) e este termo imenso eh >=1), e
que a>=3 e que e=2.781828... no finzinho.

ii) Nao ha solucao com 3<=be e r>=1.

Como obviamente tambem nao ha solucao com a=b, as unicas possiveis solucoes
tem pelo menos um dos numeros menores do que 3. Agora eh soh analisar os
casos a=1, a=2, b=1 e b=2:

-- Se a=1 entao 1-b=1, nao presta.
-- Se a=2, entao 2^b-b^2=1, isto eh, 2^b=b^2+1. Entao b eh impar, digamos,
b=2k+1, entao 2^b=4k^2+4k+2 nao eh divisivel por 4, entao b=1. De fato
(a,b)=(2,1) serve.
-- Se b=1, entao a-1=1, isto eh, a=2, o que jah vimos acima.
-- Se b=2, entao a^2-2^a=1, isto eh, 2^a=a^2-1=(a+1)(a-1). Entao ambos a-1
e a+1 tem que ser potencias de 2 (cuja diferenca eh 2!), ou seja, a-1=2 e
a+1=4. Assim, a=3.

Cade meus 50 reais? ;)

Abraco,
  Ralph

2012/8/28 João Maldonado 

>  Meu amigo me passou um desafio anteontem, falou que se eu resolvesse até
> ontem a meia-noite, ele me dava 50 reais.
> Acontece que por mais insistente que eu tenha sido não saiu muita coisa :)
>
> A aposta já acabou e ele também não sabe a resolução, e eu quero muito
> saber como se resolve isso!
> Se alguém puder me dar uma ajuda eu agradeço
>
> Prove que a^b - b^a = 1  admite única e exclusivamente a solução (3, 2),
> para a e b naturais maiores de 0.
>
>
> []'s
> João
>
>


[obm-l] Re: [obm-l] Não consigo resolver

2012-08-09 Por tôpico Ralph Teixeira
1. Bom, a chave eh olhar para os divisores de n. Se n tiver pelo menos 4
divisores positivos (distintos), digamos, 1, p, q=n/p e n, entao n^2 divide
n!. Por que? Oras, n!=1.2.3...n. Nesse produto teriamos os numeros p, q e
n, e este produto jah tem pqn=n^2.

Em suma, para que n! NAO seja divisivel por n^2, n tem que ter no maximo 3
divisores positivos distintos.

a) UM DIVISOR. Seria n=1... mas nao serve, pois 1^2=1 divide 1!=1.
b) DOIS DIVISORES. Entao n seria um numero primo, digamos, n=p primo. Entao
o fator p nao aparece em nenhum dos numeros 1,2, ..., (p-1), e portanto soh
aparece uma vez em p!. Entao p^2 nao divide p!.
c) TRES DIVISORES. Entao n seria o quadrado de um primo, digamos, n=p^2.
Mas, se p>2, teriamos os fatores 1,p,2p,p^2 em n! (pois 1

> Estou com alguns problemas aqui que não estão saindo e agradeceria
> bastante ajuda.
>
> 01. Encontre todos os números ''n'' naturais tais que n² não seja divisor
> de n!
>
> 02.Prove que dentre quaisquer cinco reais y_1, y_2, y_3, y_4, y_5, existem
> dois que satisfazem:
> 0<= (y_i - y_j)/(1+(y_i)(y_j)) <=1.
>


[obm-l] Re: [obm-l] Re: [obm-l] Re: [obm-l] equação exponencial

2012-08-08 Por tôpico Ralph Teixeira
Bom, claro que ao verificar que x=2 eh solucao e mostrar que ela eh unica,
voce resolveu a equacao... Mas entendo que voce quer saber como resolver
algebricamente uma equacao do tipo a^x+b^x=c^x (a, b e c dados).

Claro que isto depende do que "algebricamente" significa. Entao deixa eu
dizer assim: eu nao sei nenhuma maneira de escrever x como funcao de a, b e
c usando apenas as funcoes que eu conheco -- isto eh, potencias, raizes,
logaritmos, funcoes trigonometricas, e ateh algumas coisas mais obscuras
como a funcao W de Lambert. Aposto que nao eh possivel, mas nao tenho
certeza.

Abraco,
   Ralph

2012/8/8 Vanderlei * 

> *Obrigado Ralph. Mas existe um método algébrico para concluirmos que x =
> 2?*
>
> Em 8 de agosto de 2012 00:40, Ralph Teixeira  escreveu:
>
>> Lema: Se 0> Dem.: Note que x>0 satisfaz a equacao sse f(x)=(c/b)^x-(a/b)^x=1.
>> Mas esta funcao f(x) eh crescente (pois c/b>1 e a/b<1), entao tem no maximo
>> uma raiz positiva!
>>
>> (De fato, note que f(0)=0 e f(+Inf)=+Inf, entao f(x)=1 tem EXATAMENTE uma
>> raiz real positiva.)
>>
>> Abraco,
>>   Ralph
>>
>> 2012/8/7 Vanderlei * 
>>
>>> Alguém pode ajudar a resolver a equação.
>>>
>>> *[sqrt(4 - sqrt15)]^x + [sqrt(4 + sqrt15)]^x = [2sqrt2]^x*
>>>
>>> É trivial que o número 2 é solução, mas será que não existem outras?
>>>
>>> Obrigado!
>>>
>>
>>
>


[obm-l] Re: [obm-l] equação exponencial

2012-08-07 Por tôpico Ralph Teixeira
Lema: Se 00 satisfaz a equacao sse f(x)=(c/b)^x-(a/b)^x=1. Mas esta
funcao f(x) eh crescente (pois c/b>1 e a/b<1), entao tem no maximo uma raiz
positiva!

(De fato, note que f(0)=0 e f(+Inf)=+Inf, entao f(x)=1 tem EXATAMENTE uma
raiz real positiva.)

Abraco,
  Ralph

2012/8/7 Vanderlei * 

> Alguém pode ajudar a resolver a equação.
>
> *[sqrt(4 - sqrt15)]^x + [sqrt(4 + sqrt15)]^x = [2sqrt2]^x*
>
> É trivial que o número 2 é solução, mas será que não existem outras?
>
> Obrigado!
>


[obm-l] Re: [obm-l] Condição necessária e suficiente

2012-06-30 Por tôpico Ralph Teixeira
Eu penso assim: as seguintes frases sao equivalentes entre si:

"Se p, entao q"
"q, se p"
"p, somente se q"
"p implica q"
"p eh suficiente para q"
"q eh necessario para p"

Entao, eh claro, tambem sao equivalentes as seguintes frases:
"Se q, entao p"
"p, se q"
"q, somente se p"
"q implica p"
"q eh suficiente para p"
"p eh necessario para q"

Entao, para mim, eh dificil fazer a distincao que voce quer (de qual eh a
afirmacao imediata, e qual vem depois). Para mim, sao imediatemente
equivalentes:
"p, se e somente se q"
"q, se e somente se p"
"p eh necessario e suficiente para q"
"q eh necessario e suficiente para p"
"p eh equivalente a q"

Abraco, Ralph.





2012/6/30 Paulo Argolo 

> Caros Colegas,
>
> Sendo p e q proposições, parece-me que a proposição "p se, e somente se,
> q" afirma que q é condição necessária e suficiente para p. Ela não afirma,
> pelo menos de imediato, que p é condição necessária e suficiente para q.
> Gostaria de saber o que os colegas pensam a respeito.
> Abraços do Paulo!
> =
> Instru�ões para entrar na lista, sair da lista e usar a lista em
> http://www.mat.puc-rio.br/~obmlistas/obm-l.html
> =
>


Re: [obm-l] Ajuda(trigonometria)

2012-06-27 Por tôpico Ralph Teixeira
Voce quer 1+(sen(ax))^2=cosx. O lado esquerdo eh >=1, e o lado direito eh
<=1. Entao isso ai soh tem solucao se forem ambos 1, isto eh, sin(ax)=0 e
cosx=1!!

Mas as solucoes x nao-nulas de cosx=1 sao todas da forma x=2.k.pi com k
inteiro nao-nulo. Para algum destes servir na primeira, tem que ser:

sin(a.2.k.pi)=0, ou seja, 2ak=n tem que ser inteiro tambem. Assim,
a=n/(2k), isto eh, a tem que ser um numero racional.

Em suma: "sua equacao tem solucao nao-nula" IMPLICA "a eh racional".

Agora a volta: se a for racional, digamos, a=m/n com m e n inteiros, entao
note que x=2.n.pi faz com que:
sin(a.x)=sin(2.m.pi)=0
e
cosx=cos(2.n.pi)=1
Entao esse x seria uma solucao nao-nula da sua equacao.

RESPOSTA: Para a racional qualquer.
Abraco,
 Ralph
2012/6/27 marcone augusto araújo borges 

>  Determine para que valores de a a equação 1 + (senax)^2 = cosx admita
> alguma solução não nula.
> Agradeço desde já.
>


Re: [obm-l] MQ>=MA>=MG>=MH

2012-06-15 Por tôpico Ralph Teixeira
Você PODE usar essa desigualdade para quaisquer x,y,z positivos. O problema
não é na desigualdade, é de lógica.

Em outras palavras: você mostrou que SE x,y,z são positivos com x+y+z=5 e
xy+xz+yz=3 ENTÃO x<=H (onde H é um número HORROROSO, achei numericamente
H~4.565). Isto está corretíssimo! Infelizmente, isto só te dá uma COTA
SUPERIOR para x, não necessariamente seu valor máximo!

Ou seja, você não pode afirmar que, se x=H, então existem y e z tais que
x+y+z=5 e xy+xz+yz=3. Aliás, lembrando que MA=MG apenas quando x=y=z, só
haveria um tal par (x,y,z) se fosse x=y=z, ou seja, se 3x=5 e 3x^2=3, o que
é impossível.

Então: raciocínio correto, leva a uma cota superior, mas não encontra o
máximo. Infelizmente, para encontrar o MÁXIMO de x, você vai precisar de
algum raciocínio mais rebuscado... Por exemplo, note que x, y e z são as 3
raízes reais de t^3-5t^2+3t-P=0. Tome f(t)=t^3-5t^2+3t. Note que o gráfico
de f(t) é uma cúbica do tipo sobe-desce-sobe que tem 3 raízes, uma sendo 0
e as outras duas sendo positivas, digamos, x1 e x2. O que eu preciso é
escolher P para que f(t)=P tenha 3 raízes reais positivas. Graficamente, eu
preciso arrumar P de forma que a reta y=P corte o gráfico de f(t) em três
lugares, sendo o da direita o maior número possível (para que x seja maior
possível). Se você fizer o gráfico, vai ver que isto ocorrerá quando
tivermos uma raiz dupla e uma simples.

Em suma, o que você quer é y=z e x sendo a terceira raiz. Assim:

x+2y=5
2xy+y^2=3

Resolve isto para achar y=z=1/3 e x=13/3, que é a resposta desejada. Note
que 13/3=4.333

>  Olá a todos.
> Eu queria saber condições para podermos usar qualquer uma dessas relações
> de desigualdades de forma que os termos sejam reais. Por exemplo:
>
> x+y+z=5, xy+xz+yz=3 , qual o maior valor de x, para x,y,z E R?
>
> yz= 3-x(5-x)
> Usando MA>= MG temos
>
> 5/3 >= (x(3-x(5-x)) )^(1/3)
>
> Resolvendo tal equação, chegamos em um número feio... porem, isso é falso
> ! o maior valor para x,y e z E R, é 13/3
>
> Eu queria saber em que condições tal relação se torna valida.
>
> Grato.
> Coulbert
>


[obm-l] Re: [obm-l] Ajuda em combinatória

2012-06-11 Por tôpico Ralph Teixeira
Seja D(n) esse número que você quer. Então:

D(0)=1 (vazio)
D(1)=1+1=2 (1 com 0 elementos, 1 com 1 elemento)
D(2)=1+2+0=3 (vazio e os subconjuntos unitários)
D(3)=1+3+1+0=5 (vazio, os unitários e {1,3}, mas com 3 elementos não dá)

Será que eu arrumo uma recorrência? Oras, os subconjuntos que eu arrumo em
{1,2,...,n} são de 2 tipos:

-- os que contém o último número n; então eles não podem ter o n-1! Assim,
eu tenho que escolher  (e basta escolher!) um subconjunto de {1,2,...,n-2}
que não tenha dois inteiros consecutivos, o que pode ser feito de D(n-2)
jeitos;
-- os que não tem o ultimo número n; então eu tenho que escolher (e basta!)
um subconjunto de {1,2,...,n-1} corretamente, o que pode ser feito de
D(n-1) jeitos.

Assim, D(n)=D(n-1)+D(n-2) -- e a sequência dos D(n) é a sequencia de
Fibonacci, começando com 1,2,3,5,8,13,... Você agora pode arrumar uma
fórmula fechada para ela.

Abraço,
 Ralph

2012/6/11 marcone augusto araújo borges 

>  Quantos subconjuntos do conjunto {1,2,...,n} não contêm dois inteiros
> consecutivos?
>


[obm-l] Re: [obm-l] Re: [obm-l] RE: [obm-l] Colômbia- álgebra

2012-06-11 Por tôpico Ralph Teixeira
É, acho que o Bernardo tem razão. Considere o sistema:
a^12+b^12+c^12=8
a^3+b^3+c^3=6abc

Tomando por exemplo c=0, temos uma solução (a,b,c)=(2^(1/6),-2^(1/6),0),
que daria S=a^6+b^6+c^6=4 (que eu estou apostando que é o valor máximo de
S, mas não demonstrei).
Por outro lado, tomando b=c, podemos achar numericamente uma possível
solução como (a,b,c)=(0.38150,1.1225,1.1225), que daria S=2.8706, que é
totalmente diferente.

Acho que vale a pena conferir se o enunciado é exatamente esse mesmo.

Abraço,
Ralph

2012/6/10 Bernardo Freitas Paulo da Costa 

> > Sejam a,b,c reais tais que
> > a^12+b^12+c^12=8
> > [(a-b)^2+(b-c)^2+(c-a)^2]/abc= 6/(a+b+c)
> >
> > Calcule a^6+b^6+c^6.
>
> 2012/6/10 Thiago Tarraf Varella :
> > mas desenvolvendo a 2a. equação na raça, obtém-se
> > a³ + b³ + c³ = 6abc
>
> > Isso deve ajudar... alguma sugestão do que fazer agora?
>
> Isso dá uma boa ajuda, mas acho que a conclusão final é que não dá
> para determinar (numericamente) a^6 + b^6 + c^6.
>
> Eu argumentaria assim: seja P(x) = x^3 - Ax^2 + Bx - C um polinômio
> cujas raízes são a, b e c. Daí, como todas as quantidades são
> homogêneas, vamos gastar braço e escrever tudo em função de A, B e C.
> Vai dar um monte de trabalho, mas azar. Daí, você obtém uma fórmula
> feia para S_12 e outra para S_6, as somas de potências (essa até é
> "bonitinha":
>
> 3 * S_6 = A^4 (A² - 3B) - 3B^3,
>
> se eu não me enganei.)
>
> Bom, a fórmula para S_12 é feia, mas não morde. Dá uma equação de grau
> 6 homogênea em A^2 e B, fixe um, ache o outro, dado que S_12 = 8.
>
> Bom, agora vem um argumento meio abstrato.
>
> Note que a região (S_12 = 8 inter S_3 = 6C) é de dimensão 1 em C^3 (e
> também em R^3, a menos que sejam pontos isolados, mas não é o caso), e
> por ser de dimensão 1, a gente chama de "curva". Note que ela pode ter
> várias partes (componentes conexas), mas isso não importa. Note também
> que o valor que a gente quer calcular é S_6, que também é homogêneo.
> Então há duas possibilidades: ou ele é constante nessa curva de
> dimensão 1 (e basta pegar uma solução qualquer a, b, c e ver o quanto
> dá), ou ela é não-constante em *cada componente da curva*, e (em C^3)
> assume *todos* os valores complexos possíveis em cada componente
> conexa. O mais legal é que essa dicotomia continua mais ou menos
> válida em R (que é o que nos interessa!), e o comportamento em R é o
> mesmo que o de C: se for constante em C, será em R (óbvio!), e a
> recíproca também vale.
>
> Bom, daí você apela para qualquer software de cálculo algébrico, bota
> a equação, chuta uns valores para B (ou A, tanto faz), pede pra
> resolver a equação do S_12, substitui no S_6, e vê se dá igual. Se eu
> não fiz nenhuma besteira no caminho, existem valores de A e B para os
> quais dá diferente. Note que os valores de A e B que eu escolhi, muito
> provavelmente vão dar uma equação em que um de a, b e c será complexo.
> Mas isso não importa, pelo que eu disse antes: se existir alguma forma
> de S_6 mudar, mesmo que passando pelos complexos, também será o caso
> se a, b e c forem apenas reais. Ufa!
>
> Abraços,
> --
> Bernardo Freitas Paulo da Costa
>
> =
> Instruções para entrar na lista, sair da lista e usar a lista em
> http://www.mat.puc-rio.br/~obmlistas/obm-l.html
> =
>


[obm-l] Re: [obm-l] 2n+1=k², n+1=?

2012-06-09 Por tôpico Ralph Teixeira
Coom 2n+1 eh impar, se for quadrado perfeito serah quadrado de um numero
impar. Assim:

2n+1=(2k+1)^2 (onde k eh inteiro)
2n+1=4k^+4k+1
n=2k^2+2k
n+1=2k^2+2k+1=k^2+(k+1)^2

Abraco,
 Ralph

2012/6/9 Lucas Hagemaister 

>
> Se n é um inteiro positivo tal que 2n+1 é um quadrado perfeito, mostre que
> n+1 é a soma de dois quadrados perfeitos sucessivos.
>


[obm-l] Re: [obm-l] m/n = p/q => p = km e q = kn (correção do enunciado)

2012-06-09 Por tôpico Ralph Teixeira
Agora faz que nem o outro. A propriedade basica eh a seguinte:

Teorema: "Se mdc(a,b)=1 e a eh divisor de bc, entao a eh divisor de c."

Agora eh simples: temos mq=pn, entao m|pn. Como (m,n)=1, entao m|p. Assim,
p=km para algum k inteiro.

Abraco,
  Ralph

2012/6/9 Paulo Argolo 

> De fato, prezado Ralph, o enunciado está equivocado. Faço a correção:
>
> Sendo m, n, p e q inteiros positivos, tal que m/n é fração irredutível e
> m/n = p/q, como podemos provar que existe um inteiro k satisfazendo as
> igualdades p = km e q = kn?
>
> Obrigado.
> Paulo
> *
> *=
> Instru�ões para entrar na lista, sair da lista e usar a lista em
> http://www.mat.puc-rio.br/~obmlistas/obm-l.html=


Re: [obm-l] m/n = p/q => p = km e q = kn

2012-06-09 Por tôpico Ralph Teixeira
Bom, isto eh falso: 6/9=8/12, 8>6, 12>9 mas nao existe esse k.

Faltou dizer que m/n eh fracao irredutivel, talvez?

Abraco,
 Ralph

2012/6/9 Paulo Argolo 

> Caros Colegas,
>
> Sendo m, n , p e q inteiros positivos, tal que p>m, q>n e m/n = p/q,Â
> como podemos provar que existe um inteiro k, satisfazendo as igualdades
> p = km e q = kn ?
>
> Desde já, muito grato.
> Paulo
>
> =
> Instruções para entrar na lista, sair da lista e usar a lista em
> http://www.mat.puc-rio.br/~obmlistas/obm-l.html
> =
>


Re: [obm-l] Prova combinatoria

2012-06-06 Por tôpico Ralph Teixeira
O que o Bernardo disse! Usando a ideia dele, cheguei em:

"Você tem um alfabeto de apenas 3 letras -- digamos, A, B e C -- e quer
montar uma palavra com n símbolos (a ordem importa, repetições obviamente
são aceitas). Por exemplo, uma palavra válida com n=6 é ABBACA (eu sem
querer escrevi outra parecida, mas depois li o que eu tinha escrito e era
ligeiramente ofensivo... :) :) :)). Quantas destas palavras têm exatamente
k consoantes?"

Abraço,
Ralph

2012/6/6 Bernardo Freitas Paulo da Costa 

> 2012/6/6 marcone augusto araújo borges :
> > notação:(n,p)-->número binomial de numerador n e denominador p
> >
> > 1 + 2(n,1) + 4(n,2) + ...[2^(n-1)](n,n-1)+ [2 ^n](n,n) = 3^n
> > Se desenvolvermos (x + 2y)^n e substituirmos x por 1 e y por
> 1,encontraremos
> > a expressão do lado esquerdo,que será igual a (1+ 2)^n
> Veja que isso é também a expansão de (x + y)^n com x=1 e y=2.
>
> > O exercício pede para encontrar uma prova combinatória.
> Uhm, pra tentar uma prova combinatória, eu faria *antes de mais nada*
> uma prova combinatória da mesma fórmula só que com x=1 e y=1. Talvez
> você até já conheça uma. Daí tente generalizar!
>
> > Já pensei,pensei e não saiu.
>
> Abraços,
> --
> Bernardo Freitas Paulo da Costa
>
> =
> Instruções para entrar na lista, sair da lista e usar a lista em
> http://www.mat.puc-rio.br/~obmlistas/obm-l.html
> =
>


[obm-l] Re: [obm-l] Questão de combinatoria

2012-06-05 Por tôpico Ralph Teixeira
Escolher um retangulo significa escolher o canto esquerdo superior e o
canto direito inferior.

Como o canto esquerdo superior tem que ficar a "Noroeste" desta celula, sao
pq opcoes para ele.
Como o canto direito inferior tem que ficar a "Sudeste" dela, sao
(m-p+1)(n-q+1) opcoes para ele (note que isto inclui a propria celula como
possibilidade).

Total: p(m+1-p)q(n+1-q) opcoes. Eh isso?

Abraco,
  Ralph

2012/6/5 marcone augusto araújo borges 

>  Em uma tabela com m linhas e n colunas,a célula da intersecção da p-ésima
> linha com a q-ésima coluna está marcada.
> Quantos retângulos formados pelas células da tabela contêm a celula
> marcada?
>
>
>


Re: [obm-l] Problema de grafos

2012-06-05 Por tôpico Ralph Teixeira
Versao relampago:
casa-dos-pombos, 68,68,68,69,69,69,70,70,70,...,100,100,100,101,101,101,
contradicao.

Versao explicada:
Ha 101-67=34 numeros entre 68 e 101 (que sao os possiveis numeros de
amigos) de cada fulano.
Seja xi o numero de estudantes com i amigos (onde i=68,69,...,101). Note
que sao 34 numeros xi, cuja soma eh 102. Queremos provar que algum deles eh
maior ou igual a 4, certo?
Entao suponha por contradicao que todos eles sao menores ou iguais a 3.
Entao a soma eh no maximo 34x3=102, o que soh serah atingido se TODOS forem
iguais a 3. Entao chegamos aa conclusao de que o unico jeito do nosso
teorema furar seria se houvesse 3 alunos com 68 amigos, outros 3 com 69
amigos, e assim por diante, e 3 com 101 amigos, exatamente.

Agora seja yk o numero de amigos do fulano k (k=1,2,3,...,102) -- ou seja,
a lista dos numeros yk seria aquela lista lah em cima. A soma dos yk tem
que ser par (afinal, quando voce soma os yk's, voce estah contando o
"numero de amizades", soh que em dobro, porque cada amizade eh contada duas
vezes -- estamos fazendo a hipotese aqui de que, se A eh amigo de B, entao
B eh amigo de A). Como 68+68+68+69+69+69+...+101+101+101 = 17x169 eh impar,
esta nao eh uma configuracao possivel. Acabou.

Abraco,
   Ralph

2012/6/5 Mauricio de Araujo 

> Amigos, gostaria de uma luz para fazer o problema abaixo:
>
> Cada um dos 102 estudantes ´e amigo de pelo menos 68 outros alunos. Prove
> que existem quatro
> estudantes com o mesmo n´umero de amigos.
>
> --
> --
> Abraços
> oɾnɐɹɐ ǝp oıɔıɹnɐɯ
> *momentos excepcionais pedem ações excepcionais*
>
>


[obm-l] Re: [obm-l] Congruência

2012-06-04 Por tôpico Ralph Teixeira
Vai parecer magica, porque eu fiz dum jeito meio feioso e depois arrumei:

Queremos mostrar que:
2^(2p-3)-2^(p-2) + 72 = 0 (mod 100)

Farei x=2^(p-4) (note que p>=4), para enxergar isso melhor:
32x^2-4x+72=0 (mod 100)

Magiquinha:
32x^2-4x-28=0 (mod 100)

Agora dah para fatorar!
4(8x^2-x-7)=0 (mod 100)
4(8x+7)(x-1)=0 (mod 100)

Agora, como x=2^(p-4)=16^((p-4)/4), entao x=1 (mod 5).
Portanto, 8x+7=15=0 (mod 5)  e x-1=0 (mod 5). Isto mostra que (8x+7)(x-1)=0
(mod 25), e portanto acabou.

Abraco,
Ralph

2012/6/4 Alan Pellejero 

> Boa tarde,
>
> Gostaria de pedir o auxílio dos senhores para mostrar que:
>
>  2^(2p-3) + 72 .=. 2^(p-2) (mod 100), sendo "p" um múltiplo de quatro
> positivo.
>
> Nota: o símbolo .=.  significa côngruo.
>
> Agradeço a ajuda.
>
>
>
>
>


[obm-l] Re: [obm-l] Ajuda em combinatória

2012-06-04 Por tôpico Ralph Teixeira
Bom, é difícil dizer se a ordem em que as bolas são colocadas nas caçapas
importa ou não. Parece-me que não, então decida como colocar as brancas de
C15,8 maneiras, depois as pretas de C10,8 maneiras, para um total de
C15,8.C10,8=289 575 maneiras. Acho.

Se a ordem importar, primeiro escolha a ordem DAS CORES no encaçapamento:
C9,2 maneiras de ordenar as pretas, as brancas ficam no que sobrar. Agora
escolha as caçapas de cada encaçapamento, 9^9 opções. Total, (C9,7).9^9=13
947 137 604 maneiras. Acho.

(Na mesa em que eu jogava quando eu era criança, as caçapas tinham redinhas
estreitas, então olhando por fora dava para deduzir a ordem em que as bolas
tinha caído... Mas, naquelas mesas de bar onde todas as caçapas levam a uma
gaveta, a ordem quase que desaparece. Isto dito, nunca vi uma mesa com 9
caçapas então Ah, droga, estou sendo prolixo, mante este parágrafo pro
lixo. Acho. )

Abraçho,
 Ralph

2012/6/4 marcone augusto araújo borges 

>  De quantas maneiras podemos colocar 7 bolas de bilhar brancas e duas
> pretas em 9 caçapas?(algumas caçapas podem permanecer vazias e as caçapas
> são consideradas distintas).
>
> Se fossem todas brancas ou todas pretas(mais fácil) seriam C17,8 maneiras?
>


Re: [obm-l] Problema Sobre Contagem de elementos ,

2012-06-02 Por tôpico Ralph Teixeira
Nao ha equivoco! Soh faltou o ultimo passo:

Como
3% dos funcionarios sao mulheres menores de idade
15% dos funcionarios sao menores de idade
entao
3%/15%=20% dos menores de idade sao mulheres. Resposta (E).

Abraco,
Ralph

P.S.: Eu costumo fazer isto montando uma tabelinha, comecando por um numero
arbitrario toal de elementos. Ficaria mais ou menos assim (esta tabela soh
vai ficar boa em fonte de tamanho constante):

xxx..Mai.Men..Tot
Hom...60..12...72
Mul...25..03...28
Tot...85..15..100

Eu montaria a tabela usando os dados do enunciado na seguinte ordem: 100 ->
72,28 -> 60,12 -> 85,15 -> 25,03. Pronto, agora eh facil responder qualquer
pergunta: 3/15 dos menores sao mulheres; 3/28 das mulheres sao menores; 3%
(do total) sao mulheres menores de idade.

2012/6/2 Paulo Barclay Ribeiro 
>
> Prezados , Bom dia.
> peço ajuda para o seguinte Problema.não ,
>
> ( Unirio) Um engenheiro ao fazer o levantamento do quadro de pessoal de
uma fábrica,obteve os seguintes resultados:
> 28% são mulheres
> 1\6 dos homens são menores de idade
> 85% dos funcionários são maiores de idade.
> Qual a percentagem dos menores de idade que são mulheres.
> As opções são : a)30% ;b)25% ;28% ; d)23% ; e)20%
>
> A solucao que imaginei foi:
>
> logo H(homens)=100%-28%=72%;H=72%
>
> Se 1\6 são homens menores de idade5|6 serão homens são maiores de
idade
> Como H=72%--->H menore de idade = 12% e homens maiores =60% .
>
> Se 85%   são funcionarios (Homem   , e Mulher) maiores de idade>15%
serão funcionarios menores de idade.
> Mas se Homens menores de idade são 12% então mulheres menores de idade
serão 15%-12%=3%!!!
>
> Por favor me indiquem onde estou me equivocando.
>
> Desde já agradeço a atenção de vocês.
>
> Um abraço
>
> paulo


[obm-l] Re: [obm-l] combinatória

2012-05-22 Por tôpico Ralph Teixeira
Eu ia tentar escolher dois numeros a e b com a+b<100, depois tomar
c=100-(a+b) e no fim eliminar os casos onde ha uma repeticao Mas entao
resolvi fazer no braco mesmo:

Se o menor numero for 1, basta escolher agora dois numeros (maiores que 1)
que somem 99. Pode ser 2+97, 3+96,...,49+50. Total: 48 maneiras.
Se o menor numero for 2, basta escolher agora dois numeros (maiores que 2)
que somem 98. Pode ser 3+95, 4+94,...,48+50. Total: 46 maneiras.
Se o menor numero for 3, basta escolher agora dois numeros que somem 97.
Pode ser 4+93, 5+92,...,48+49. Total: 45 maneiras.
Se o menor numero for 4, basta escolher blah blah blah 96. Pode ser 5+91,
..., 47+49. Total: 43 maneiras.
Se 5, soma 95, de 6 a 47, total 42 maneiras.
Se 6, S=94, 7 a 46, sao 40.
...
Se 31, S=69, 32+37 ou 33+36 ou 34+35, 3 jeitos.
Se 32, S=68, 33+35, 1 maneira.
Se 33, nao dah mais.

Entao o numero que a gente quer eh 48+46+45+43+42+40+39+...+3+1+0. Separe
em duas PAs de algum jeito e corra pro abraco. Pode ser por exemplo:

(0+3+6+9+...+48)+(1+4+7+10+..+46)=3*16*17/2+16*47/2=8*98=784.

Acertei?

Abraco,
  Ralph
2012/5/22 Fabio Bernardo 

> Oi amigos,
>
> Preciso de uma ajudinha.
>
> Considere o conjunto {1,2,3,4,...,100}
> De quantas maneiras podemos escolher 3 elementos distintos
> de modo que a soma deles seja exatamente igual a 100?
>
> a) 781
> b) 782
> c) 783
> d) 784
> e) 785
>
>
>


[obm-l] Re: [obm-l] RE: [obm-l] Re: [obm-l] Polígono convexo e ângulos internos

2012-05-21 Por tôpico Ralph Teixeira
Neste caso, acho que é verdade sim. Não sei se dá para formalizar como a
seguir, mas tentemos...

Suponha por contradição que todos os ângulos do seu polígono são menores
que 180 graus, mas ele não é convexo. Você pode perfeitamente supor que não
há ângulos de 180 graus (se houver, elimine os vértices onde isto acontece,
e junte os lados correspondentes num só).

Então você pode achar um segmento de reta com vértices dentro do polígono
mas que não está completamente contido no polígono. Este segmento pode ser
subdividido em pedaços pelos lados do polígono -- pelo menos um dos pedaços
estará completamente do lado de fora (exceto pelos seus vértices, que
estarão nos lados do polígono).

Então seja AB um tal segmento (todo do lado de fora, exceto pelos vértices
A e B sobre os lados do seu polígono original). Há dois polígonos formados
pelos lados do polígono original (interrompidos pelos pontos A e B), mais
este segmento, sendo que um deles terá como ângulos internos os
REPLEMENTARES do polígono original (exceto em A e B, cujos ângulos me são
completamente desconhecidos). Seja P este último polígono, digamos que ele
tenha N lados.

Bom, este polígono será simples (os lados do polígono original não se
intersectavam, e este segmento AB não intersecta o polígono original exceto
nos vértices A e B). Seus ângulos são todos maiores que 180 graus (bom, N-2
de seus ângulos, pois não sabemos nada sobre os ângulos em A e B), então a
soma dos seus ângulos será maior que 180(N-2). Mas a soma dos ângulos
internos de um polígono simples é 180(N-2), absurdo!

Reconheço que esta ideia aí em cima não está muito formal ainda -- teríamos
que estabelecer melhor:
i) A existência do segmento AB (tenho certeza que é possível, mas ele é
chato de definir formalmente, até porque o segmento original não contido no
polígono original poderia ter pedaços inteiros de alguns lados);
ii) A existência do tal polígono P (eu vejo perfeitamente que um dos dois
polígonos criados por A e B tem como ângulos os replementares do original,
mas isto deveria ser formalizado usando alguma espécie de orientação do
polígono original).

Abraço,
   Ralph

2012/5/21 Paulo Argolo 

>
> Caro Ralph ( e demais colegas )
>
> Gostaria de me referir somente aos polígonos planos simples.
> Nesse caso, o teorema é válido?
> Abraços.
> Paulo
> ___
>
> Date: Sun, 20 May 2012 21:59:30 -0300
> Subject: [obm-l] Re: [obm-l] Polígono convexo e ângulos internos
> From: ralp...@gmail.com
> To: obm-l@mat.puc-rio.br
>
> Isto eh falso. Pegue, por exemplo, um icosagono estrelado (ligando os
> pontos "de 3 em 3"). Abraco,   Ralph
>
> 2012/5/20 Paulo  Argolo 
>
> Caros Colegas,
>
>
>
> Aproveitando a resposta dada sobre a questão "Paralelogramo é convexo",
> formulo nova questão:
>
>
>
> — Mostrar que um polígono é convexo se, e somente se, qualquer de seus
> ângulos internos mede menos de 180 graus.
>
>
>
> Defino:  Um polígono ( = região poligonal) é convexo se, e somente se,
> qualquer segmento de reta com extremidades pertencentes ao polígono está
> contido no polígono.
>
>
>
> Abraços do Paulo.
>
>
>
> =
>
> Instru�ões para entrar na lista, sair da lista e usar a lista em
>
> http://www.mat.puc-rio.br/~obmlistas/obm-l.html
>
> =
>
>
>
>
> =
> Instru�ões para entrar na lista, sair da lista e usar a lista em
> http://www.mat.puc-rio.br/~obmlistas/obm-l.html
> =
>


[obm-l] Re: [obm-l] Polígono convexo e ângulos internos

2012-05-20 Por tôpico Ralph Teixeira
Isto eh falso. Pegue, por exemplo, um icosagono estrelado (ligando os
pontos "de 3 em 3").

Abraco,
   Ralph

2012/5/20 Paulo Argolo 

> Caros Colegas,
>
> Aproveitando a resposta dada sobre a questão "Paralelogramo é convexo",
> formulo nova questão:
>
> — Mostrar que um polígono é convexo se, e somente se, qualquer de seus
> ângulos internos mede menos de 180 graus.
>
> Defino:  Um polígono ( = região poligonal) é convexo se, e somente se,
> qualquer segmento de reta com extremidades pertencentes ao polígono está
> contido no polígono.
>
> Abraços do Paulo.
>
> =
> Instru�ões para entrar na lista, sair da lista e usar a lista em
> http://www.mat.puc-rio.br/~obmlistas/obm-l.html
> =
>


Re: [obm-l] Ajuda em combinatoria

2012-05-19 Por tôpico Ralph Teixeira
A chave eh decidir onde vao os impares e onde vao os pares... quero dizer,
decidir qual das seguintes "formas" seu numero vai ser:
"Forma" 1: PPPIII
"Forma" 2: PPIPII
...
"Forma" ??: IIIPPP
(I= impar, P=par; tecnicamente, ??=C(6,3), mas isto nao interessa agora)
Isto dito, a gente vai ter que evitar com carinho colocar um 0 na primeira
posicao. Entao eu separaria em dois casos:

A) Com o primeiro algarismo impar:
"Formas" possiveis: basta escolher dois outros lugares para os I: C(5,2)
possibilidades.
Agora, para cada I ha 5 opcoes, e para cada P ha 5 opcoes.
Entao aqui ha um total de C(5,2).5^3.5^3 opcoes.

B) Com o primeiro algarismo par:
"Formas" possiveis: basta escolher dois outros lugares para os P: C(5,2)
possibilidades de novo.
Agora, para o primeiro P ha 4 opcoes, para cada I ha 5 opcoes, para os
outros dois P ha 5 opcoes cada.
Total: C(5,2).4.5^5

Total geral: (A)+(B)=C(5,2).5^5.(5+4)=281250

---///---

Outra maneira eh contar todas as sequencias com 3 digitos pares e 3 impares
(C(6,3)*5^6 opcoes) e retirar as que comecam por 0 (C(5,3)*5^5 opcoes).

Abraco,
 Ralph

2012/5/19 marcone augusto araújo borges 

>  Quantos numeros de 6 algarismos têm 3 algarismos pares e 3 impares?
>
> Tentei bastante,ainda não saiu.
> Se fossem algarismos distintos seria mais fácil.
>


[obm-l] Re: [obm-l] RE: [obm-l] RE: [obm-l] Dúvida Indução

2012-05-17 Por tôpico Ralph Teixeira
Cuidado: ao passar de n=k para n=k+1 no Passo de Inducao... o ultimo
termo "era" 3n-1, agora eh 3(n+1)-1=3n+2 -- nao eh questao de "somar
um no termo", eh "trocar n por n+1".

Abraco,
 Ralph

2012/5/17 Thiago Bersch :
> Então eu estava tentando fazer mas parava no mesmo ponto, fazia
> 2+5+8+...+(3n-1)+[(3n-1)+1], chegando aí eu me perco
>
> 
> From: joao_maldona...@hotmail.com
> To: obm-l@mat.puc-rio.br
> Subject: [obm-l] RE: [obm-l] Dúvida Indução
> Date: Mon, 14 May 2012 15:24:47 -0300
>
> Vamos dizer que para n respeite a formula
> Logo 2+4+6+...+2n=n.(n+1)
> Somando 2n+2
> 2+4+6+...+(2n+2=n(n+1)+2n+2=(n+1)(n+2) que respeita a formula
> Logo se vale para n, vale para n+1
> Como vale para 1, vale para 2, e  entao para 3, 4, 5...
> Vale para qualquer natural
>
> Tente fazer o segundo agora
> []s Joao
>
> 
> From: thiago_t...@hotmail.com
> To: obm-l@mat.puc-rio.br
> Subject: [obm-l] Dúvida Indução
> Date: Mon, 14 May 2012 01:09:39 -0300
>
>  2 + 4 + . . . + 2n.
>  2 + 5 + 8 + . . . + (3n-1).
> Bem eu sei que o primeiro irá dar n(n+1) e o segundo n(3n+1)/2
> O que em si eu não entendi o resultado
> O primeiro eu tentei fazer assim:
> 2+4...+2n
> +
> n+2n+(2n+1),  e fiquei parado nisso e o segunda também, gostaria de uma
> explicação passo-a-passo pois não entendo.

=
Instruções para entrar na lista, sair da lista e usar a lista em
http://www.mat.puc-rio.br/~obmlistas/obm-l.html
=


[obm-l] Re: [obm-l] problema difícil

2012-05-16 Por tôpico Ralph Teixeira
A resposta eh "nao", este nao eh o ponto que maximiza o angulo ACB, e
"sim", eh possivel resolver isso com Geometria "Cearense" (muito mais
elegante que G.A.!).

Para derrubar a conjectura, note que se AB for perpendicular aa reta
r, entao o ponto que minimiza o perimetro ACB claramente estah em AB
tambem, mas o angulo ACB eh 0, certamente nao eh um maximo.

Entao, para encontrar o ponto C que maximiza O ANGULO ACB,
geometricamente, pense assim:

-- Esqueca a reta por enquanto, e fixe o angulo x. O lugar geometrico
dos pontos D do plano tais que angulo(ACB)=x  eh um arco capaz, que
passa por A e B.
-- Agora, se x for pequeno, o arco capaz eh bem "aberto", chega beeem
longe de A e B, e certamente cruza a reta r. Ou seja, se x eh pequeno,
na intersecao do arco capaz com r voce encontra um ponto D tal que
angulo(ADB)=x.
-- Por outro lado, aa medida que voce aumenta x, o arco capaz vai
"diminuindo", e, no caso extremo em que x=180 graus, o arco capaz vira
o segmento AB, que nao corta a reta r.
-- Qual o maior x que faz o arco capaz cortar a reta r? Bom, serah um
arco capaz que eh TANGENTE aa reta r. Entao o que voce precisa fazer
eh encontrar um circulo que passa por A e B e eh tangente aa reta r. O
ponto de tangencia eh o ponto C procurado que maximiza o angulo ACB.
-- Construcao geometrica: note que, sendo P a intersecao da reta AB
com a reta r, temos PA.PB=PC^2 (usando a potencia do ponto P com
relacao ao circulo "magico" que eh tangente a r passando por A e B).
Entao, para construir C, voce pode estender AB ateh cortar r (e
encontrar P), construir a media geometrica de PA e PB do seu jeito
favorito, e marcar PC na reta r para encontrar C.
-- Uma maneira de achar a media geometrica: supondo que PA>PB, trace o
circulo de diametro PA, trace a perpendicular por B a AB, intersecte
ambos para achar E; note que PE^2=PA.PB, entao PE eh a distancia PC
que voce quer.

Fiz um Geogebrinha com esta construcao, mas acho que a lista nao deixa
passar arquivos GGB. :(

Abraco,
  Ralph

2012/5/16 Samuel Wainer :
> Considere uma reta r num plano. Considere dois pontos fixos A e B fora da
> reta e no no plano, de forma que estes pontos estejam no mesmo semi-plano
> determinado pela reta r.
> Seja C um ponto qualquer da reta, para que a distância do percurso AC CB
> seja mínima devemos refletir o ponto B pela reta r para B' e considerarmos o
> percurso reto AB' e a intersecção de AB' com a reta com r será C .
>
> Agora a questão é que o ângulo ACB será máximo quando o percurso AC CB for
> mínimo?  Tem como resolver isso só com geometria sem usar ga?

=
Instruções para entrar na lista, sair da lista e usar a lista em
http://www.mat.puc-rio.br/~obmlistas/obm-l.html
=


[obm-l] Re: [obm-l] Eu não entendo este problema

2012-05-14 Por tôpico Ralph Teixeira
Oi, Drechum.

Suponho que você seja de Cuba? Olá, bemvindo!

Acho que ele quer o raio da circunferência da figura (veja anexo),
onde F é o centro.

Um jeito de fazer é usar Pitágoras. Seja EF=FA=R o raio pedido. Note
que AG=2+raiz(3) (lado do quadrado mais altura do triângulo
equilátero), então FG=2+raiz(3)-R. Então no triângulo FGE temos:

(2+raiz(3)-R)^2+1^2=R^2

Resolvendo, encontramos R=2.

---///---

Outra maneira mais geométrica é notar que:
(i) FA e BE são paralelos (por simetria);
(ii) FB e AE são perpendiculares (pois FA=FE e BA=BE, então FB é a
mediatriz de AE).
Portanto, FABE é um losango, e assim FA=AB=2.

Abraço,
Ralph
2012/5/13 drechum :
> ABC é um triângulo eqüilateral; BCDE é um quadrado 2 lateral construído
> exteriormente ao triângulo. Os vértices A, D e E que eles pertencem à mesma
> circunferência. Acha o valor do rádio da circunferência.

=
Instruções para entrar na lista, sair da lista e usar a lista em
http://www.mat.puc-rio.br/~obmlistas/obm-l.html
=


[obm-l] Re: [obm-l] Eu não entendo este problema

2012-05-14 Por tôpico Ralph Teixeira
Acho que a figura não foi estou tentando de novo.

2012/5/13 drechum :
> ABC é um triângulo eqüilateral; BCDE é um quadrado 2 lateral construído
> exteriormente ao triângulo. Os vértices A, D e E que eles pertencem à mesma
> circunferência. Acha o valor do rádio da circunferência.
<>

Re: [obm-l] Binomais de m+n tomados p a p

2012-05-08 Por tôpico Ralph Teixeira
Oi, Ruy.

Para mim, o argumento combinatorio (separando em duas classes, etc.)
eh o mais elegante, e (para mim) mais do que serve como demonstracao.

Se voce realmente quiser fazer por inducao... bom, vamos supor que
C(n,p) seja definido indutivamente do jeito que a gente monta o
Triangulo de Pascal, isto eh:

C(0,0)=1 e C(0,p)=0 se p inteiro nao-nulo
C(n,p)=C(n-1,p)+C(n-1,p-1) para n=1,2,3,... e p inteiro qualquer.
(Consequencia mais ou menos imediata, que pode ser mostrada por
inducao se desejado: C(n,p)=0 se p<0 ou p>n)

Entao vamos provar o seu negocio por inducao em N=m+n onde m,n sao
inteiros nao-negativos.

a) Se N=0, entao tem de ser m=n=0. Entao fica C(0,p)=SOMA
C(0,k).C(0,p-k). Esta soma vai ser sempre nula, EXCETO quando p=0 e
entao temos um termo C(0,0).C(0,0) quando k=0. Ou seja, se p=0 a soma
dah 1=C(0,0), e se p<>0 dah 0=C(0,p). Funcionou.

b) Agora suponha que a propriedade eh valida para todos os pares (a,b)
com a+b=N, isto eh:
C(N,p)=C(a+b,p) = SOMA (C(a,k) x C(b, p-k)) (onde o somatorio eh em k).

Entao, se tivermos um par m+n=N+1, vem:
C(N+1,p)=C(N,p)+C(N,p-1)=C(m+n-1,p)+C(m+n-1,p-1)= (usando a hipotese
de inducao com a=m e b=n-1)
=SOMA(C(m,k).C(n-1,p-k) + SOMA(C(m,k).C(n-1,p-1-k)) =
= SOMA (C(m,k).(C(n-1,p-k)+C(n-1,p-1-k)))=
=SOMA(C(m,k).C(n,p-k))
mostrando que a propriedade eh valida para todos os pares (m,n) com m+n=N+1.

Por inducao em N, acabou.

(Note que, para mim, o indice k do somatorio vai de -Inf a +Inf, ou de
0 a p, em todos os somatorios -- sempre que ele sair das combinacoes
"usuais", minha definicao diz que a combinacao correspondente eh 0,
nao afetando os somatorios. Alias, isto tem que ser pensado assim para
enunciar a propriedade do jeito que voce colocou -- se voce insistir
que C(a,-1) e C(a,a+1) nao existem e nao podem ser escritas, entao a
propriedade deveria ser:

C(m+n,p) = somatório(max(0,p-n)<=k<=min(p,m))  (C(m,k) . C(n, p-k))

o que, convenhamos, eh um porre. :) )

Abraco,
  Ralph

2012/5/8 ruy de oliveira souza :
> Faz muitos anos que não uso indução, estou apanhando para demonstrar que
> C(m+n,p) = somatório(0<=k<=p)C(m,k) x C(n, p-k).  O argumento de separarmos
> em duas classes  m e n para para combinarmos todos os agrupamentos  com 0
> elemtos da classe com m e p elemtos da classe  com n ou agruparmos de todas
> as maneiras 1 elemento da classe com m e p -1 elementos da classe com  n
> ou...ou p elemtos da classe  com m e nenhum elemento da classe  com n,
> fazendo uso do princípio fundamental da contagem é válido como demonstração?
> Alguém consegue por indução??? Agradeço antecipadamente a quem por ventura
> responder. Abraço.

=
Instruções para entrar na lista, sair da lista e usar a lista em
http://www.mat.puc-rio.br/~obmlistas/obm-l.html
=


Re: [obm-l] Probabilidade

2012-05-07 Por tôpico Ralph Teixeira
Ha uma ordem especifica para a diferenca, tipo "vermelho - branco", ou
eh sempre "maior - menor"? Vou supor este ultimo.

De um jeito ou de outro, eu faria uma tabelinha com as 36
possibilidades equiprovaveis:

\ 1 2 3 4 5 6 <-(primeiro dado aqui)
1 0 1 2 3 4 5
2 1 0 1 2 3 4
3 2 1 0 1 2 3
4 3 2 1 0 1 2   <---diferencas dentro da tabela
5 4 3 2 1 0 1
6 5 4 3 2 1 0
^ (segundo dado aqui)

Entao, sendo D a diferenca, vem:
Pr(D=0)=6/36
Pr(D=1)=10/36
Pr(D=2)=8/36
...

Ou seja, diferenca de 1 eh o mais provavel, com probabilidade 10/36.

(Se fosse sempre vermelho - branco, seria Pr(D=0)=6/36, jah que D=1,
D=2, etc, se dividiriam em duas probabilidades menores.)

Abraco, Ralph

P.S.: Tecnicamente, frequencia e probabilidade nao sao exatamente a
mesma coisa Entao, nao dah para dizer que D=1 SERAH o mais
frequente Eh verdade que se voce jogar os dados MUITAS vezes, eu
aposto MUITO dinheiro que D=1 serah a mais frequente -- mas certeza
mesmo nao ha. O que dah para dizer COM CERTEZA eh que 1 eh o valor de
D com maior PROBABILIDADE, que foi o que fiz acima, e acho que era
isso que a questao queria.

2012/5/7 Pedro Júnior :
> Se lançarmos diversas vezes dois dados, um vermelho e um branco,
> e cacularmos a diferença entre os pontos obtidos, quais as diferenças
> mais frequêntes?
>
> --
>
> Pedro Jerônimo S. de O. Júnior
>
> Professor de Matemática
>
> Geo João Pessoa – PB
>
>

=
Instruções para entrar na lista, sair da lista e usar a lista em
http://www.mat.puc-rio.br/~obmlistas/obm-l.html
=


[obm-l] Re: [obm-l] triângulo esférico

2012-05-03 Por tôpico Ralph Teixeira
Pense o que acontece se voce sair do polo sul, andar 1km para N, 1 km
para E, e 1 km para S.

(Agora, tecnicamente, nao ha ursos no polo sul, entao o problema nao
funciona do jeito que ele disse. Tinha que comecar 1 km para o SUL.)

Abraco,
Ralph

2012/5/3 Marco Antonio Leal :
> Durante uma aula, meu professor comentou sobre um urso que se encontra em um
> ponto do planeta terra e caminha 1 km em direção ao norte, para, e vira 90
> graus a direita onde caminha mais um km, para novamente, vira noventa graus
> a direita e caminha mais um km, entretanto, para no ponto inicial de
> partida. Qual a cor do urso. ele me disse que isso só era possivel se
> considerarmos triângulos esféricos. Isso é possivel?

=
Instruções para entrar na lista, sair da lista e usar a lista em
http://www.mat.puc-rio.br/~obmlistas/obm-l.html
=


[obm-l] Re: [obm-l] Provar por indução

2012-05-02 Por tôpico Ralph Teixeira
Em primeiro lugar, note que 1/2=1/3+1/6. Dividindo por k dos dois
lados, note que 1/(2k)=1/(3k)+1/(6k)

Então usando esta ideia, você pode ir abrindo assim:

1=1/2+1/3+1/6 (use k=3 para abrir o 1/6)
1=1/2+1/3+1/9+1/18 (use k=9 para abrir o 1/18)
1=1/2+1/3+1/9+1/27+1/54 (use k=18 para abrir o 1/54)
1=1/2+1/3+1/9+1/27+1/81+1/162

Agora é só escrever isso formalmente, usando indução.

Abraço, Ralph.

P.S.: Em outras palavras:
1=1/2+1/3+1/9+1/27+1/81+1/243+...+1/3^k+1/(2.3^k)
o que podia ser provado usando simplesmente a fórmula da soma dos
termos de uma P.G. (na P.G. ali do miolo)

P.P.S.: Em outras palavras, em base 3:
(1/2)=(0....) = (0.11)+(0.001...)
Então
1=1/2+0.1+0.01+0.001+...+0....1+(0.1)/2

2012/5/2 marcone augusto araújo borges :
> Prove por indução que para cada numero natural p > = 3,existem p numeros
> naturais distintos dois a dois :
> n1,n2,...,np tais que
>
>
> 1/n1 + 1/ n2 ...+ 1/np = 1
>
> Essa complicou pra mim,conto com ajuda,agradeço desde já
>
>
>
>
>
>

=
Instruções para entrar na lista, sair da lista e usar a lista em
http://www.mat.puc-rio.br/~obmlistas/obm-l.html
=


[obm-l] Re: [obm-l] "Paradoxos" da Matemática

2012-04-27 Por tôpico Ralph Teixeira
Acho que um dos problemas mais comuns quando as pessoas pensam em
"infinito" é achar que, se algo vale para todo N, então deve valer
para "N=Infinito", seja lá o que isto for, como se "Infinito" fosse um
número natural... Acho que ambos as "contradições" que você cita caem
nesta categoria:

1) É verdade que qualquer conjunto com N pontos tem dimensão 0,
qualquer que seja N natural, por maior que seja N. Mas isto NÃO
implica que qualquer conjunto de pontos tenha dimensão 0. É
perfeitamente razoável que um conjunto INFINITO de pontos tenha
dimensão maior que 0. Por que haveria problema? :)

2) A "área sob a curva" numa integral é feita de uma infinidade de
"segmentos de reta" de área 0. É verdade que a área da união de N
conjuntos de área 0 dá sempre um total de 0, qualquer que seja N
natural, não interessa quão grande N seja. Mas isto não significa que
a área da união de **infinitos** segmentos de reta de área 0 tenha que
dar 0.

Note que (com a definição usual de integral dada nos cursos de cálculo
1) uma integral não é **definida** como um somatório infinito de áreas
zero; é um LIMITE quando N->+Inf de um somatório finito (com N termos)
de várias áreas, cada uma variando com N. É verdade que CADA área
tende a 0 quando N cresce, mas a QUANTIDADE delas aumenta com N, então
é possível que a integral dê um número positivo! Por exemplo:

-- Uma coisa é somar N parcelas todas iguais a 1/N e depois tomar
N->+Inf (a resposta é 1);
-- Outra coisa é tomar 1/N quando N->+Inf (que dá 0) e depois somar N
vezes (que não faz muito sentido porque você já tinha tomado N->+Inf,
mas alguns diriam que dá 0).

A ordem é importante! Os processos acima são simplesmente coisas
diferentes, não tem que ser iguais, não há contradição alguma. No caso
da integral, note a diferença entre:

-- SOMAR N ÁREAS RETANGULARES (que dependem de N), VER QUANTO DÁ (em
função de N), e DEPOIS tomar N->+Inf (a integral é uma coisa assim);
-- PEGAR CADA ÁREA, TOMAR N->+Inf, E DEPOIS SOMAR TODAS (o que não faz
sentido, já que serão infinitas áreas e eu não sei somar infinitos
números, ainda mais números que não param quietos; se você arrumar um
jeito de somar infinitos números, pode ser que isto dê 0 mesmo,
dependendo de como você somar... mas isto definitivamente NÃO é a
definição de integral do cálculo 1)

Vou inventar outros exemplos deste tipo (os raciocínios a seguir são FALSOS).

3) Como 1/N é positivo para todo N, então tomando N=+Inf concluímos
que 0 é positivo!
4) Como 0,9...9<1 (onde ali tem N noves), então 0,<1 (com
infinitos noves)!
5) Como a soma finita de N parcelas não depende da ordem das parcelas,
então a soma de uma série infinita também não!
(Por exemplo, é possível mostrar que
1-1/2+1/3-1/4+1/5-1/6+1/7-1/8+...=ln(2); mas
1+1/3-1/2+1/5+1/7-1/4+1/9+1/11-1/6+...=3.ln(2)/2)

Todos estas "aparentes" contradições são explicadas com o mesmo "mantra":
"SÓ PORQUE VALE PARA TODO N NATURAL, NÃO SIGNIFICA QUE VALE PARA "N=INFINITO""

Abraço,
 Ralph

2012/4/27 luiz silva :
> Prezados,
>
> Como a matemática lida com as seguintes questões :
>
> 1 - como pode algo sem dimensão dar origem a algo dimensional (ponto -
> curva)
>
> 2 - como pode um somatório infinito de áreas zero ter como resultado algo
> diferente de zero, como ocorre nas integrais ?
>
> Abs
> Felipe

=
Instruções para entrar na lista, sair da lista e usar a lista em
http://www.mat.puc-rio.br/~obmlistas/obm-l.html
=


[obm-l] Re: [obm-l] Maior potência tem maior base

2012-04-27 Por tôpico Ralph Teixeira
Sejam x e y números reais positivos.

Como já vimos num E-mail anterior, se a1<=b1, a2<=b2, ..., an<=bn, com
todos positivos, então a1a2...an<=b1b2...bn

(Tá, se eu me lembro direito tínhamos feito isso com a1y^n, então
x>y."). Então acabou!

Abraço,
 Ralph

P.S.: A contrapositiva da implicação "Se p, então q" é a implicação
"Se (não q), então (não p)". Apesar do nome parecer sugerir algum tipo
de conflito, lembre que a contrapositiva de uma implicação é
EQUIVALENTE à implicação original. Provou uma, provou outra.

2012/4/27 Paulo  Argolo :
> Caros Colegas,
>
> Como podemos provar que a desigualdade x^n > y^n implica x > y , sendo x e y
> números reais positivos, e n inteiro positivo?
>
>
> Abraços do Paulo.
> =
> Instruções para entrar na lista, sair da lista e usar a lista em
> http://www.mat.puc-rio.br/~obmlistas/obm-l.html
> =

=
Instruções para entrar na lista, sair da lista e usar a lista em
http://www.mat.puc-rio.br/~obmlistas/obm-l.html
=


Re: [obm-l] Desigualdade com radicais

2012-04-24 Por tôpico Ralph Teixeira
Tome a=x^n e b=y^n (com x e y positivos). Entao voce quer mostrar quea raiz 
n-esima de (x^n+y^n) eh menor que x+y, isto eh, que
x^n+y^n<=(x+y)^n
Se voce abrir o lado direito pelo binomio de Newton, fica facil.
Serve assim?
Abraco,Ralph
2012/4/24 ennius :> Prezados amigos da Lista:>> Como podemos 
provar que a raiz n-ésima de (a + b) é menor que soma das raízes n-ésimas de a 
e b?>> ( n é natural diferente de zero, a e b são números reais positivos.)>>> 
Abraços!> Ennius Lima> 
=> 
Instru�ões para entrar na lista, sair da lista e usar a lista em> 
http://www.mat.puc-rio.br/~obmlistas/obm-l.html> 
=
=
Instru��es para entrar na lista, sair da lista e usar a lista em
http://www.mat.puc-rio.br/~obmlistas/obm-l.html
=


Re: [obm-l] Produto de desigualdades

2012-04-24 Por tôpico Ralph Teixeira
Comece com n=2; suponha a1a2a1.

Agora eh facil generalizar para n desigualdades, basta usar o caso n=2
varias vezes. Assim:

a1:
> Caros Colegas,
>
> Sendo a_1 < b_1 , a_2 < b_2 , ... a_n < b_n , como provar que vale a 
> desigualdade abaixo?
>
> a_1 . a_2 . ... . a_n  <  b_1 . b_2 ... . b_n
>
> (Todos os números dados são reais positivos.)
>
>
> Abraços do Paulo
> =
> Instruções para entrar na lista, sair da lista e usar a lista em
> http://www.mat.puc-rio.br/~obmlistas/obm-l.html
> =

=
Instruções para entrar na lista, sair da lista e usar a lista em
http://www.mat.puc-rio.br/~obmlistas/obm-l.html
=


[obm-l] Re: [obm-l] RE: [obm-l] Re: [obm-l] Quadrado mágico

2012-04-23 Por tôpico Ralph Teixeira
A frase " o espaco solucao tem dimensao 3" eh um conceito de Algebra
Linear. Significa (mais ou menos) que tem 3 constantes arbitrarias na
solucao geral do sistema -- neste caso, a, b e S.

Em linhas bem gerais, eh o seguinte: se voce tiver um sistema com
14 equacoes "DE VERDADE", lineares e homogeneas, com 23 incognitas,
entao a solucao do sistema terah 23-14=9 graus de liberdade, isto eh,
9 das incognitas vao ficar indeterminadas e as outras 14 voce consegue
escrever em funcao dessas 9.

"Equacao Linear Homogenea" significa equacoes lineares com termo
independente nulo; em outras palavras, coisas do tipo
a1.x1+a2.x2+...+an.xn=0 onde a1, a2,..., an sao constantes e x1, x2,
..., xn sao as incognitas.
""DE VERDADE"" significa que as equacoes sao REALMENTE diferentes;
tipo, se a 6a equacao for a soma das 4 primeiras mais 2 vezes a 5a,
entao ela nao traz informacao adicional alguma (diz-se que ela eh
LINEARMENTE DEPENDENTE das outras) e deve ser descartada antes de
"contar" o numero de equacoes. Entao, quando eu digo "14 equacoes ""de
verdade""", estou dizendo "14 equacoes linearmente independentes",
isto eh, nenhuma delas pode ser escrita como combinacao linear (somas
com coeficientes) das outras.

Mas isso eh soh a ideia geral, coloquei porque achei que voce poderia
achar mais ou menos intuitivo -- as definicoes formais e o enunciado
correto do teorema estao em bons livros de Algebra Linear.

Abracao,
 Ralph

2012/4/23 marcone augusto araújo borges :
> Eu estava pensando em números positivos mesmo
> Não sei sei o q quer dizer ´´espaço solução tem dimensão 3``
> Mas ai eu deveria ler sobre isso
> Mais uma vez obrigado.
> 
> Date: Tue, 17 Apr 2012 15:21:51 -0300
> Subject: [obm-l] Re: [obm-l] Quadrado mágico
> From: ralp...@gmail.com
> To: obm-l@mat.puc-rio.br
>
> Não, não pode ser. Afinal, a definição de quadrado mágico consiste em 8
> equações lineares com 10 incógnitas (as incógnitas são as 9 entradas do
> quadrado e a soma S de cada linha ou coluna). No entanto, uma das 8 equações
> é linearmente dependente das outras (se a soma de cada uma das 3 linhas é S,
> e a soma de cada uma de 2 colunas é S, então a 3a coluna tem de somar S
> também), então são de fato 7 equações linearmente independentes com 10
> incógnitas. Portanto, o espaço solução tem dimensão 3. O conjunto das P.A.s
> tem dimensão 2 (basta saber o primeiro termo e a razão) -- tá, há as 9!
> permutações possíveis dos termos dentro do quadrado, mas isto não aumenta a
> dimensão... Então eu tenho quase certeza que há quadrados mágicos cujos
> termos não são P.A.s.
>
> Sejamos mais explícitos: se as minhas contas estiverem certas, o quadrado
> mágico genérico 3x3 com soma S é:
> a[1 -1 0; -1 0 1; 0 1 -1] + b[0 -1 1; 1 0 -1; -1 1 0] + S/3[1 1 1; 1 1 1; 1
> 1 1]
> (Usei notação Matlab -- ou seja, são matrizes 3x3 ali entre colchetes, com
> linhas separadas por ";").
>
> Em particular. hmmm... tomando a=1, b=4, S=0 vem [1 -5 4; 3 0 -3; -4 5
> -1], que não estão em P.A.
>
> (Se você quiser um exemplo onde todos os números são positivos, basta somar,
> digamos, 13, a cada entrada da matriz, o que corresponde a tomar S=39)
>
> Abraço,
>           Ralph
>
> 2012/4/17 marcone augusto araújo borges 
>
> Os  números de um quadrado mágico 3 x 3,escritos em ordem crescente ou
> decrescente,formam sempre uma PA?
> As somas dos elementos de uma linha ou coluna ou diagonal são todas iguais.

=
Instruções para entrar na lista, sair da lista e usar a lista em
http://www.mat.puc-rio.br/~obmlistas/obm-l.html
=


[obm-l] Re: [obm-l] Quadrado mágico

2012-04-17 Por tôpico Ralph Teixeira
Não, não pode ser. Afinal, a definição de quadrado mágico consiste em 8
equações lineares com 10 incógnitas (as incógnitas são as 9 entradas do
quadrado e a soma S de cada linha ou coluna). No entanto, uma das 8
equações é linearmente dependente das outras (se a soma de cada uma das 3
linhas é S, e a soma de cada uma de 2 colunas é S, então a 3a coluna tem de
somar S também), então são de fato 7 equações linearmente independentes com
10 incógnitas. Portanto, o espaço solução tem dimensão 3. O conjunto das
P.A.s tem dimensão 2 (basta saber o primeiro termo e a razão) -- tá, há as
9! permutações possíveis dos termos dentro do quadrado, mas isto não
aumenta a dimensão... Então eu tenho quase certeza que há quadrados mágicos
cujos termos não são P.A.s.

Sejamos mais explícitos: se as minhas contas estiverem certas, o quadrado
mágico genérico 3x3 com soma S é:
a[1 -1 0; -1 0 1; 0 1 -1] + b[0 -1 1; 1 0 -1; -1 1 0] + S/3[1 1 1; 1 1 1; 1
1 1]
(Usei notação Matlab -- ou seja, são matrizes 3x3 ali entre colchetes, com
linhas separadas por ";").

Em particular. hmmm... tomando a=1, b=4, S=0 vem [1 -5 4; 3 0 -3; -4 5
-1], que não estão em P.A.

(Se você quiser um exemplo onde todos os números são positivos, basta
somar, digamos, 13, a cada entrada da matriz, o que corresponde a tomar
S=39)

Abraço,
  Ralph

2012/4/17 marcone augusto araújo borges 

>  Os  números de um quadrado mágico 3 x 3,escritos em ordem crescente ou
> decrescente,formam sempre uma PA?
> As somas dos elementos de uma linha ou coluna ou diagonal são todas iguais.
>


[obm-l] Re: [obm-l] RE: [obm-l] RE: [obm-l] Re: [obm-l] RE: [obm-l] RE: [obm-l] Provar que é irracional...

2012-04-01 Por tôpico Ralph Teixeira
Bom, do jeito que eu escrevi seria f_0=0. Entao voce tinha razao quando
disse que eu estava errado.

Eu acho. :)

Abraco,
  Ralph

2012/4/1 marcone augusto araújo borges 

>  Esqueça.claro,f_0=2.Obrigado.
>
>  --
> From: marconeborge...@hotmail.com
> To: obm-l@mat.puc-rio.br
> Subject: [obm-l] RE: [obm-l] Re: [obm-l] RE: [obm-l] RE: [obm-l] Provar
> que é irracional...
> Date: Sat, 31 Mar 2012 01:01:42 +
>
> Era sim.
> f_0=0,não?
>
>  --
> Date: Sun, 25 Mar 2012 17:59:28 -0300
> Subject: [obm-l] Re: [obm-l] RE: [obm-l] RE: [obm-l] Provar que é
> irracional...
> From: ralp...@gmail.com
> To: obm-l@mat.puc-rio.br
>
> Defina a sequencia f_0=2 e f_(n+1)=raiz(2+f_n) para n=0,1,2,...
>
> Note que f_1=raiz(2) eh irracional (bom, espero que isto tenha sido
> demonstrado anteriormente).
>
> Agora, note que se f_(n+1) fosse RACIONAL, entao f_n=(f_(n+1))^2-2 tambem
> seria RACIONAL. Ou seja, se f_n eh IRRACIONAL, entao f_(n+1) eh IRRACIONAL.
>
> Assim, como f_1 eh irracional, por inducao, todos os f_n sao irracionais
> (n=1,2,3,...).
>
> Era isso?
>
> Abraco,
>   Ralph
>
> 2012/3/24 marcone augusto araújo borges 
>
>  Obrigado.Eu vi essa questão numa lista de indução.
> Vejo uma idéia de indução ai,mas,se não for abusar da sua boa vontade,como
> seria uma solução com um
> procedimento mais explicito de indução?
>
>
>   --
> From: joao_maldona...@hotmail.com
> To: obm-l@mat.puc-rio.br
> Subject: [obm-l] RE: [obm-l] Provar que é irracional...
> Date: Sat, 24 Mar 2012 19:34:57 -0300
>
>
> Bom, sendo f(x) = raiz(2 +raiz(2 + raiz(2+...), x vezes, é óbvio que
> f(x+1) > f(x), Logo o valor máximo é f(infinito), mas se x tende ao
> infinito, temos que f(x) = raiz(2 + f(x)), que elevando ao quadrado temos
> f(x) = 2, logo para qualuqer x diferente do infinito (que é o caso), f(x) <
> 2, além disso f(x) > 0 e f(x) >= f(1) = raiz(2) =~ 1.4
>
> Elevando ao quadrado desse modo:
> f(x) = raiz(2 +raiz(2 + raiz(2+...)  -> f(x)² - 2 = f(x-1) -> (f(x)²-2)²-2
> = f(x-2), repetindo isso x vezes temos ->
>  ((f(x)²-2)²-2)²-2...²-2)=0, que expandindo tem coeficiente lider
> 1 e termo independendo -2, logo pelo teorema das raízes racionais, se f(x)
> é racional, é -2, -1, 1, ou 2, absurdo, logo f(x) é irracional.
>
> []'s
> João
>
>  --
> From: marconeborge...@hotmail.com
> To: obm-l@mat.puc-rio.br
> Subject: [obm-l] Provar que é irracional...
> Date: Sat, 24 Mar 2012 21:56:30 +
>
> Como provar q raiz(2+raiz(2+raiz(2+...raiz(2)),generalizando para n
> raizes,é irracional?
>
>
>


[obm-l] Re: [obm-l] RES: [obm-l] Re: [obm-l] Dúvidas

2012-03-15 Por tôpico Ralph Teixeira
Mas quem disse que o proximo termo eh sin(60°)? ;)

Abraco,
Ralph

2012/3/15 Albert Bouskela :
> Olá!
>
>
>
> O enunciado da 2ª questão está completamente errado!
>
>
>
> sin(30°)/sin(15°) = 2cos(15°) = 1,93
>
>
>
> Supondo que o próx. termo da sequência seja sin(60°): sin(60°)/sin(30°) =
> 2cos(30°) = 1,73
>
>
>
> Pior: — A função “sin” é periódica (cresce e decresce periodicamente). Daí
> nunca poderá formar uma PG! E mais: — Toda PG é uma função polinomial,
> monotonamente crescente para razões maiores do que 1, e decrescente para
> razões menores do que 1. A função “sin” não é polinomial!
>
>
>
> Quem elaborou esta questão???
>
>
>
> Albert Bouskela
>
> bousk...@msn.com
>
>
>
> De: owner-ob...@mat.puc-rio.br [mailto:owner-ob...@mat.puc-rio.br] Em nome
> de Gabriel Merêncio
> Enviada em: 15 de março de 2012 09:47
> Para: obm-l@mat.puc-rio.br
> Assunto: [obm-l] Re: [obm-l] Dúvidas
>
>
>
> 1 - João atravessa o percurso BT, de 12 km, com velocidade média de 15 km/h,
> o que significa que ele leva 12/15 * 60 = 48 minutos. Daí, restam 42 minutos
> para a trajetória CB, a qual João percorre com velocidade média de 10 km/h.
> Em 42 minutos (ou seja, 42/60 = 7/10 de hora), João percorre uma distância
> de 7 km, que é o valor de CB.
>
>
>
> Agora note que o ângulo CBA é suplementar ao ângulo CBT, então os senos são
> equivalentes. Como o seno é cateto oposto sobre hipotenusa: 0,54 = AC/CB =>
> AC = 0,54 * 7 = 3,78. Alternativa (A), portanto.
>
>
>
> 2 - Podemos escrever seno de 30 como:
>
>
>
> sen(30º) = sen(15º + 15º) = sen(15º)cos(15º) + sen(15º)cos(15º) =
> 2sen(15º)cos(15º)
>
>
>
> Dividindo esse valor por sen(15º), que é o termo anterior, obtemos
> 2cos(15º). Alternativa (D).
>
> 2012/3/14 Vanessa Nunes de Souza 
>
>
> Olá, mas uma vez gostaria da ajuda dos colegas em algumas questões de
> concurso.
>
>
>
> 1-João se desloca diariamente de sua casa (ponto C) até o trabalho (ponto
> T), passando pelo (ponto B) em trajetórias retilíneas, conforme mostra a
> figura :
>
>
>
> Desenho em anexo
>
>
>
> Considere-se que, num determinado dia, João percorreu: - a distância CB, com
> velocidade média de 10 km/h; - a distância BT = 12 km, com velocidade média
> de 15 km/h;
>
> - toda essa trajetória em 1h30min.
>
> Se o seno do ângulo CBT vale 0,54, o segmento CA,
>
> perpendicular à reta AT, mede, em km:
>
> (A) 3,78
>
> (B) 3,29
>
> (C) 2,56
>
> (D) 2,14
>
>
>
> 2-A sequência (sen15º, sen30º,...) é uma progressão geométrica.
>
> A razão dessa progressão é igual a:
>
> (A)
>
> (B) 2
>
> (C) sen2°
>
> (D) 2cos15°
>
>
>
> Agradeço quem puder ajudar.
>
> Vanessa Nunes
>
>

=
Instruções para entrar na lista, sair da lista e usar a lista em
http://www.mat.puc-rio.br/~obmlistas/obm-l.html
=


Re: [obm-l] onde estou errado (derivadas)

2012-03-15 Por tôpico Ralph Teixeira
Argh favor trocar "o numerador se aproxima de 1" por "o numerador
se aproxima de -1". Tambem, eu deveria dizer:
lim_(h->0-) ([n+h]-[n])/h=+Infinito

Agora acho que acertei? :)

2012/3/15 Ralph Teixeira :
> Oi, Hermann.
>
> A definicao de derivada (do Calculo I), eh esta:
>
> lim_(h->0) (f(x+h)-f(x))/h
>
> Acho que eh isto que voce quer dizer com "limite incremental"? Pois
> bem, quando a gente diz "limite quando h vai para zero", note que h
> pode ir para 0 PELOS DOIS LADOS (positivo ou negativo). Ou seja, a
> definicao de derivada inclui taxas de variacoes usando pontos aa
> esquerda ou aa direita do ponto x dado.
>
> Se voce tomar f(x)=[x], voce nota que, nos pontos do tipo x=n com n
> inteiro, o limite de (f(n+h)-f(n))/h nao existe quando h se aproxima
> de 0 pela esquerda -- o numerador se aproxima de 1 e o denominador se
> aproxima de 0. Eh verdade que, quando h se aproxima de 0 pela DIREITA,
> o limite dah 0, mas isto nao basta para a derivada existir. Em suma:
>
> lim_(h->0+) ([n+h]-[n])/h=0
> lim_(h->0-) ([n+h]-[n])/h=-Infinito
> Portanto
> lim_(h->0) ([n+h]-[n])/h nao existe
>
> Entao a funcao f(x)=[x] NAO TEM DERIVADA nos inteiros. Ou seja, a
> letra (b) estah correta.
>
> Bom, o que eu falei acima estah correto, mas eu nao sei se era isto
> que voce estava perguntando... Tomara que eu tenha acertado!
>
> Abraco,
>         Ralph
>
> 2012/3/15 Hermann :
>> Senhores,
>>
>> (a) pela definição de derivada, limite da raiz incremental existir no ponto
>> ele é derivável, a função maior inteiro [[x]] é derivável em todos os seus
>> pontos
>>
>> (b) mas pelo teorema que diz: se é derivável é contínua, a função maior
>> inteiro [[x]] NÃO é derivável nos pontos inteiros??
>>
>> Gostaria de uma ajuda.
>>
>> Obrigado
>>
>> Hermann
>>

=
Instruções para entrar na lista, sair da lista e usar a lista em
http://www.mat.puc-rio.br/~obmlistas/obm-l.html
=


Re: [obm-l] onde estou errado (derivadas)

2012-03-15 Por tôpico Ralph Teixeira
Oi, Hermann.

A definicao de derivada (do Calculo I), eh esta:

lim_(h->0) (f(x+h)-f(x))/h

Acho que eh isto que voce quer dizer com "limite incremental"? Pois
bem, quando a gente diz "limite quando h vai para zero", note que h
pode ir para 0 PELOS DOIS LADOS (positivo ou negativo). Ou seja, a
definicao de derivada inclui taxas de variacoes usando pontos aa
esquerda ou aa direita do ponto x dado.

Se voce tomar f(x)=[x], voce nota que, nos pontos do tipo x=n com n
inteiro, o limite de (f(n+h)-f(n))/h nao existe quando h se aproxima
de 0 pela esquerda -- o numerador se aproxima de 1 e o denominador se
aproxima de 0. Eh verdade que, quando h se aproxima de 0 pela DIREITA,
o limite dah 0, mas isto nao basta para a derivada existir. Em suma:

lim_(h->0+) ([n+h]-[n])/h=0
lim_(h->0-) ([n+h]-[n])/h=-Infinito
Portanto
lim_(h->0) ([n+h]-[n])/h nao existe

Entao a funcao f(x)=[x] NAO TEM DERIVADA nos inteiros. Ou seja, a
letra (b) estah correta.

Bom, o que eu falei acima estah correto, mas eu nao sei se era isto
que voce estava perguntando... Tomara que eu tenha acertado!

Abraco,
 Ralph

2012/3/15 Hermann :
> Senhores,
>
> (a) pela definição de derivada, limite da raiz incremental existir no ponto
> ele é derivável, a função maior inteiro [[x]] é derivável em todos os seus
> pontos
>
> (b) mas pelo teorema que diz: se é derivável é contínua, a função maior
> inteiro [[x]] NÃO é derivável nos pontos inteiros??
>
> Gostaria de uma ajuda.
>
> Obrigado
>
> Hermann
>

=
Instruções para entrar na lista, sair da lista e usar a lista em
http://www.mat.puc-rio.br/~obmlistas/obm-l.html
=


Re: [obm-l] Raizes da unidade

2012-03-06 Por tôpico Ralph Teixeira
Para o primeiro problema, considere a=x.exp(iA), b=y.exp(iB) e c=z.exp(iC).
Note que Kr é a parte imaginária de S(r)=a^r+b^r+c^r.

Seja Q(w) o polinômio (mônico) de grau 3 cujas raízes são a, b e c. Note
que:

Q(w)=w^3-S.w^2+D.w-P

onde

S=a+b+c=S(1) é real pois sua parte imaginária, K1, é nula;
D=ab+ac+bc=(S(1)^2-S(2))/2 é real pois S(1) e S(2) são reais (já que
K1=K2=0);
P=abc=xyz exp(i(A+B+C)) é real pois A+B+C é múltiplo de pi (e x,y,z são
reais).

Em suma, Q(w) tem coeficientes reais. Agora, "lembre" que, para todo r
natural, temos:
S(r+3)-S.S(r+2)+D.S(r+1)-P.S(r)=0
ou seja
S(r+3)=S.S(r+2)-D.S(r+1)+P.S(r)
Como S(0)=3 é real, assim como S(1) e S(2), fica claro (já que os
coeficientes da recorrência acima também são reais) que S(n) é real para
todo n. Em outras palavras, Kn=0 para todo n natural.

Abraço,
Ralph

>
>> Em 3 de março de 2012 17:02, Heitor Bueno Ponchio Xavier <
>> heitor.iyp...@gmail.com> escreveu:
>>
>> Não estou conseguindo resolver os seguintes problemas:
>>>
>>> 1) Sejam x,y,z,A,B,C reais tais que (A+B+C)/π é inteiro.
>>> Defina Kr = (x^r) sen (rA) +(y^r) sen (rB) + (z^r) sen (rC)
>>> Prove se K1 = K2 = 0 então Kn = 0 para todo n>0
>>>
>>> 2)Seja (1+x+x²+x³+x^4)^496 = A0 +A1 x+A2x² +...+A1984 x^1984
>>> i)Determine MDC(A3,A8,...,A1983)
>>> ii)Prove que 10^340 >>
>>
>>
>


Re: [obm-l] Geometria

2012-02-20 Por tôpico Ralph Teixeira
Tem um argumento rapido para mostrar que o maximo existe, mas usa Analise:
as circunferencias C1, C2 e C3 sao conjuntos compactos; a funcao Area:
C1xC2xC3->R (que leva os pontos A, B e C na area do triangulo ABC,
incluindo area 0 para triangulos degenerados) eh continua. Toda funcao
continua definida num compacto tem de assumir maximo e minimo neste
conjunto. Acabou... :)

...mas eu concordo que este raciocinio nao deve ser muito satisfatorio para
quem estah no ensino medio... :( :( :(

Abraco,
  Ralph
2012/2/20 terence thirteen 

> Ou, de outra forma, se existir máximo então O é ortocentro.
> Boa pergunta: existe máximo?
>
> Outra questão é: e se quisermos minimizar o perímetro?
>
> Em 20 de fevereiro de 2012 11:31, Ralph Teixeira 
> escreveu:
> > Vou supor que o triangulo ABC de area maxima existe (o que eh bem
> razoavel,
> > e eh verdade, mas nao eh obvio usando soh geometria).
> >
> > Entao seja ABC esse triangulo de area maxima. Fixe o lado BC e pense nas
> > possiveis posicoes de A. Como o triangulo ABC tem area maxima, entao A
> eh o
> > ponto da circunferencia C1 mais "longe" de BC que voce puder arrumar. Em
> > outras palavras, a tangente a C1 por A eh paralela a BC. Ou seja, a reta
> OA
> > (que eh perpendicular aaquela tangente) eh perpendicular a BC.
> >
> > Em suma, AO eh (um pedaco da) altura do triangulo ABC.
> >
> > Analogamente, BO e CO sao perpendiculares aos lados AC e AB. Entao O eh o
> > ortocentro de ABC.
> >
> > (O que a gente provou eh que O ser ortocentro eh condicao NECESSARIA para
> > este triangulo ABC de area maxima, que me parece ser o que a questao
> > queria.)
> >
> > Abraco,
> >   Ralph
> >
> > 2012/2/20 Bob Roy 
> >>
> >> Olá ,
> >>
> >> Poderiam me ajudar nesta  questão ?
> >>
> >> Considere  C1 ,C2 e C3 três circunferências concêntricas de centro "O" e
> >> de raios respectivamentes iguais a :1 , 2 e 3 .  Sejam A , B e C pontos
> >> sobre  C1 , C2 e C3 , respectivamente . Como deve estar o centro "O"
> para
> >> que a área do triângulo  ABC seja máxima ?
> >>
> >>
> >> Agradeço qualquer resposta
> >>
> >> Bob
> >
> >
>
>
>
> --
> /**/
> 神が祝福
>
> Torres
>
> =
> Instru�ões para entrar na lista, sair da lista e usar a lista em
> http://www.mat.puc-rio.br/~obmlistas/obm-l.html
> =
>


Re: [obm-l] Geometria

2012-02-20 Por tôpico Ralph Teixeira
Oi, Bob.

Eu fiz uma hipotese "pesada": de que o triangulo ABC de area maxima existe.
Entao a primeira frase eh importante: eu supus que ABC JAH EH o triangulo
pedido, o de area maxima apoiado nos 3 circulos (bom, para ser exato, UM
DOS triangulos de area maxima, eu nunca supus que ele eh unico). Como ele
tem area maxima, se voce fixar B e C, o ponto A JAH TEM DE ESTAR na posicao
maximizante; analogamente, se voce fixar A e C, o ponto B jah tem que estar
na posicao maximizante. Analogamente para C. Ou seja, para este triangulo
ABC de area maxima, AO, BO e CO tem de ser alturas. Este foi o raciocinio
que eu usei, que depende fundamentalmente do triangulo existir. Ou seja, o
que provamos foi:

"SE ABC eh um triangulo de area maxima, ENTAO O eh o seu ortocentro."
ou seja
"O ser ortocentro eh NECESSARIO para que ABC tenha area maxima."

Agora, com meu raciocinio, nao sabemos a veracidade da reciproca, ou seja,
nao sabemos a veracidade de:
-- "SE O eh ortocentro de ABC, ENTAO ABC tem area maxima (serah ???)"
ou equivalentemente
-- "O ser ortocentro de ABC eh SUFICIENTE para concluir que ABC tem area
maxima (serah ???).

Melhorou?

Abraco,
 Ralph

Lembrete: dizer que p ==> q (SE p ENTAO q), eh o mesmo que dizer:
"p eh SUFICIENTE para q" (ou seja, se p acontece, eh garantido que q
acontece tambem)
que tambem eh o mesmo que dizer:
"q eh NECESSARIO para p" (ou seja, se q nao acontece, nao ha maneira de p
ocorrer)

2012/2/20 Bob Roy 

> Olá  Ralph ,
>
> Obrigado pela atenção , mas tenho uma dúvida :
>
> No momento em que foi fixado o lado BC ( por exemplo) e foi feita a
> análise de que AO tem como reta suporte a  altura relativa a BC , para que
> tenhamos a área máxima ; como posso garantir que BO e CO ( perpendiculares
> aos lados AC e BC) farão partes do mesmo triângulo ?
>
> É possível existir um triângulo de área máxima com apenas AO um pedaço da
> altura ? ou  seja , sem o ponto O como  ortocentro ?
>
> Foi isto que vc quis observar com NECESSÁRIA ?
>
> Abraços
>
> Bob
>
> Em 20 de fevereiro de 2012 10:31, Ralph Teixeira escreveu:
>
> Vou supor que o triangulo ABC de area maxima existe (o que eh bem
>> razoavel, e eh verdade, mas nao eh obvio usando soh geometria).
>>
>> Entao seja ABC esse triangulo de area maxima. Fixe o lado BC e pense nas
>> possiveis posicoes de A. Como o triangulo ABC tem area maxima, entao A eh o
>> ponto da circunferencia C1 mais "longe" de BC que voce puder arrumar. Em
>> outras palavras, a tangente a C1 por A eh paralela a BC. Ou seja, a reta OA
>> (que eh perpendicular aaquela tangente) eh perpendicular a BC.
>>
>> Em suma, AO eh (um pedaco da) altura do triangulo ABC.
>>
>> Analogamente, BO e CO sao perpendiculares aos lados AC e AB. Entao O eh o
>> ortocentro de ABC.
>>
>> (O que a gente provou eh que O ser ortocentro eh condicao NECESSARIA para
>> este triangulo ABC de area maxima, que me parece ser o que a questao
>> queria.)
>>
>> Abraco,
>>   Ralph
>>
>> 2012/2/20 Bob Roy 
>>
>>> Olá ,
>>>
>>> Poderiam me ajudar nesta  questão ?
>>>
>>> Considere  C1 ,C2 e C3 três circunferências concêntricas de centro "O" e
>>> de raios respectivamentes iguais a :1 , 2 e 3 .  Sejam A , B e C pontos
>>> sobre  C1 , C2 e C3 , respectivamente . Como deve estar o centro "O" para
>>> que a área do triângulo  ABC seja máxima ?
>>>
>>>
>>> Agradeço qualquer resposta
>>>
>>> Bob
>>>
>>
>>
>


Re: [obm-l] Geometria

2012-02-20 Por tôpico Ralph Teixeira
Vou supor que o triangulo ABC de area maxima existe (o que eh bem razoavel,
e eh verdade, mas nao eh obvio usando soh geometria).

Entao seja ABC esse triangulo de area maxima. Fixe o lado BC e pense nas
possiveis posicoes de A. Como o triangulo ABC tem area maxima, entao A eh o
ponto da circunferencia C1 mais "longe" de BC que voce puder arrumar. Em
outras palavras, a tangente a C1 por A eh paralela a BC. Ou seja, a reta OA
(que eh perpendicular aaquela tangente) eh perpendicular a BC.

Em suma, AO eh (um pedaco da) altura do triangulo ABC.

Analogamente, BO e CO sao perpendiculares aos lados AC e AB. Entao O eh o
ortocentro de ABC.

(O que a gente provou eh que O ser ortocentro eh condicao NECESSARIA para
este triangulo ABC de area maxima, que me parece ser o que a questao
queria.)

Abraco,
  Ralph

2012/2/20 Bob Roy 

> Olá ,
>
> Poderiam me ajudar nesta  questão ?
>
> Considere  C1 ,C2 e C3 três circunferências concêntricas de centro "O" e
> de raios respectivamentes iguais a :1 , 2 e 3 .  Sejam A , B e C pontos
> sobre  C1 , C2 e C3 , respectivamente . Como deve estar o centro "O" para
> que a área do triângulo  ABC seja máxima ?
>
>
> Agradeço qualquer resposta
>
> Bob
>


[obm-l] Re: [obm-l] Re: [obm-l] Dúvida de Lógica

2012-02-08 Por tôpico Ralph Teixeira
H nao sei nao. Vou usar "C" para "estah contido" e "E" para
"pertence a".

Concordo que toda inclusao de conjuntos pode ser pensada como uma
implicacao (bom, com um quantificador "para todo"). Afinal:

A C B
eh o mesmo que dizer
para todo x, xEA => xEB

Por isso, concordo que a Teoria dos Conjuntos e a Logica Matematica estao
profundamente emaranhadas Mas nem toda sentenca logica tem um conjunto
-- tem que ter uma variavel na sentenca logica!

Assim, se voce considera a proposicao "x^3+x+1>0", a ela estah associada o
conjunto A dos valores de x que satisfazem aquela proposicao. Mas aquela
proposicao eh ABERTA, isto eh, tem uma variavel x SOLTA nela, entao faz
sentido falar do conjunto dos x que a satisfaz. (Variavel SOLTA eh uma que
pode ser substituida por valores; uma proposicao ABERTA nunca eh nem V nem
F -- mas se voce substituir valores em todas as variaveis soltas de uma
proposicoa aberta, ai ela passa a ser V ou F, dependendo dos valores)

Agora, na proposicao "2>3", nao tem x. Entao nao faz sentido pensar no
conjunto A que voce cita, pelo menos nao diretamente. Voce poderia forcar a
barra e dizer que a proposicao eh r(x): 2>3, e entao eu concordo contigo,
mas tem que dizer algo explicito sobre a variavel que NAO aparece na
proposicao.

(Isto eh mais ou menos equivalente aa diferenca entre escrever f=5 ou
f(x)=5; se voce escreve do primeiro jeito, eu vou pensar que f eh um
NUMERO, igual a 5; do segundo jeito, eu penso que f eh a FUNCAO CONSTANTE e
igual a 5, cuja variavel x calhou de nao aparecer na expressao de f. Sao
objetos distintos.)

Idem para a segunda proposicao: "Todo brasileiro eh desonesto", nao tem
variavel solta, eh V ou F e acabou. Sim, ela eh equivalente a "Para todo y,
se y eh brasileiro entao y eh desonesto", mas note que nesta sentenca y nao
eh uma variavel SOLTA -- o "para todo y" da frente te proibe de substituir
valores em y. A proposicao "se y eh brasileiro entao y eh desonesto" eh
ABERTA e permite susbtituir valores para y, mas ela nao tem o "para todo".
Uma grande parte da confusao eh que a gente costuma pensar numa implicacao
e incluir o "para todo" nela sem escrever...

To sendo confuso e chato, eu sei mas eh que eu acho que o tempo que se
gasta em teoria dos conjuntos no ensino fundamental e medio podia ser MUITO
melhor aproveitado indo direto na logica -- e eu nao sei explixar tudo isso
em poucas linhas.

Abraco,
  Ralph

2012/2/7 Francisco Barreto 

> Saudações a todos!
> Seja A o conjunto dos objetos que satisfazem a propriedade r  de que 2 > 3.
> Seja B o conjunto dos objetos que satisfazem a propriedade s de que "Todo
> brasileiro é desonesto"
>
> Sabemos que A é o conjunto vazio. O conjunto vazio está contido em
> qualquer conjunto, incluindo B. Portanto os elementos de A (se você
> encontrar) devem satisfazer a  propriedade s de que todo brasileiro é
> desonesto.
>
>
>
> 2012/2/7 Pedro Chaves 
>
>>
>> Caros Colegas,
>>
>> Pode-se dizer que "2 > 3 => Todo brasileiro é desonesto"?   (O símbolo =>
>> indica implicação lógica.)
>>
>>
>> Sei que é verdadeira a proposição condicional "Se 2 > 3, então todo
>> brasileiro é desonesto", mas me parece que não existe implicação lógica.
>>
>> Desde já, muito obrigado.
>> Um abraço do Pedro Chaves!
>> =
>> Instruções para entrar na lista, sair da lista e usar a lista em
>> http://www.mat.puc-rio.br/~obmlistas/obm-l.html
>> =
>>
>
>
>
> --
> Sinceramente,
> Francisco Costa D. Barreto
>
>


[obm-l] Re: [obm-l] Dúvida de Lógica

2012-02-07 Por tôpico Ralph Teixeira
Sim, **logicamente**, a frase "2>3 ==> Eu sou o papa" estah correta. A
implicacao logica eh um simbolo DEFINIDO por esta tabela-verdade:

p  q  p => q
V V V
V F F
F V V
F F V

e, a principio, eh soh isso. Nada em "p=>q" **intrinsicamente** significa
causa, efeito, razao ou qualquer coisa assim. Para ilustrar isto, considere
as seguintes implicacoes VERDADEIRAS:

"Se estah chovendo na rua nas ultimas 2 horas, entao ela estah molhada"
(este eh o uso tipico: causa => efeito, antes => depois)
"Se voce estah de ressaca, entao voce bebeu." (mas aqui, efeito => causa
!!,  depois => antes !!)
"Se voce nasceu no Brasil, entao nasceu na America do Sul." (na minha
opiniao, nao ha causa/efeito aqui, nem antes/depois)

Agora, que se diga: a gente costumeiramente usa implicacoes logicas para
explicar raciocinios e fazer demonstracoes. Neste contexto:

i) Nao basta a implicacao estar correta. Afinal,
7>0 => O ultimo Teorema de Fermat eh verdadeiro
estah correta (e, ainda por cima, vale que 7>0)... mas isto nao demonstra o
UTF. Numa demonstracao, o que voce quer eh usar implicacoes logicas que
sejam OBVIAMENTE verdadeiras (seja de axiomas previamente estabelecidos, ou
de teoremas previamente estabelecidos, ou algo convincente). "Obviamente" e
"convincente" nao sao conceitos precisos, e dependem do leitor. :) :)
(Bom, eh possivel ser mais logico e formal e delimitar claramente o que sao
"raciocinios obvios previamente estabelecidos", mas entao tudo fica muito
chato e ninguem faz matematica assim, exceto quem se interessa diretamente
na logica como area de conhecimento.)

ii) Ha convencoes sobre como escrever implicacoes que vao um pouco alem da
logica formal. Por exemplo, para mostrar que 2ab<=a^2+b^2 para todo a e b
reais, alguem escreveria algo como:
(a-b)^2 >=0   =>
=>  a^2-2ab+b^2 >=0   =>
=>   a^2+b^2 >= 2ab

Tecnicamente, a frase p=>q=>r nao faz sentido... Voce poderia ter p=>(q=>r)
ou (p=>q)=>r, que sao construcoes raramente usadas e com significados
distintos. Mas, o que aquela pessoa QUIS dizer foi:
1. p
2. p=>q
3. q=>r
4. De 1, 2 e 3, concluo r.

ou seja

1. A afirmacao (a-b)^2>=0 vale para quaisquer a, b reais.
2. A implicacao (a-b)^2 >=0 => a^2-2ab+b^2 >=0 vale para quaisquer a, b
reais.
3. A implicacao a^2-2ab+b^2 >=0 =>  a^2+b^2 >= 2ab vale para quaisquer a, b
reais.
4. Entao, usando a propriedade de encadeamento das implicacoes logicas,
concluimos que a^2+b^2>=2ab para todo a, b reais.

Esta "propriedade de encadeamento" eh tao obvia na cabeca das pessoas que
ninguem escreve isto explicitamente. Pior, eh tao comum escrever p => q jah
imaginando que p eh verdadeira, que eh comum nem se dizer nada sobre a
validade de p. Por isso muita gente confunde "p=>q" com "p e q". Tambem,
por isso que raciocinios por contradicao deixam algumas pessoas tao
griladas ("como voce estah usando uma implicacao p=>q se p eh falsa?")...

Entao, em suma: p=>q significa nada mais nada menos do que aquela tabela
ali em cima (p eh F ou q eh V). p=>q nao diz nada sobre p sozinho, nem
sobre q sozinho. A implicacao significa qualquer coisa EXCETO p=V e q=F.
Nao ha necessariamente conexao fisica, temporal ou de causa/efeito entre p
e q.

Tudo isso na interpretacao LOGICA. O uso destas palavras na lingua
portuguesa pode ser ligeiramente diferente:)

Abraco,
   Ralph
2012/2/7 Pedro Chaves 

>
> Caros Colegas,
>
> Pode-se dizer que "2 > 3 => Todo brasileiro é desonesto"?   (O símbolo =>
> indica implicação lógica.)
>
>
> Sei que é verdadeira a proposição condicional "Se 2 > 3, então todo
> brasileiro é desonesto", mas me parece que não existe implicação lógica.
>
> Desde já, muito obrigado.
> Um abraço do Pedro Chaves!
> =
> Instruções para entrar na lista, sair da lista e usar a lista em
> http://www.mat.puc-rio.br/~obmlistas/obm-l.html
> =
>


Re: [obm-l] Probabilidade

2012-01-30 Por tôpico Ralph Teixeira
Oi, galera.

Dah para resumir a simetria do raciocinio do Ponce... Basta considerar os
eventos:

X = "A obtem mais caras do que B"
Y = "A obtem mais coroas do que B"

Note que X e Y nao podem ocorrer ao mesmo tempo (A tem apenas UMA moeda a
mais) mas pelo menos um deve ocorrer (A tem mais MOEDAS que B). Como, por
simetria, P(X)=P(Y), eh 50% para cada.

Abraco,
   Ralph
2012/1/30 Rogerio Ponce 

> Ola' Joao,
> chamemos de X(k) o numero de caras obtidas pelo jogador X em k lancamentos,
> e chamemos de P[z] a probabilidade do evento z ocorrer.
> Assim, nosso problema é calcular o valor de P[A(n+1) > B(n)]
>
> Agora, imagine que "B" tenha feito n lances, e que em seguida, "A" tambem
> tenha feito n lances.
> Ainda falta "A" fazer um lance, e as seguintes situacoes podem acontecer:
> 1) "A" ja' obteve mais caras que "B" com os n primeiros lances, e sua
> chance de ultrapassar B vale 100%.
> 2) "A" esta' empatado com "B", e tem 50% de chance de obter uma cara.
> 3) "A" tem menos caras que "B", e tem 0% de chance de ultrapassar B.
>
> Portanto, somando-se as probabilidades das 2 primeiras situacoes, obtemos
> P[A(n+1)>B(n)]  =  100% * P[A(n)>B(n)] + 50% * P[A(n)=B(n)]
>
> Sabemos que
>   P[A(n)B(n)] = 1
> Por simetria,
>   P[A(n)>B(n)]  =  P[B(n)>A(n)]
> de forma que
> 2*P[A(n)>B(n)]  + P[A(n)=B(n)] = 1
> ou seja,
> P[A(n)=B(n)]  =  1 -  2* P[A(n)>B(n)]
>
> Aplicando essa relacao 'a expressao anterior, obtemos
>  P[A(n+1)>B(n)] = 50%
>
> Ou seja, a probabilidade de "A" obter mais caras que "B" e' de 50%.
>
> []'s
> Rogerio Ponce
>
>
>
> 2012/1/18 João Maldonado 
>
>>
>>  Se A e B lançam respectivamente n + 1 e n moedas não-viciadas, qual é a
>> probabilidade Pn de que A obtenha mais “caras” do que B?
>>
>> []`s
>> Joao
>>
>
>


[obm-l] Re: [obm-l] Binômio de Newton

2012-01-18 Por tôpico Ralph Teixeira
Pense no triangulo de Pascal modulo 2, isto eh, soh marcando pares (0) e
impares (1):

1
11
101

10001
110011
1010101


...

Etc. Ha varios padroes a serem explorados ali, varias repeticoes de
triangulos anteriores, que podem ser demonstradas por inducao, por exemplo.
Em particular, voce soh pode ter ...1 na linha n se tiver 1001
na linha n+1. Entao voce pode tentar mostrar que 1...0001 ocorre sse na
linha n=2^s.

Ajuda?

Abraco,
 Ralph

2012/1/18 marcone augusto araújo borges :
> Seja n um inteiro positivo.Demonstrar que todos os coeficientes do
> desenvolvimento do binomio de Newton (a+b)^n sao impares se,e somente se,n
> é da forma 2^s - 1.
> Agradeço a quem puder ajudar


[obm-l] Fwd: [obm-l] Quantidade mínnima de tentativas

2012-01-16 Por tôpico Ralph Teixeira
-- Forwarded message --
From: Ralph Teixeira 
Date: 2012/1/16
Subject: RE: [obm-l] Quantidade mínnima de tentativas
To: obm-l@mat.puc-rio.br


Hugo, nao desanime! Com um pequeno ajuste, sua solucao ainda dah 22 testes!

(Eu tinha mandado isso para a lista, mas acho que foi barrado por
causa de um anexo)

Chutei o balde: coloquei as 70 opções para as 4 pilhas boas numa
planilha Excel, em ordem lexicográfica, para ver bem o que está
acontecendo. A cada passo, cobri as opções com os testes do Hugo
usando cores bonitinhas (mando a planilha por E-mail para quem quiser,
ajuda pacas a ver o que estamos fazendo).
Então percebi algumas coisas na solução do Hugo... Resumindo
cripticamente:

1. ABC e FGH (2 testes, eliminando 10 opções)
2a. (D ou E) com todos os pares em ABC (6t, -21op)
2b. (D ou E) com todos ou FGH (6t, -21op)
3a. ABE, BDE, CDE (2t, -9op)
3b. ABF, ABG (2t, -3op)
4. CFG, CFH, +CGH (-3t, -6op)
Total: 22 testes, 70 opções.

Note algo interessante: retirei ABH do passo 3b! Afinal, se ABH fosse
bom, as pilhas boas seriam ABCH, ABDH, ABEH, ABFH ou ABGH. Mas em cada
um desses casos, já teríamos uma combinação boa (respectivamente, em
1, 2a, 2a, 3b, 3b). Então ABH é desnecessário!

Por outro lado, adicionei CGH no passo 4. O motivo é que a solução do
Hugo não cobria os casos ACGH e BCGH, pelo menos não que eu tenha
visto.

Ou seja, deu 22 testes! Alguém dá menos?

Abraço,
          Ralph

2012/1/16 Hugo Fernando Marques Fernandes :
> Fiz assim:
>
> Considere três grupos: abc, de, fgh
>
> Testo o primeiro grupo (abc): se falhar este grupo tem 1 ou 2 pilhas boas.
> Testo o terceiro grupo (fgh): se falhar este grupo tem 1 ou 2 pilhas boas.
>
> Testo cada elemento do segundo grupo contra os pares formados pelos
> elementos dos outros grupos. São 12 testes, a saber:
> abd, acd, bcd, abe, ace, bce
> e tb fgd, fhd, ghd, fge, fhe, ghe
>
> Note que o segundo grupo (de) pode ter 0, 1 ou 2 pilhas boas.
> 1) Se tiver 0 então existe duas boas no grupo (abc) e duas boas em (fgh)
> 2) Se tiver 1 boa, então um dos grupos (abc) ou (fgh) tem duas boas (e o
> outro uma). Nesse caso, um dos doze testes acima teria funcionado. Logo, se
> não funcionou, podemos excluir essa hipótese.
> 3) Se tiver duas boas, então cada um dos grupos (abc) e (fgh) tem só 1 boa
> também.
>
> Se pensarmos primeiro no caso 3, podemos testar (ade), (bde), (cde) e uma
> vai funcionar.
>
> Se não funcionar, resta o caso 1, e os testes (abf), (abg) e (abh) devem
> funcionar - se não funcionar, então com certeza c funciona junto com fg ou
> fh, ou seja, temos mais dois testes, (cfg) e (cfh)
>
> Então no pior caso temos, 1+1+12+3+3+2 = 22
>
> Estou certo ou há alguma falha no raciocínio?
>
> Abs a todos.
>
> Hugo.
>
>
> Em 13 de janeiro de 2012 23:00, Breno Vieira 
> escreveu:
>>
>> Como eu ja disse, achei 23:
>>
>> 1. Teste ABC, se nao funcionar sabemos que pelo menos uma entre A, B e C
>> nao funciona.
>> 2. Teste as combinacoes entre DEFGH
>> (DEF,DEG,DEH,DFG,DFH,DGH,EFG,EFH,EGH,FGH), se nenhuma funcionar temos que
>> tres entre DEFGH nao funcionam, portando duas entre ABC e duas entre DEFGH
>> funcionam.
>> 3. Sabemos que AB, AC ou BC sao formadas por duas que funcionam e que pelo
>> menos uma entre D,E,F,G funciona, bastam entao mais 12 testes totalizando
>> 23.
>>
>> PS:Ainda tem mais outros dois algoritmos um pouco mais complicados que eu
>> fiz e que tambem chegam em 23. Quem da menos?
>
>

=
Instruções para entrar na lista, sair da lista e usar a lista em
http://www.mat.puc-rio.br/~obmlistas/obm-l.html
=


Re: [obm-l] Fobonacci

2012-01-16 Por tôpico Ralph Teixeira
Bom, eu não sabia disso mas agora que você falou...

A recorrência que define os termos de ordem ímpar da seq. de Fibonacci
pode ser obtida assim:

F(2n+1)=F(2n)+F(2n-1)
F(2n)=F(2n-1)+F(2n-2)
F(2n-2)=F(2n-1)-F(2n-3) (tô fazendo um esforço para só deixar os de
ordem ímpar do lado direito)

Então

F(2n+1)=3F(2n-1)-F(2n-3)

Agora deixa eu ver os "alegados" quadrados. Seriam:
F(n)5F(n)^2-4
1   1=1^2
2   16=4^2
5   121=11^2
13 841=29^2
......

Será que a sequencia da direita tem alguma ordem razoável... Digo,
olhando para 1,4,11,29..., qual é a recorrência? Hmmm, parece que cada
termo é 3 vezes o anterior menos ao anteanterior, de novo! Bom, mas
isso tudo é chute, vamos ver se a gente consegue MOSTRAR isso.

TEOREMA: Defina A(0)=1, A(1)=2 e A(n+1)=3A(n)-A(n-1) (n>=2). Defina
também B(0)=1, B(1)=4 e B(n+1)=3B(n)-B(n-1). Afirmo que:

i) 5A(n)^2-4=B(n)^2
ii) você já vai ver que preciso de algo mais aqui.

Prova: i) Para n=0 e n=1 é só verificar direto. Por indução, se a
propriedade vale para n=k e n=k-1, então:
5A(k+1)^2-4 = 5(3A(k)-A(k-1))^2-4 = 45A(k)^2-30A(k)A(k-1)+5A(k-1)^2-4
= (9B(k)^2+36)-30A(k)A(k-1)+B(k-1)^2

Ah, droga, eu não tenho a mínima ideia do que fazer com aquele
A(k)A(k-1)... Se fosse algo conhecido, razoável tipo, eu acho que
a coisa toda vai dar (3B(k)-B(k-1))^2, né? Para isso valer, eu
precisava que fosse 36-30A(k)A(k-1)=-6B(k)B(k-1), isto é, eu queria
que fosse 5A(k)A(k-1)=B(k)B(k-1)+6... Como provar isto? Façamos por
indução, ora! Então adicione lá no enunciado o seguinte:

ii) 5A(n)A(n-1)=B(n)B(n-1)+6

Esta propriedade claramente vale para n=1 e n=2. Agora o passo de
indução (note que estou usando (i) com n=k, então a indução é feita
com (i) e (ii) ao mesmo tempo!):

5A(k+1)A(k) = 5(3A(k)-A(k-1))A(k) = 15A(k)^2-5A(k)A(k-1) =
(3B(k)^2+12)-B(k)B(k-1)-6 =
= 3B(k)^2-B(k)(3B(k)-B(k+1))+6 = B(k)B(k+1)+6

Pronto! Este era o pedaço que faltava para terminar a indução em (i). Acabou!

Abraço,
Ralph

P.S.: Agora, para DESCOBRIR que estes números funcionam, dê uma olhada
na teoria de Equação de Pell, que ajuda a resolver coisas do tipo
5n^2-4=p^2.

2012/1/15 marcone augusto araújo borges :
> Provar q a equação x^2+y^2+z^2=3xyz tem infinitas soluções inteiras.
>
> Essa questão ja foi resolvida na lista
> Um colega tentou uma soluçao diferente:
> Fez y=n e z=1,encontrando x^2 - 3nx +n^2 +1=0
> x= (3n + - raiz(5n^2 - 4))/2
> 5n^2 - 4 deve ser um quadrado perfeito
> Tentei mostrar q existem infinitos valores de n para os quais 5n^2 - 4 é um
> quadrado perfeito e não consegui
> Mas o colega me informou q para n igual aos termos de ordem impar da
> sequencia de fibonacci, a referida expressão
> é um quadrado perfeito(1,2,5,13,34,...)
> Não sabemos provar
> Alguem poderia esclarecer?

=
Instruções para entrar na lista, sair da lista e usar a lista em
http://www.mat.puc-rio.br/~obmlistas/obm-l.html
=


[obm-l] Re: [obm-l] Re: [obm-l] Número de sextas-feiras 13

2012-01-13 Por tôpico Ralph Teixeira
Uma pergunta divertida ligeiramente relacionada: escolha um dia 13
aleatoriamente (todos os dias 13 de todos os meses de todos os anos com a
mesma probabilidade; suponha que o numero de anos eh BEM grande, mas todos
no calendario gregoriano para evitar complicacoes). Qual a probabilidade de
este dia ser uma 6a feira?

Se eu me lembro direito, surpreendentemente a resposta NAO EH 1/7 -- nem
pegando o "estado estacionario" quando o numero de anos vai para infinito!
Mas deixo o raciocinio exato como exercicio para o leitor...

(Traducao: to c/ preg.)

Abraco,
Ralph

2012/1/13 Pedro Nascimento 

> Vendo as classes de congruencia mod 7 temos:
> 0
> (0+31)=3  mod 7
> (3+29)=4  mod 7
> (4+31)=0  mod 7
> (0+30)=2  mod 7
> (2+31)=5  mod 7
> (5+30)=0  mod 7
> (0+31)=3  mod 7
> (3+31)=6  mod 7
> (6+30)=1  mod 7
> (1+31)=4  mod 7
> (4+30)=6  mod 7
>
> a classe que aparece mais eh zero, podemos atribuir a ela uma sexta-feira
> e temos assim que o maximo sao 3 meses mesmo.
>
> Em 13 de janeiro de 2012 09:32, Mauricio de Araujo <
> mauricio.de.ara...@gmail.com> escreveu:
>
>> Este ano de 2012 possuirá 3 sextas-feiras 13: em janeiro, abril e julho.
>>
>> Pergunta-se: Considerando anos bissextos, qual o número máximo de meses
>> com sexta-feira 13 pode haver em um mesmo ano? 2012 possuirá 3 meses.
>>
>> --
>> --
>> Abraços
>> oɾnɐɹɐ ǝp oıɔıɹnɐɯ
>> De Luto pelo Brasil até, no mínimo, 2014.
>>
>>
>>
>> "*NÃO À OBRIGATORIEDADE DO VOTO!*"
>>
>>
>>
>


[obm-l] Re: [obm-l] RE: [obm-l] Quantidade mínnima de tentativas

2012-01-11 Por tôpico Ralph Teixeira
Nao precisa testar 53 trincas nao! Rapidinho, arrumo um algoritmo com
38 testes...

 Sejam ABCDEFGH as 8 pilhas, seja X o conjunto das 4 que funcionam. Ha
C(8,4)=70 possibilidades para X.

Agora, voce testa ABC; se NAO funcionar, isto jah elimina 5
possibilidades para X (a saber, ABCD, ABCE, ABCF, ABCG, ABCH).
Teste CDE. Se nao funcionar, eliminamos ACDE, BCDE, CDEF, CDEG e CDEH.
EFG elimina mais 5; GHA elimina mais 5.
Tente agora ADF, CFH, EHB e GBD. Se nada disso funcionar, jah
eliminamos no total 40 possibilidades -- ateh aqui, todas disjuntas!
Explicitamente, sobram apenas as seguintes 30 possibilidades para X:
ABDE ABDH ABEF ABEG ABFG ABFH ACDG ACDH ACEF ACEG ACEH ACFG ADEG ADEH AEFH
BCDF BCDH BCEF BCEG BCFG BCGH BDEF BDFH BFGH CDFG CDGH CEGH DEFH DEGH DFGH

Mesmo que voce agora escolha uma trinca de cada um desses 30
conjuntos, seria um total de 8+30=38 testes. Mas ainda dah para
diminuir bastante, jah que varias trincas aparecem em varias dessas
quadras!

Quem dah menos? :)

Abraco,
   Ralph




2012/1/11 Felippe Coulbert Balbi :
>
>
> 
> From: joao_maldona...@hotmail.com
> To: obm-l@mat.puc-rio.br
> Subject: [obm-l] Quantidade mínnima de tentativas
> Date: Wed, 11 Jan 2012 18:15:19 -0200
>
> Dados um controle que precisa dde três pilha para funcionar, temos que em um
> bolo de 8 pilhas, 4 estão carregadas e 4 não
> Qual a quantidade mínima de tentativas para se assegurar que o controle
> funcione ?
>
> Lembro de um problema desses em uma edição da eureka , mas se não me engano
> tinha a ver com grafos,  o poblema é que eu não faço a minima idéia do que
> seja um grafoo :)
>
> Há alguma outra eplicação para   alguem como eu (ou até memo uma que involva
> grafos com um pouco mais de explicação?
>
>
> ___
>
> C(8,3)- C(4,3)+1= 8.7-4+1= 53

=
Instruções para entrar na lista, sair da lista e usar a lista em
http://www.mat.puc-rio.br/~obmlistas/obm-l.html
=


[obm-l] Re: [obm-l] Valor máximo para a soma dos senos

2011-11-24 Por tôpico Ralph Teixeira
Procure "derivadas parciais". :)

2011/11/24 João Maldonado 

>
>
> Recentemente vi um problema na lista sobre como calcular a soma  dos 3
> senos de um triângulo,  em que a resposta foi p/R
> Fiquei pensando então qual deveria ser o valor máximo para esta soma
>
> Fiz  assim:
>
> Dada uma circunferência de raio R,   e um dos  lados do triângulo, que
> chamaremos de w,  temos  necessariamente que um  ângulo (que chamaremos de
> W) já está determinado,  o valor máximo de  p então seria o  valor máximo
> da  soma dos outros lados.
>
> Como a² + b² -  2abcosW = w² ->  (a+b)² - 2ab(1+co sW) = w²
>
> Como w.h/2 = a.b.senW/2,  temos  que ab = h.w/senW
> Logo  (a+b)² = w² + 2hw(1+cosW)/senW
>
> Como   o lado e o ângulo já estão determinados,  e 1+cosW>0  e senW>0,   o
> valor máximo de a+b se dá no valor máximo da altura,  vem que  o triângulo
> é isósceles
>
> E  fazendo  y = 90-w/2
>
> Daí a soma dos 3 senos  é 2sen(y) + sen(2y)  que derivando dá 2(cos(y) +
> cos(2y)  e igualando a 0
>
> 2cos²(y) +  cos(y) - 1 = 0 ->  cos(y) = 1/2 ou  -1
>
> Substituindo  temos  que o valor máximo é cos(y) = 1/2  e a soma vale
> 3(3)^(1/2)/2
>
>
>
>
> Queria saber se há alguma derivada  de 2 variáveis,  no caso  a e b que
> desse o valor  máximo se sen(a) +  sen(b) + sen(a+b)
> Já  ouvi falar em integral dupla (na verdade só  ouvi falar, não faço a
> mínima idéia do que seja,  mas pensei  que tivesse alguma coisa a ver com a
>  integral de 2 variáveis)
> Entretanto procurei derivada dupla no google e não encontrei nada
>
> Existe isso?
>
> []'s
>
> João
>
>


Re: [obm-l] Questao de probabilidade: o sapo e a mosca

2011-10-16 Por tôpico Ralph Teixeira
Para mim, falta alguma especie de hipotese na distribuicao de probabilidade
a priori dos numeros nos envelopes -- nem que seja uma chutada inventada da
minha cabeca.

Por outro lado, reconheco que estou pensando no problema mais simples --
olho a distribuicao de probabilidade a priori, calculo a probabilidade do
outro envelope ser maior que 173 (dado que este eh 173, se for o caso),
decido. Deve haver raciocinios mais espertos que eu nao estou tentando
fazer, ou maneiras espertas de estimar esta a priori baseado em que sabemos
de programas de auditorio.

(Por exemplo: sabendo o que eu sei de programas de auditorio, aposto que
todos os numeros nos envelopes sao positivos; e nunca vi um numero
com mais de 9 algarismos num envelope, entao eu eliminaria estes da minha
distribuicao de probabilidade.)

Abraco,
 Ralph

2011/10/16 Jeferson Almir 

> Aproveitando o momento probabilistico vejamos tem este problema que estive
> pensando e nao consegui:
> Voce esta em um programa de auditorio. O apresentador tem dois envelope
> cada um com um numero dentro (numeros diferentes).
> Voce escolhe um envelope, abre e ve o numero 173.
>
> Ele te pergunta:voce gostaria de trocar o seu envelope por este outro ou
> nao?
>
> Ao final se o envelope que voce esolher tiver o maior numero voce ganha
> R$1.000.000.000,00 !!!
>
> E ai voce quer trocar ou nao?
> Em outras palavras, existe uma estrategia que ele possa adotar para este
> jogo que te de uma probabilidade estritamente
> maior que 1/2 de vencer?
>
>
> Em 15 de outubro de 2011 15:24, Ralph Teixeira escreveu:
>
>> Eu devia ter frisado melhor, que minha solucao imagina um sapo que estah
>> **SEMPRE FAMINTO** (que eh equivalente ao original, como expliquei na outra
>> mensagem). Por isso uso a cumulativa,  X>=3.
>>
>> Na sua solucao, temos que consertar um par de coisas:
>> i) Se eu entendi o que "consegue escapar" significa, entao os eventos nao
>> sao excludentes. Entao tem que subtrair a probabilidade do sapo comer 3
>> moscas *E* a mosca escapar.
>> ii) Se seguirmos aa risca o enunciado original, onde o sapo para de comer
>> moscas quando chega em 3, entao a distribuicao nao eh exatamente binomial --
>> a distribuicao binomial supoe independencia de experimentos, o que deixa de
>> ser verdade quando o sapo estah saciado. Para consertar isso, o jeito eh
>> mudar o sapo para um que nao para de comer -- foi o que eu fiz na minha
>> solucao -- mas ai temos de considerar 3, 4 ou 5 moscas comidas em 5.
>>
>> Entao ficaria:
>>
>> Pr(sexta mosca viver)=S+Pr(mosca escapar)-S.Pr(mosca escapar)
>>
>> onde S=B(3;5,0.5)+B(4;5,0.5)+B(5;5,0.5)=1/2. Note como usei que a 6a mosca
>> "escapar" eh independente do que aconteceu nas 5 primeiras na hora de
>> calcular o termo a ser subtraido. A ideia eh que este "escapar" eh soh a
>> capacidade do sapo pegar ou nao a mosca, nao incluindo a "escapada" quando o
>> sapo estah saciado.
>>
>> Abraco,
>>   Ralph
>> 2011/10/15 Rafael Forte 
>>
>>> Olá Pessoal,
>>>
>>> Eu raciocinei da seguinte forma: A sexta mosca vai viver se o sapo já
>>> comeu 3 moscas *ou* se ela conseguir escapar. Portanto isso seria a
>>> probabilidade de 3 sucessos em 5 tentativas (o sapo comendo 3 moscas das 5
>>> que passaram) mais a probabilidade de a sexta mosca escapar. Assim:
>>> Pr(sexta mosca viver) = B(3; 5, 0.5) + Pr(mosca escapar) = [C(5, 3) *
>>> 0.5^3 * 0.5^2] + 0.5 = 0.8125
>>>
>>> Ralph, eu tenho duas dúvidas quanto ao item (ii): (1) Porque a binomial é
>>> cumulativa e (2) porque voce usou X>=3 (pelo que entendi do problema, o sapo
>>> *nunca* come mais de 3 moscas)?
>>>
>>> Abracos,
>>> Rafael
>>>
>>> 2011/10/13 Ralph Teixeira 
>>>
>>>> Entendi o enunciado assim: um sapo, *quando faminto*, tem 50% de chance
>>>> de comer uma mosca que passe por perto dele. Ele só deixa de estar faminto
>>>> após comer 3 moscas -- depois da 3a, é 0% de chance de ele comer a mosca.
>>>> Digo isso porque, tecnicamente, não é verdade que "uma mosca tenha 50/50 de
>>>> chance de escapar", pois isto depende de quando a mosca chega.
>>>>
>>>> Bom, então imagino primeiro um sapo que está sempre faminto... Há duas
>>>> hipóteses excludentes para a 6a mosca escapar:
>>>> i) O sapo decide não comê-la: p1=50%
>>>> ii) O sapo decide comê-la, mas ele já comeu PELO MENOS 3 antes:
>>>> p2=(Pr(X>=3)).50%
>>>> onde X é o número de moscas que este sapo decidiu comer dentre as 5
>>&

Re: [obm-l] Questao de probabilidade: o sapo e a mosca

2011-10-15 Por tôpico Ralph Teixeira
Eu devia ter frisado melhor, que minha solucao imagina um sapo que estah
**SEMPRE FAMINTO** (que eh equivalente ao original, como expliquei na outra
mensagem). Por isso uso a cumulativa,  X>=3.

Na sua solucao, temos que consertar um par de coisas:
i) Se eu entendi o que "consegue escapar" significa, entao os eventos nao
sao excludentes. Entao tem que subtrair a probabilidade do sapo comer 3
moscas *E* a mosca escapar.
ii) Se seguirmos aa risca o enunciado original, onde o sapo para de comer
moscas quando chega em 3, entao a distribuicao nao eh exatamente binomial --
a distribuicao binomial supoe independencia de experimentos, o que deixa de
ser verdade quando o sapo estah saciado. Para consertar isso, o jeito eh
mudar o sapo para um que nao para de comer -- foi o que eu fiz na minha
solucao -- mas ai temos de considerar 3, 4 ou 5 moscas comidas em 5.

Entao ficaria:

Pr(sexta mosca viver)=S+Pr(mosca escapar)-S.Pr(mosca escapar)

onde S=B(3;5,0.5)+B(4;5,0.5)+B(5;5,0.5)=1/2. Note como usei que a 6a mosca
"escapar" eh independente do que aconteceu nas 5 primeiras na hora de
calcular o termo a ser subtraido. A ideia eh que este "escapar" eh soh a
capacidade do sapo pegar ou nao a mosca, nao incluindo a "escapada" quando o
sapo estah saciado.

Abraco,
  Ralph
2011/10/15 Rafael Forte 

> Olá Pessoal,
>
> Eu raciocinei da seguinte forma: A sexta mosca vai viver se o sapo já comeu
> 3 moscas *ou* se ela conseguir escapar. Portanto isso seria a probabilidade
> de 3 sucessos em 5 tentativas (o sapo comendo 3 moscas das 5 que passaram)
> mais a probabilidade de a sexta mosca escapar. Assim:
> Pr(sexta mosca viver) = B(3; 5, 0.5) + Pr(mosca escapar) = [C(5, 3) * 0.5^3
> * 0.5^2] + 0.5 = 0.8125
>
> Ralph, eu tenho duas dúvidas quanto ao item (ii): (1) Porque a binomial é
> cumulativa e (2) porque voce usou X>=3 (pelo que entendi do problema, o sapo
> *nunca* come mais de 3 moscas)?
>
> Abracos,
> Rafael
>
> 2011/10/13 Ralph Teixeira 
>
>> Entendi o enunciado assim: um sapo, *quando faminto*, tem 50% de chance de
>> comer uma mosca que passe por perto dele. Ele só deixa de estar faminto após
>> comer 3 moscas -- depois da 3a, é 0% de chance de ele comer a mosca. Digo
>> isso porque, tecnicamente, não é verdade que "uma mosca tenha 50/50 de
>> chance de escapar", pois isto depende de quando a mosca chega.
>>
>> Bom, então imagino primeiro um sapo que está sempre faminto... Há duas
>> hipóteses excludentes para a 6a mosca escapar:
>> i) O sapo decide não comê-la: p1=50%
>> ii) O sapo decide comê-la, mas ele já comeu PELO MENOS 3 antes:
>> p2=(Pr(X>=3)).50%
>> onde X é o número de moscas que este sapo decidiu comer dentre as 5
>> primeiras (que seria uma distribuição binomial cumulativa). Ou seja:
>> Pr(X>=3)=C(5,3).(1/2)^5+C(5,4).(1/2)^5+C(5,5).(1/2)^5=1/2
>> (Faz sentido pela simetria do problema: das 5 primeiras, a probabilidade
>> de ele comer 0,1 ou 2 moscas é idêntica à probabilidade de ele comer 3,4 ou
>> 5 moscas.)
>>
>> Então no fim das contas: Pr(6a mosca escapar)=1/2+1/4=3/4=75%
>>
>> Abraço,
>>  Ralph
>> 2011/10/13 Rafael Forte 
>>
>>> Uma mosca passando próxima a um sapo tem 50% de chances de escapar e 50%
>>> de virar alimento do sapo. Mas o sapo come apenas 3 moscas por dia. Qual é a
>>> probabilidade da sexta mosca a passar próxima ao sapo sobreviver?
>>>
>>> Abracos,
>>> Rafael
>>>
>>
>>
>


Re: [obm-l] Questao de probabilidade: o sapo e a mosca

2011-10-13 Por tôpico Ralph Teixeira
Oi, Luan.

Pois é, tem dois sapos aqui que parecem ser distintos:

i) O sapo do problema; após comer 3 moscas, ele nunca mais decide nada, pois
está satisfeito.
ii) O meu sapo. Ele sempre **decide** comer uma mosca com 50% de chance --
mas, se ele já comeu 3, ele desiste de comer a mosca, DEPOIS de ter tomado a
decisão.

Na prática, esses sapos são equivalentes, por isso que eu prefiro o sapo
(ii) -- na minha solução, sempre que eu escrevo "comeu", pense "decidiu
comer" (mesmo que ele desista em seguida). Então a 6a mosca escapa em duas
hipóteses:

a) Se meu sapo decide não comê-la
ou
b) Se meu sapo decide comê-la, mas já tinha decidido comer pelo menos 3
antes.

Abraço,
  Ralph
2011/10/13 Luan Gabriel 

>  Ralph, obrigado, a segunda parte eu consertei,mas esqueci do caso "0 das
> cinco primeiras". A sua foi muito mais elegante hehe.
>
> Mas só fiquei em dúvida quanto a uma coisa: quando o sapo decide comê-la,
> ele não tinha que ter comido menos de 3 moscas ?Se ele comeu pleo menos 3,
> ele sempre teria que decidir não comê-la,não? Mas foi muito melhor enxergar
> duas situações excludentes apenas =]
>
> --
> Date: Thu, 13 Oct 2011 10:34:19 -0300
> Subject: Re: [obm-l] Questao de probabilidade: o sapo e a mosca
> From: ralp...@gmail.com
> To: obm-l@mat.puc-rio.br
>
>
> Oi, Luan. Sua solução funciona, mas tem que consertar dois pontinhos:
>
> o) Faltou o caso "0 das 5 primeiras", o que adiciona mais 1/64 na sua
> resposta.
>
> iii) Por outro lado, o número de casos em (iii) é menor, pois você já
> fixou uma mosca comida em cada subcaso. Seria:
> iiia) Há apenas C(4,2)=6 casos, já que você já definiu que a 3a mosca
> comida é a 5a.
> iiib) Há apenas C(3,2)=3 casos (a 3a mosca comida tem de ser a 4a).
> iiic) Um caso apenas, como você disse.
> Então a soma do caso (iii) é 6/32+3/16+1/8=16/32
>
> Juntando tudo:
> 1/64+5/64+10/64+16/32=75%
>
> Abraço,
> Ralph
>
> 2011/10/13 Luan Gabriel 
>
>  Cara eu pensei assim:
> Divida o problema em 3 casos: o sapo come 1 mosca das cinco primeiras
> moscas; o sapo come duas das cinco primeiras; o sapo come três das cinco
> primeiras.
> i) Se o sapo comer 1 das cinco primeiras, então há cinco opções para a
> mosca comida, e a probabilidade de cada mosca escapar será 1/2 e a da
> comida, 1/2 . Então, a probabilidade
> da sexta mosca escapar será 1/2 vezes as probabilidades anteriores :
> 5.(1/2)^6
> ii) Se o sapo comer 2 das cinco primeiras, então há C(5,2)=10 opções para a
> escolha das moscas comidas, e analogamente ao caso anterior a resposta será
> 10.(1/2)^6
> iii) Se o sapo comer 3 das cinco primeiras. Nesse caso, há mais uma divisão
> dos casos:
>  a) Se a 3º mosca comida é a 5º:as probabilidades das moscas que
> escapam é 1/2 e a das comidas, 1/2. Há C(5,3)=10 opções para escolher as
> moscas comidas : 10.(1/2)^5. Depois disso, a probabilidade da sexta escapar
> será 1, pois o sapo não comerá mais de 3 moscas por dia => 1.10.(1/2)^5
>  b) Se a 3º mosca comida é a 4º, o processo será análogo :
> C(4,3).(1/2)^4. Depois disso a probabilidade das duas seguintes fugirem será
> 1 =>1.1 4.(1/2)^4
>  c) Se a 3º mosca comida é a 3º, analogamente obtemos (1/2)^3.
> Então, a probabilidade da sexta mosca sobreviver será
> 5/64 + 10/64 + 10/32+ 4/16 + 1/8 = 59/64=92,2%
> Espero que eu esteja coerente.
> --
> Date: Thu, 13 Oct 2011 07:43:14 -0400
> Subject: [obm-l] Questao de probabilidade: o sapo e a mosca
> From: rcforte.profissio...@gmail.com
> To: obm-l@mat.puc-rio.br
>
>
> Uma mosca passando próxima a um sapo tem 50% de chances de escapar e 50% de
> virar alimento do sapo. Mas o sapo come apenas 3 moscas por dia. Qual é a
> probabilidade da sexta mosca a passar próxima ao sapo sobreviver?
>
> Abracos,
> Rafael
>
>
>


Re: [obm-l] Questao de probabilidade: o sapo e a mosca

2011-10-13 Por tôpico Ralph Teixeira
Oi, Luan. Sua solução funciona, mas tem que consertar dois pontinhos:

o) Faltou o caso "0 das 5 primeiras", o que adiciona mais 1/64 na sua
resposta.

iii) Por outro lado, o número de casos em (iii) é menor, pois você já
fixou uma mosca comida em cada subcaso. Seria:
iiia) Há apenas C(4,2)=6 casos, já que você já definiu que a 3a mosca comida
é a 5a.
iiib) Há apenas C(3,2)=3 casos (a 3a mosca comida tem de ser a 4a).
iiic) Um caso apenas, como você disse.
Então a soma do caso (iii) é 6/32+3/16+1/8=16/32

Juntando tudo:
1/64+5/64+10/64+16/32=75%

Abraço,
Ralph

2011/10/13 Luan Gabriel 

>  Cara eu pensei assim:
> Divida o problema em 3 casos: o sapo come 1 mosca das cinco primeiras
> moscas; o sapo come duas das cinco primeiras; o sapo come três das cinco
> primeiras.
> i) Se o sapo comer 1 das cinco primeiras, então há cinco opções para a
> mosca comida, e a probabilidade de cada mosca escapar será 1/2 e a da
> comida, 1/2 . Então, a probabilidade
> da sexta mosca escapar será 1/2 vezes as probabilidades anteriores :
> 5.(1/2)^6
> ii) Se o sapo comer 2 das cinco primeiras, então há C(5,2)=10 opções para a
> escolha das moscas comidas, e analogamente ao caso anterior a resposta será
> 10.(1/2)^6
> iii) Se o sapo comer 3 das cinco primeiras. Nesse caso, há mais uma divisão
> dos casos:
>  a) Se a 3º mosca comida é a 5º:as probabilidades das moscas que
> escapam é 1/2 e a das comidas, 1/2. Há C(5,3)=10 opções para escolher as
> moscas comidas : 10.(1/2)^5. Depois disso, a probabilidade da sexta escapar
> será 1, pois o sapo não comerá mais de 3 moscas por dia => 1.10.(1/2)^5
>  b) Se a 3º mosca comida é a 4º, o processo será análogo :
> C(4,3).(1/2)^4. Depois disso a probabilidade das duas seguintes fugirem será
> 1 =>1.1 4.(1/2)^4
>  c) Se a 3º mosca comida é a 3º, analogamente obtemos (1/2)^3.
> Então, a probabilidade da sexta mosca sobreviver será
> 5/64 + 10/64 + 10/32+ 4/16 + 1/8 = 59/64=92,2%
> Espero que eu esteja coerente.
> --
> Date: Thu, 13 Oct 2011 07:43:14 -0400
> Subject: [obm-l] Questao de probabilidade: o sapo e a mosca
> From: rcforte.profissio...@gmail.com
> To: obm-l@mat.puc-rio.br
>
>
> Uma mosca passando próxima a um sapo tem 50% de chances de escapar e 50% de
> virar alimento do sapo. Mas o sapo come apenas 3 moscas por dia. Qual é a
> probabilidade da sexta mosca a passar próxima ao sapo sobreviver?
>
> Abracos,
> Rafael
>


Re: [obm-l] Questao de probabilidade: o sapo e a mosca

2011-10-13 Por tôpico Ralph Teixeira
Entendi o enunciado assim: um sapo, *quando faminto*, tem 50% de chance de
comer uma mosca que passe por perto dele. Ele só deixa de estar faminto após
comer 3 moscas -- depois da 3a, é 0% de chance de ele comer a mosca. Digo
isso porque, tecnicamente, não é verdade que "uma mosca tenha 50/50 de
chance de escapar", pois isto depende de quando a mosca chega.

Bom, então imagino primeiro um sapo que está sempre faminto... Há duas
hipóteses excludentes para a 6a mosca escapar:
i) O sapo decide não comê-la: p1=50%
ii) O sapo decide comê-la, mas ele já comeu PELO MENOS 3 antes:
p2=(Pr(X>=3)).50%
onde X é o número de moscas que este sapo decidiu comer dentre as 5
primeiras (que seria uma distribuição binomial cumulativa). Ou seja:
Pr(X>=3)=C(5,3).(1/2)^5+C(5,4).(1/2)^5+C(5,5).(1/2)^5=1/2
(Faz sentido pela simetria do problema: das 5 primeiras, a probabilidade de
ele comer 0,1 ou 2 moscas é idêntica à probabilidade de ele comer 3,4 ou 5
moscas.)

Então no fim das contas: Pr(6a mosca escapar)=1/2+1/4=3/4=75%

Abraço,
 Ralph
2011/10/13 Rafael Forte 

> Uma mosca passando próxima a um sapo tem 50% de chances de escapar e 50% de
> virar alimento do sapo. Mas o sapo come apenas 3 moscas por dia. Qual é a
> probabilidade da sexta mosca a passar próxima ao sapo sobreviver?
>
> Abracos,
> Rafael
>


Re: [obm-l] Vetores

2011-09-28 Por tôpico Ralph Teixeira
Eu gosto de fazer assim, usando o produto interno  de dois vetores u e
v:

u+v eh paralelo aa bissetriz sse
/|u||u+v| = /|v||u+v| (pois estes sao os cossenos dos angulos
de u+v com u e v respectivamente; note-se que estou usando que u+v sempre
estah no menor angulo ENTRE u e v) sse
(+)|v| = (+)|u| sse
(|u||v|-)(|u|-|v|)=0 sse
|u|=|v|

jah que, como u e v nao sao paralelos, temos |u||v| > . Note que tambem
usei que |u|,|v| e |u+v| sao nao nulos, o que vem do fato de u e v serem
lados de um paralelogramo supostamente nao degenerado.

Abraco,
  Ralph

2011/9/27 Kleber Bastos 

> Alguém poderia dar uma luz na seguinte questão:
>
> *No paralelogramo de lados u e v, prove que u + v é paralelo à bissetriz
> do ângulo formado por u e v se, e somente se, | u | = | v |*
>
> --
> Bastos
>


Re: [obm-l] Desafio 7 e 10

2011-09-28 Por tôpico Ralph Teixeira
Formalmente, o problema estah mal definido. Informalmente, este negocio
sempre foi divertido, acho legal brincar com coisas assim, como o problema
dos quatro quatros. Podemos votar na resposta mais bacana, usando simbolos
mais banais, etc. :)

Que tal:
77/7-7=4

Abraco,
Ralph

2011/9/28 Rhilbert Rivera 

>  Como tornar as igualdades verdadeiras usando símbolos e operações
> aritméticas elementares
>
>
> 7  7  7  7  = 4
>
> 10   10   10   10  = 4
>
>
> Obrigado
>
>
>


[obm-l] Re: [obm-l] Valor mínimo

2011-09-23 Por tôpico Ralph Teixeira
Pffft -- nao usei que a e b sao positivos, entao o que escrevi vale para
todos os casos. :)
2011/9/23 Ralph Teixeira 

> Vou supor que a e b sao positivos -- os outros casos sao analogos.
>
> Seja c=raiz(a^2+b^2). Seja y tal que cosy=a/c e siny=b/c -- note que y
> existe sim, jah que (a/c)^2+(b/c)^2=1.
>
> Entao a nossa expressao eh
>
> c(cosy.sinx+siny.cosx)=c.sin(x+y)
>
> cujo minimo eh -c (que ocorre quando x+y=2kpi+3pi/2).
>
> Abraco,
>   Ralph
> 2011/9/23 marcone augusto araújo borges 
>
>>  Como provar que o valor minimo de asenx + bcosx = - raiz(a^2 + b^2) ?
>>
>
>


[obm-l] Re: [obm-l] Valor mínimo

2011-09-23 Por tôpico Ralph Teixeira
Vou supor que a e b sao positivos -- os outros casos sao analogos.

Seja c=raiz(a^2+b^2). Seja y tal que cosy=a/c e siny=b/c -- note que y
existe sim, jah que (a/c)^2+(b/c)^2=1.

Entao a nossa expressao eh

c(cosy.sinx+siny.cosx)=c.sin(x+y)

cujo minimo eh -c (que ocorre quando x+y=2kpi+3pi/2).

Abraco,
  Ralph
2011/9/23 marcone augusto araújo borges 

>  Como provar que o valor minimo de asenx + bcosx = - raiz(a^2 + b^2) ?
>


Re: [obm-l] ajuda geometria

2011-09-05 Por tôpico Ralph Teixeira
Bom, vou reclamar que o enunciado esqueceu de dizer que E pertence a AC...
mas o João espertamente adivinhou que tinha que ser isto (senão não saía) e
matou o problema.

Abraço,
Ralph

2011/9/5 João Maldonado 

>
>
> ABC = CED,  logo AED =  180 - ABC e  o quadrilátero EABD é inscritível,
>  logo  EAD = ABD =  45°
>
> []'s
> João
>
> --
> Date: Mon, 5 Sep 2011 11:28:06 -0700
> From: cacar...@yahoo.com
> Subject: [obm-l] ajuda geometria
> To: obm-l@mat.puc-rio.br
>
>
> Como pensar...?
>
> " Num triangulo ABC retangulo em A, o cateto AC é maior que AB. Pelo ponto
> D, pé da bissetriz do angulo reto, trace DE, perpendicular a BC. Mostre que
> o angulo EBD mede 45 graus"
>
> Obrigado
>


Re: [obm-l] Enc: Outro Probleminha

2011-09-02 Por tôpico Ralph Teixeira
Sim, os 4 lados não determinam a diagonal. Mas vejo na sua outra mensagem
que você já matou o que faltava -- explicitar a diagonal certa como sinz
para determinar o quadrilátero.

Abraço,
 Ralph

2011/9/2 luiz silva 

> Ola Ralph,
>
> O que temos é o seguinte :
>
> o triangulo x,y e z é semelhante ao triangulo senx, seny e senz. Os
> triangulos senx, seny e senz e sen z, cosx e cosy tem ângulos opostos ao
> lado senz suplementares.
>
> É possível termos um quadrilátero com os lados senx, seny, cosx e cosy sem
> que a diagonal seja o lado comum, sen z ?
>
> Abs
> Felipe
>
> --- Em *sex, 2/9/11, Ralph Teixeira * escreveu:
>
>
> De: Ralph Teixeira 
> Assunto: Re: [obm-l] Enc: Outro Probleminha
> Para: obm-l@mat.puc-rio.br
> Data: Sexta-feira, 2 de Setembro de 2011, 1:40
>
> Eh, mas falta algo -- nao ha garantia nenhuma que o triangulo formado por
> dois lados do quadrilatero e pela diagonal seja este que eh semelhante ao
> triangulo original.
>
> Alias, note que o enunciado original soh menciona os lados do quadrilatero,
> e, em geral, os lados NAO determinam o quadrilatero. Entao, com o que foi
> dado, o problema eh indeterminado.
>
> Para dar um exemplo mais concreto: tome X=Y=45 graus. Entao os lados sao
> todos iguais, ou seja, ABCD eh um losango. Mas nem todo losango eh
> inscritivel!
>
> Para determinar o problema, voce vai ter que dar mais alguma informacao,
> como uma diagonal, ou um angulo do quadrilatero.
>
> Abraco,
> Ralph
>
> 2011/9/1 luiz silva 
> http://mc/compose?to=luizfelipec...@yahoo.com.br>
> >
>
> Ola João,
>
> Só consegui visualizar sua resposta no site da obm (nao sei pq, nao chegou
> para mim).
>
> A questão vem da constatação de que se x,y e z são os lados de um
> triangulo, então senx, seny e senz são os lados de um triangulo semelhante
> (lei dos senos). Assim, temos que :
>
> senz^2 = senx^2 + seny^2 - 2 senxsenycosz
>
> Além disso, se resolvermos o sistema foemdo por z = xcosy+ycosx; x = zcosy
> + ycosz e y = xcosz + zcosx chegaremos a conclusão que senz^2 = cosx^2 +
> cosy^2 + 2cosxcosycosz (ou seja, um triangulo com lados senz, cosx e cosy e
> ângulo 180-z).
>
> O interessante é que podemos construir o quadrilátero de duas formas : uma
> onde a outra diagonal será 1 e outra onde ela sera sen(x-y).
>
> Abs
> Felipe
> Abs
> Felipe
>
> --- Em *qui, 1/9/11, luiz silva 
> http://mc/compose?to=luizfelipec...@yahoo.com.br>
> >* escreveu:
>
>
> De: luiz silva 
> http://mc/compose?to=luizfelipec...@yahoo.com.br>
> >
> Assunto: Outro Probleminha
> Para: "Matematica Lista" 
> http://mc/compose?to=obm-l@mat.puc-rio.br>
> >
> Data: Quinta-feira, 1 de Setembro de 2011, 13:00
>
>Pessoal,
>
> Consitnuando com minhas "investigações" em geometria, achei mais um
> probleminha legal :
>
> Demonstre que dado um triângulo (vamos assumir acutângulo, mas acho que
> serve para qqer um) com angulos X, Y e Z, que o quadrilátero ABCD, onde
> AB=SenY; BC = CosY; CD = CosX e DA = Sen X é inscritível.
>
> Abs
> Felipe
>
>
>


Re: [obm-l] Enc: Outro Probleminha

2011-09-01 Por tôpico Ralph Teixeira
Eh, mas falta algo -- nao ha garantia nenhuma que o triangulo formado por
dois lados do quadrilatero e pela diagonal seja este que eh semelhante ao
triangulo original.

Alias, note que o enunciado original soh menciona os lados do quadrilatero,
e, em geral, os lados NAO determinam o quadrilatero. Entao, com o que foi
dado, o problema eh indeterminado.

Para dar um exemplo mais concreto: tome X=Y=45 graus. Entao os lados sao
todos iguais, ou seja, ABCD eh um losango. Mas nem todo losango eh
inscritivel!

Para determinar o problema, voce vai ter que dar mais alguma informacao,
como uma diagonal, ou um angulo do quadrilatero.

Abraco,
Ralph

2011/9/1 luiz silva 

> Ola João,
>
> Só consegui visualizar sua resposta no site da obm (nao sei pq, nao chegou
> para mim).
>
> A questão vem da constatação de que se x,y e z são os lados de um
> triangulo, então senx, seny e senz são os lados de um triangulo semelhante
> (lei dos senos). Assim, temos que :
>
> senz^2 = senx^2 + seny^2 - 2 senxsenycosz
>
> Além disso, se resolvermos o sistema foemdo por z = xcosy+ycosx; x = zcosy
> + ycosz e y = xcosz + zcosx chegaremos a conclusão que senz^2 = cosx^2 +
> cosy^2 + 2cosxcosycosz (ou seja, um triangulo com lados senz, cosx e cosy e
> ângulo 180-z).
>
> O interessante é que podemos construir o quadrilátero de duas formas : uma
> onde a outra diagonal será 1 e outra onde ela sera sen(x-y).
>
> Abs
> Felipe
> Abs
> Felipe
>
> --- Em *qui, 1/9/11, luiz silva * escreveu:
>
>
> De: luiz silva 
> Assunto: Outro Probleminha
> Para: "Matematica Lista" 
> Data: Quinta-feira, 1 de Setembro de 2011, 13:00
>
>Pessoal,
>
> Consitnuando com minhas "investigações" em geometria, achei mais um
> probleminha legal :
>
> Demonstre que dado um triângulo (vamos assumir acutângulo, mas acho que
> serve para qqer um) com angulos X, Y e Z, que o quadrilátero ABCD, onde
> AB=SenY; BC = CosY; CD = CosX e DA = Sen X é inscritível.
>
> Abs
> Felipe
>
>


Re: [obm-l] Probabilidade

2011-08-31 Por tôpico Ralph Teixeira
Eh isso mesmo. Talvez o papo a seguinte ajude com o 2o caso.

Primeiro note que, se B nao sabe onde estah o carro, ele nao pode GARANTIR
que nunca abrirah a porta do carro -- uma das regras do problema
classico foi quebrada. Mas vamos lah (nao estou AFIRMANDO que o seguinte
acontece exatamente assim, mas afirmo que as PROPORCOES estao corretas e
correspondem aas probabilidades certas), supondo que as outras regras sao
mantidas, e usando N=100:

De cada 100 shows:
-- Em UM show, A acerta no inicio (que sorte!?); entao B, evitando a porta
de A, acaba abrindo 98 burros. Isto eh UM show daqueles 100.
-- Nos outros 99, A erra desde o inicio. Entao B vai escolher 98 portas
aleatoriamente para abrir... Como B nao sabe onde o carro estah, ele tem
98/99 de probabilidade de abrir a porta do carro. Isto eh:
 --- Em 98 destes, B acaba abrindo a porta do carro. Que pena, o show
acaba e fica sem graca.
 --- Em UM destes, B abre 98 portas com burros, e, por muita sorte,
acaba por deixar o carro na ultima porta.

Em suma, nestas condicoes, se B abre 98 portas com burros, algo excepcional
(2% de chance) acaba de acontecer -- ou eh o primeiro show, ou eh o ultimo!
O problema eh que tudo que a gente sabe eh que o show eh um daqueles 2 -- em
1 A acertou no comeco, no outro A errou. Entao, DADO que B abriu 98 burros,
fica 50% de chance para cada porta restante.

Repito: se B nao sabe onde estah o carro e abre 98 portas com burros, a
primeira sensacao eh "puxa! que coisa incomum acaba de acontecer! em quase
todos os shows que eu assisto, B abre a porta do carro!"; em seguida, "50%
para cada uma das portas restantes".

Abraco,
   Ralph
2011/8/31 Francisco Barreto 

> Se o sujeito B agir conforme o caso clássico, para n portas, as chances do
> sujeito A aumentam de 1/n para 1 - 1/n (as chances de A errar na primeira
> escolha de porta). Nas vezes em que A errar na primeira, trocar de porta
> levará A ao carro. Se acertar de primeira, bem, é claro que trocar não
> ganha.
> Agora, se B não souber onde está o carro e começar a abrir portas que não
> sejam a escolhida por A até restarem 2 portas fechadas, para A tanto faz
> trocar, pela mesma razão: cometer um erro e trocar de porta não lhe
> garantirá um carro. Esta última parte ainda não é muito clara pra mim.
>
> Em 31 de agosto de 2011 21:04, Ralph Teixeira escreveu:
>
> Oi, Douglas. Vamos lah.
>> 2011/8/31 
>>
>>> **
>>>
>>> Olá, gostaria de saber como se faz a seguinte questão:
>>>
>>> 1) Existem 100 portas numeradas de 1 a 100, atrás de 99 delas existe um
>>> burro, e em uma delas existe um carro, um rapaz começa a abrir as portas,
>>> sabendo que ele abriu 98 delas e em todas havia um burro, qual a
>>> probabilidade de que na outra ele encontre o carro?
>>>
>>
>> Olha, do jeito que voce enunciou o problema, nao faz sentido. Como assim
>> "na outra" -- ainda ha 2 portas fechadas, neh? Do jeito que voce enunciou, a
>> resposta eh 50% para cada uma das duas portas restantes (pressupondo que o
>> carro tinha a mesma chance de estar em qualquer porta no comeco).
>>
>> Agora, talvez voce queira o problema classico de Monty Hall, cujo
>> enunciado preciso eh assim:
>> i) 100 portas numeradas de 1 a 100, o carro tem inicialmente a mesma
>> chance de estar em qualquer uma delas;
>> ii) Um "participante" A (que nao sabe onde estah o carro) escolhe uma
>> porta, mas nao a abre;
>> iii) Um "rapaz" B abre 98 portas seguindo as regras:
>>  -- B NUNCA abre a porta que A escolhera;
>>  -- B NUNCA abre a porta do carro.
>>  -- Se B ainda assim tiver alguma escolha (o que eh raro -- soh acontece
>> se A escolheu o carro de primeira!), ele escolhe 98 portas com burros,
>> aleatoriamente, para abrir.
>> (Note que, para estas regras serem seguidas, B tem que SABER onde estah o
>> carro!)
>> iv) Neste momento, qual a chance da porta que A escolheu ter o carro? Qual
>> a chance da outra porta fechada ter o carro?
>>
>> Com este enunciado, as respostas sao 1/100 e 99/100 respectivamente. Era
>> este o problema que voce tinha em mente?
>>
>> ---///---
>>
>>
>>>  2) Num concurso de música, eistem 3 jurados, e um publico geral, e
>>> esses jurados aprovam ou não um candidato conforme a opinião do público e a
>>> tabela abaixo
>>>
>>>
>>>
>>> público geraljurado 1 /   jurado 2/  jurado 3
>>>
>>> aprova   50% 75%   80%
>>>
>>> não aprova 50% 40%25%
>>>
>>>
>>>
>>> qual a diferença entre as probabilidades de um candidato ser aprovado
>>&

Re: [obm-l] Probabilidade

2011-08-31 Por tôpico Ralph Teixeira
Oi, Douglas. Vamos lah.
2011/8/31 

> **
>
> Olá, gostaria de saber como se faz a seguinte questão:
>
> 1) Existem 100 portas numeradas de 1 a 100, atrás de 99 delas existe um
> burro, e em uma delas existe um carro, um rapaz começa a abrir as portas,
> sabendo que ele abriu 98 delas e em todas havia um burro, qual a
> probabilidade de que na outra ele encontre o carro?
>

Olha, do jeito que voce enunciou o problema, nao faz sentido. Como assim "na
outra" -- ainda ha 2 portas fechadas, neh? Do jeito que voce enunciou, a
resposta eh 50% para cada uma das duas portas restantes (pressupondo que o
carro tinha a mesma chance de estar em qualquer porta no comeco).

Agora, talvez voce queira o problema classico de Monty Hall, cujo
enunciado preciso eh assim:
i) 100 portas numeradas de 1 a 100, o carro tem inicialmente a mesma chance
de estar em qualquer uma delas;
ii) Um "participante" A (que nao sabe onde estah o carro) escolhe uma porta,
mas nao a abre;
iii) Um "rapaz" B abre 98 portas seguindo as regras:
 -- B NUNCA abre a porta que A escolhera;
 -- B NUNCA abre a porta do carro.
 -- Se B ainda assim tiver alguma escolha (o que eh raro -- soh acontece se
A escolheu o carro de primeira!), ele escolhe 98 portas com burros,
aleatoriamente, para abrir.
(Note que, para estas regras serem seguidas, B tem que SABER onde estah o
carro!)
iv) Neste momento, qual a chance da porta que A escolheu ter o carro? Qual a
chance da outra porta fechada ter o carro?

Com este enunciado, as respostas sao 1/100 e 99/100 respectivamente. Era
este o problema que voce tinha em mente?

---///---


> 2) Num concurso de música, eistem 3 jurados, e um publico geral, e esses
> jurados aprovam ou não um candidato conforme a opinião do público e a tabela
> abaixo
>
>
>
> público geraljurado 1 /   jurado 2/  jurado 3
>
> aprova   50% 75%   80%
>
> não aprova 50% 40%25%
>
>
>
> qual a diferença entre as probabilidades de um candidato ser aprovado caso
> o público geral o aprove e caso o público geral não o aprove??
>
>
>
Esta tabela nao esta muito clara para mim... Vou supor que estes
numeros significam o seguinte:

-- Dado um candidato aprovado pelo publico; ele tem 50% de chance de ser
aprovado pelo jurado 1, 75% de chance de ser aprovado pelo jurado 2, e 80%
de chance de ser aprovado pelo jurado 3.
-- Dado um candidato nao aprovado pelo publico, ele tem 50% de chance de ser
aprovado pelo jurado 1, 40% de chance de ser aprovado pelo jurado 2, e 25%
de chance de ser aprovado pelo jurado 3.

A outra coisa que nao estah clara: o que eh necessario para um candidato ser
"aprovado"? Unanimidade, ou maioria?

Entao, vou fazer as seguintes hipoteses adicionais:
i) O candidato eh aprovado se pelo menos 2 dos jurados o aprovam, e eh soh.
O voto do publico nao conta diretamente (mesmo que, indiretamente, o publico
afete a decisao dos jurados)
ii) Outra hipotese que se faz necessaria: vou supor que, apos o publico ter
votado, as decisoes dos jurados sao independentes entre si. Note que esta
hipotese eh sutil, e usualmente nao verdadeira! Usualmente, se o candidato
eh bom, ele tem uma maior chance de ser aprovado; entao, o fato de que o
jurado 2 aprovou eh uma indicacao de que o candidato eh bom, o que afeta a
probabilidade do jurado 3 aprova-lo! Entao a gente precisaria de varias
outras probabilidades condicionais para terminar o problema... A hipotese de
independencia eh como se os jurados NAO olhassem para o candidato, nem uns
para os outros; por assim dizer, eles veem a reacao do publico, e jogam uma
moeda (enviesada) para decidir se aprovam ou nao o candidato.

Bom, entao, vou usar J1 para indicar o evento "Jurado 1 APROVA" (idem para
J2 e J3):

-- Se o publico aprova, a probabilidade de pelo menos 2 jurados aprovarem o
candidato eh:
p(J1 e J2) + p(J2 e J3) + p(J1 e J3) - 2 p(J1 e J2 e J3) =
(0.5)(0.75)+(0.75)(0.80)+(0.5)(0.8)-2(0.5)(0.75)(0.8)=77.5%
(O "-2" eh necessario pois, nos 3 primeiros termos, contamos 3 vezes o
candidato que eh aprovado por todos os 3 jurados - descontamos 2 para
conta-lo uma vez)
(Note como eu usei fortemente a independencia de J1, J2 e J3 ao trocar "e"
por produtos de probabilidades)

-- Se o publico nao aprova...
Idem  = (0.5)(0.4)+(0.4)(0.25)+(0.5)(0.25)-2(0.5)(0.4)(0.25)=32.5%

Abraco,
 Ralph


[obm-l] Re: [obm-l] demonstração

2011-08-17 Por tôpico Ralph Teixeira
Oi, Marcus.

Eh, o jeito que voce escreveu estah estranho. Voce partiu do que queria
provar, e chegou na hipotese -- isto nao demonstra o teorema.

Isto dito, eh facil consertar a sua ideia:

i) PRIMEIRA OPCAO: basta escrever o que voce fez, mas na ordem correta:

"Suponha a>b>0.
Como a e b sao positivos, podemos dividir ambos os lados de a>b por ab.
Entao a/(ab)>b/(ab), isto eh, 1/b>1/a."

ii) SEGUNDA OPCAO: se voce escrever as coisas na ordem que voce escreveu,
mas usando claramente EQUIVALENCIAS, sua demonstracao eh valida. Em suma,
usando o simbolo <=> para a seta dupla do "se e somente se":

"Suponha a, b positivos. Entao:
1/b>1/a <=>
<=> (ab)(1/b)>(ab)(1/a) <=>
<=> a>b

Como a>b eh verdadeiro e usamos EQUIVALENCIAS (isto eh, implicacoes
REVERSIVEIS), estah provado que 1/a>1/b."

Abraco,
  Ralph

2011/8/17 Marcus Aurelio Gonçalves Rodrigues 

> Galera acho que estou fazendo alguma coisa errada nessa demonstração alguém
> pode da uma olhada para mim.
>
> Proposi ção: Se a > b > 0 então 1/b > 1/a
>
> Demonstra ção:
>  1/b > 1/a
>
> (ab) . 1/b > (ab) .1/a
>
> a> b e b > 0 porque como a e b são positivos todos os números envolvidos
> são positivos. Então conclu mos que a > b > 0 que~ e verdade por hip otese,
> logo tamb em e verdade que 1/b > 1/a
>
>
>
> --
> Prof Marcus
>


[obm-l] Re: [obm-l] RE: [obm-l] Re: [obm-l] Fórmula alternativa(equação do segundo grau)

2011-08-07 Por tôpico Ralph Teixeira
Eu pensei numa relacao bacana, usando a formula quadratica usual, nao sei se
voce vai gostar.

Vamos resolver ax^2+bx+c=0. Vamos supor que x=0 nao eh uma raiz (ou seja,
suponha c<>0); entao, dividindo por x^2, vem

a+b/x+c/x^2=0

Isto quer dizer que 1/x eh raiz da quadratica P(z)=cz^2+bz+a. Se voce
aplicar a formula quadratica (que nao eh Baskara em lugar nenhum do mundo),
vem z=[-b+-sqrt(b^2-4ac)]/2c. Como x=1/z... acabou.

(No fundo no fundo, foi isso que o Joao fez, mas ele desenvolveu as contas
mais completamente)

Diga-se de passagem, a formula alternativa NAO funciona bem quando c=0 -- a
formula vira uma coisa do tipo 0/0, e a gente fica sem achar as raizes.

Abraco,
   Ralph


2011/8/7 marcone augusto araújo borges 

>  Certo,Bruno.Mas eu queria ver uma maneira de construir a expressão
> desse´´x´´.Obrigado.
>
>  --
> From: bfr...@gmail.com
> Date: Sun, 7 Aug 2011 01:45:00 -0300
> Subject: [obm-l] Re: [obm-l] Fórmula alternativa(equação do segundo grau)
> To: obm-l@mat.puc-rio.br
>
> Basta você substituir esse "x" na equação original e verificar que vc chega
> numa expressão válida.
>
> --
> Bruno FRANÇA DOS REIS
>
> msn: brunoreis...@hotmail.com
> skype: brunoreis666
> tel: +55 11 9961-7732
>
> http://brunoreis.com
> http://brunoreis.com/tech (en)
> http://brunoreis.com/blog (pt)
>
> GPG Key: http://brunoreis.com/bruno-public.key
>
> e^(pi*i)+1=0
>
>
> 2011/8/7 marcone augusto araújo borges 
>
>  Eu vi em um site, sugerido aqui nessa lista há um bom tempo, a fórmula
>
> x = 2c/(-b + - raiz(b^2 - 4ac)),para achar as raízes de uma equação do
> segundo grau
> A demonstração dessa fórmula pode ser feita usando a fórmula mais
> conhecida,racionalizando o seu numerador
> Desculpem a simplicidade da questão,mas eu gostaria de saber se há outra
> maneira de demonstrar essa fórmula alternativa
> Agradeço desde já
> Abraços,Marcone.
>
>
>


Re: [obm-l] Complexo

2011-08-04 Por tôpico Ralph Teixeira
Correto, mas para ser chato, eu diria que i^i eh indeterminado e pode
assumir varios valores distintos, dependendo da sua definicao de
exponenciacao nos complexos.

Ilustrando o que eu disse: i=e^(i pi/2 + 2k.i.pi) onde k eh um inteiro
qualquer. Agora, fazendo o que o Victor fez, vem:

i^i = (e^(i pi/2 + 2.k.pi.i))^i = e^(-pi/2).e^(-2.k.pi)

o que assume varios valores **reais distintos** dependendo do k que voce
escolhe (k=0 dah a resposta mais usual).

(Para o pessoal pensar: existe alguma situacao em que a conta z^y NAO da
este tipo de problema? Em outras palavras, quais sao as condicoes em z e y
complexos para que z^y tenha claramente um valor unico?)

Abraco,
  Ralph

On Wed, Aug 3, 2011 at 10:55 PM, Victor Seixas Souza wrote:

> Escrevendo i na forma polar, temos:
> i = e ^ (i pi/2)
> Para calcular i ^ i, fazemos:
> i ^ i = e ^ ln ( i^i ) = e ^ i ln i
> Utilizando a forma polar, verificamos que
> ln i = ln e ^(i pi/2) = i pi/2
> Portanto,
> i ^ i = e ^ ( i (i pi/2) ) = e ^ (-pi/2)
>


[obm-l] Re: [obm-l] Essa ainda não consegui!!!

2011-07-24 Por tôpico Ralph Teixeira
Ah... aposto que o problema original era para mostrar que um dos PONTOS
MEDIOS desses 10 segmentos tem coordenadas inteiras, nao? Ai tudo faz
sentido: basta olhar a paridade de ambas as coordenadas. Ha 4 "classes" de
possibilidades: (Par,Par), (Par, Impar), (Impar, Par), (Impar, Impar). Como
voce tem 5 pontos, pombas, tem que haver dois deles dentro da mesma
"classe", digamos, X e Y. Mas entao as coordenadas de X+Y serao ambas pares,
isto eh, as coordenadas do ponto medio (X+Y)/2 serao inteiras.

Aposto 10 pratas que era esse o problema! Em dolar! :)

Abraco,
Ralph

2011/7/24 Pedro Júnior 

> Sejam A, B, C, D e E pontos do plano cartesiano de coordenadas inteiras.
> Três quaisquer desses pontos não estão alinhados, logo formam dez segmentos.
> Mostre que pelo menos um dos pontos de intersecção desses segmentos é um
> ponto, também, de coordenadas inteiras.
> Desde já agradeço.
>
> --
>
> Pedro Jerônimo S. de O. Júnior
>
> Professor de Matemática
>
> Geo João Pessoa – PB
>
>


[obm-l] Re: [obm-l] Essa ainda não consegui!!!

2011-07-24 Por tôpico Ralph Teixeira
Estranho... eh isso mesmo?

Estritamente falando, A seria a intersecao de AB com AC, e A tem coordenadas
inteiras. Mas imagino que o problema queira uma interseccao de coordenadas
inteiras que NAO seja um dos pontos originais.

Entao resolvi me divertir com o Geogebra, botei 5 pontos no plano, desenhei
as tais 10 retas (nem peguei *segmentos*, peguei as retas logo), e fui catar
todas as intersecoes...

Quantas intersecoes, sem contar os pontos ABCDE originais? Vejamos...
escolha uma das 10 retas (digamos, XY); depois, voce tem que escolher uma
reta que NAO passe nem por X nem por Y, ou seja, escolha a reta ZW onde Z e
W sao um dos 3 pontos restantes (3 possibilidades). Entao sao 10x3=30 pontos
de intersecao, mas, oops, contei cada um deles duas vezes (XY com ZW = ZW
com XY), entao sao de fato 15 pontos de intersecao. Bom, no maximo 15, jah
que algumas das retas poderiam ser paralelas... Mas, tah, no maximo 15.

Botei A(0,0), B(10,-3), C(9,1), D(7,5) e E(2,8); estou aqui olhando as 15
intersecoes, e nenhuma delas parece ter coordenadas inteiras... Mas estou
num Netbookzinho com tela tao pequena que mal dah para enxergar a figura
toda, e estou com muita preguica de fazer no braco, entao eh perfeitamente
possivel que eu tenha comido mosca, marcado o ponto errado, etc. Alguem
confirma?

Abraco,
  Ralph

2011/7/24 Pedro Júnior 

> Sejam A, B, C, D e E pontos do plano cartesiano de coordenadas inteiras.
> Três quaisquer desses pontos não estão alinhados, logo formam dez segmentos.
> Mostre que pelo menos um dos pontos de intersecção desses segmentos é um
> ponto, também, de coordenadas inteiras.
> Desde já agradeço.
>
> --
>
> Pedro Jerônimo S. de O. Júnior
>
> Professor de Matemática
>
> Geo João Pessoa – PB
>
>


[obm-l] Re: [obm-l] FW: Combinatória

2011-07-20 Por tôpico Ralph Teixeira
Eu gosto MUITO da solucao do Fernando, tambem acho que eh a mais elegante.

Isto dito, vou ser chato muito muito chato: nada no problema sugere nem pede
probabilidades. Entao vamos usar a otima ideia, mas mudar a linguagem:

"Para cada possibilidade que tenha a ordem ABC, teremos possibilidades
correspondentes com as ordens ACB, CAB, etc. Ou seja, o numero de
classificacoes possiveis eh 6 vezes o numero de classificacoes pedido.
Assim, existem 6!/6=120 classificacoes possiveis onde Felipe se torna
campeao."

E notem que isto NAO significa que a probabilidade de Felipe ser
campeao eh 1/6, jah que ninguem sabe se as "possibilidades" sao
realmente igualmente provaveis.

(Alias, considerando o momento da Red Bull e do Felipe... :) :) :) :) )

Abraco,
  Ralph

2011/7/20 João Maldonado 

>  Olá.
>
> Eu não vejo erro, aliás é até mais prática do que as soluções já
> apresentradas.
>
> Só esclarecendo  para os outros, demorei um pouco para entender:
>
>
> ABC  significa que A está a frente de B que está a frente de C  (não
> importa de que jeito)
> Ex:
> ADBECF ou ABDEFC, DEFABC, etc
>
> --
> Date: Tue, 19 Jul 2011 11:30:17 -0300
> Subject: Combinatória
> From: facande...@gmail.com
> To: joao_maldona...@hotmail.com
>
> Prezador colega.
>
> Outra maneira seria a seguinte, usando sua convenção para o nome dos
> concorrentes.
> Os possíveis resultados seriam uma das seis permutações,   ABC, ACB, BAC,
> BCA, CAB, CBA. todas equiprováveis. Como a ordem  favorável é  ABC,
> e probabilidade de sua ocorrência será 1/6, que será também a de Felipe ser
> campeão,  como você achou
> Está certa essa linha de raciocínio?
> Sds.
> Fernando Candeias
>
>
>
>


[obm-l] Re: [obm-l] RE: [obm-l] Re: [obm-l] RE: [obm-l] Re: [obm-l] Integral difícil

2011-07-10 Por tôpico Ralph Teixeira
G foi cuidadosamente escolhida para que isto valha. Afinal, note que:

d(FG)/dw=F`G+FG`

e note que G=e^(Int b), entao pela Regra da Cadeia G`=e^(Int b)(d(Int
b)/dw)=b.e^(Int b)=bG

Para achar quaisquer constantes de integracao, substitua um valor conhecido
(t=0 e v(0)=v_0, como voce sugeriu) e calibre a constante.

(No exemplo em questao, K=F_0G_0=(v_0)^2, como voce disse, *desde que voce
tome G(0)=1*)

Abraco,
   Ralph
2011/7/11 João Maldonado 

>  Valeu Ralph, Mas ainda não entendi porque
>
> dF G(w) + b(w) F G(w) = d(FG)/dw
>
> Aliás, consegui resolver a integral desse modo :)
> Como acho o valor de K? seria o Vo ²?
>
> []'s
> João
>
> --
> Date: Sun, 10 Jul 2011 22:52:23 -0300
> Subject: [obm-l] Re: [obm-l] RE: [obm-l] Re: [obm-l] Integral difícil
> From: ralp...@gmail.com
> To: obm-l@mat.puc-rio.br
>
> Oi, Joao.
>
> Certamente, ha um monte de teoria sobre Equacoes Diferenciais Ordinarias,
> um monte mesmo; ha varios tipos de EDOs que se resolvem por varios metodos,
> e um monte de EDOs que nao tem solucao ou que nao se resolve por integrais
> simples.
>
> Essa ai bom, eu nao acompanhei a discussao, mas u, R e g sao
> constantes? Se forem, voce pode:
>
> i) Trocar de funcao; ao inves de v, trabalhe com F(w)=(v(w))^2; entao
> v.dv/dw eh (1/2)(dF/dw), e a EDO em F eh mais simples que a EDO em v.
> ii) De fato, voce fica com uma EDO linear de primeira ordem em F, que pode
> ser escrita na forma F`+b(w).F=c(w). Estas EDOs podem ser resolvidas por um
> metodo chamado FATOR INTEGRANTE, que eh:
> -- Multiplique os dois lados por G(w)=e^(Integral de b(w)).
> -- Agora o lado esquerdo eh d(F.G)/dw, isto eh, ficamos com
> d(FG)/dw=G(w)c(w).
> -- Integrando dw, fica FG=Int(Gc)+K, ou seja, F=(Int(Gc)+K)/G
>
> Abraco,
>   Ralph
> 2011/7/10 João Maldonado 
>
>  Valeu Eduardo.
>
> Há algum geito de resolvermos a equação diferencial de um modo geral?
> Como tinha dito sou aluno do terceiro colegial não tenho nenhuma idéia de
> como resolver .
>
> Aliás, teria se não existesse o uv²/R, daí passaria o dw para o outro lado
> e integraria  os 2 lados, o  primeiro em função de v e o segundo em função
> de w
> Mas com o uv²/R, ao integrar, ficaria v²/2R =  -g(-cosw  + usenw )-
>  (u/R)  I(v².dw)
> I(v².dw) =  g(cosw -usenw)R/u - v²
>
> Como resolvo isso?  Tentei chuta as funções v até que uma desse certo, mas
>  não consegui
>
> --
> Date: Sun, 10 Jul 2011 09:42:09 -0700
> From: eduardowil...@yahoo.com.br
> Subject: [obm-l] Re: [obm-l] Integral difícil
> To: obm-l@mat.puc-rio.br
>
> O problema de sinal é delicado. Devemos tomar cuidado com a convenção do
> ângulo
>
> alpha, que agora tomo medido da base no sentido anti-horário:
>
> -  (1/R).v.(dv)/(dw) = g.sen w + u.g.cos w + u.v^2/R ,
>
> que se parece mais com a (sua?) versão do jaumzaum indicada no link
> doSammyS.
>
> Digo "parece" pois há a diferença, p.ex., do sinal negativo no primeiro
> membro.
>
> Curioso que para u = 0,5 pode-se resolver facilmente a eq. dif. mas dá um
> problemão para alfa = 0 ...
>
>
>
> --- Em *sex, 8/7/11, João Maldonado *escreveu:
>
>
> De: João Maldonado 
> Assunto: [obm-l] Integral difícil
> Para: obm-l@mat.puc-rio.br
> Data: Sexta-feira, 8 de Julho de 2011, 21:55
>
>  Boa Tarde a todos
>
> Recentemente postei uma integral que não consegui resolver no fórum
>  PHYSICSFORUMS mas não obtive nenhuma resposta satisfatória.
> O problema a seguir é uma preparação para a IPhO, embora só a parte
> matemática interesse
>
> http://www.physicsforums.com/showthread.php?t=512186
>
> Reduzi o problema a equação encontrada no link acima, queria achar a função
> de velocidade em função da distância, S.
>
>
>


[obm-l] Re: [obm-l] RE: [obm-l] Re: [obm-l] Integral difícil

2011-07-10 Por tôpico Ralph Teixeira
Oi, Joao.

Certamente, ha um monte de teoria sobre Equacoes Diferenciais Ordinarias, um
monte mesmo; ha varios tipos de EDOs que se resolvem por varios metodos, e
um monte de EDOs que nao tem solucao ou que nao se resolve por integrais
simples.

Essa ai bom, eu nao acompanhei a discussao, mas u, R e g sao constantes?
Se forem, voce pode:

i) Trocar de funcao; ao inves de v, trabalhe com F(w)=(v(w))^2; entao
v.dv/dw eh (1/2)(dF/dw), e a EDO em F eh mais simples que a EDO em v.
ii) De fato, voce fica com uma EDO linear de primeira ordem em F, que pode
ser escrita na forma F`+b(w).F=c(w). Estas EDOs podem ser resolvidas por um
metodo chamado FATOR INTEGRANTE, que eh:
-- Multiplique os dois lados por G(w)=e^(Integral de b(w)).
-- Agora o lado esquerdo eh d(F.G)/dw, isto eh, ficamos com
d(FG)/dw=G(w)c(w).
-- Integrando dw, fica FG=Int(Gc)+K, ou seja, F=(Int(Gc)+K)/G

Abraco,
  Ralph
2011/7/10 João Maldonado 

>  Valeu Eduardo.
>
> Há algum geito de resolvermos a equação diferencial de um modo geral?
> Como tinha dito sou aluno do terceiro colegial não tenho nenhuma idéia de
> como resolver .
>
> Aliás, teria se não existesse o uv²/R, daí passaria o dw para o outro lado
> e integraria  os 2 lados, o  primeiro em função de v e o segundo em função
> de w
> Mas com o uv²/R, ao integrar, ficaria v²/2R =  -g(-cosw  + usenw )-
>  (u/R)  I(v².dw)
> I(v².dw) =  g(cosw -usenw)R/u - v²
>
> Como resolvo isso?  Tentei chuta as funções v até que uma desse certo, mas
>  não consegui
>
> --
> Date: Sun, 10 Jul 2011 09:42:09 -0700
> From: eduardowil...@yahoo.com.br
> Subject: [obm-l] Re: [obm-l] Integral difícil
> To: obm-l@mat.puc-rio.br
>
> O problema de sinal é delicado. Devemos tomar cuidado com a convenção do
> ângulo
>
> alpha, que agora tomo medido da base no sentido anti-horário:
>
> -  (1/R).v.(dv)/(dw) = g.sen w + u.g.cos w + u.v^2/R ,
>
> que se parece mais com a (sua?) versão do jaumzaum indicada no link
> doSammyS.
>
> Digo "parece" pois há a diferença, p.ex., do sinal negativo no primeiro
> membro.
>
> Curioso que para u = 0,5 pode-se resolver facilmente a eq. dif. mas dá um
> problemão para alfa = 0 ...
>
>
>
> --- Em *sex, 8/7/11, João Maldonado *escreveu:
>
>
> De: João Maldonado 
> Assunto: [obm-l] Integral difícil
> Para: obm-l@mat.puc-rio.br
> Data: Sexta-feira, 8 de Julho de 2011, 21:55
>
>  Boa Tarde a todos
>
> Recentemente postei uma integral que não consegui resolver no fórum
>  PHYSICSFORUMS mas não obtive nenhuma resposta satisfatória.
> O problema a seguir é uma preparação para a IPhO, embora só a parte
> matemática interesse
>
> http://www.physicsforums.com/showthread.php?t=512186
>
> Reduzi o problema a equação encontrada no link acima, queria achar a função
> de velocidade em função da distância, S.
>
>


Re: [obm-l] Problemas(polinomoi- ideias!)

2011-07-04 Por tôpico Ralph Teixeira
Como voce disse, se a eh uma raiz de P(x), entao a^2+1 tem que ser raiz de
P(x) tambem. Entao se voce pegar as raizes de P(x) e "aplicar" x^2+1 nelas,
voce ainda tem que cair em raizes. Portanto, dada uma raiz qualquer a, temos
que a^2+1, (a^2+1)^2+1, etc. gera varias raizes de P(x). Como P(x) tem que
ter um numero finito de raizes distintas, essa sequencia tem que gerar um
ciclo, tem que repetir em algum momento.

Que ciclo? O caso mais simples seria fazer um ciclo de tamanho 1, ou seja,
fazer logo a^2+1=a, para o ciclo soh ter um termo. Foi a ideia que voce
colocou... mas nao dah certo -- a condicao deste ciclo ser finita eh
NECESSARIA para ter a igualdade pedida, mas ter um ciclo de raizes nao eh
SUFICENTE para garantir a igualdade pedida.

O Shine botou a bola embaixo do braco e levou para casa: nao ha polinomio
com a condicao pedida... :)

Mas, olha soh:* o Marcone, que propos o problema original, me mandou um
E-mail dizendo que realmente o enunciado original era mesmo
p(x^2+1)=(p(x))^2+1, mas por algum motivo ele nao conseguiu colocar a
correcao na lista.* Entao ainda ha um problema interessante (mas bem
diferente) para fazer.

(Eu jah vi isso em algum lugar, mas nao lembro onde...)

Abraco,
Ralph
2011/7/4 Johann Dirichlet 

> Puxa! Mas onde esta o erro da minha solução?
>
> Anyway, inicialmente pensei em fatorar o dito polinomio.
> Creio que ele seja mônico, abrindo a expressão geral o fator máximo é
> a^2=a.
> Aí, escreve ele na forma deprodutos (x-a_i).. Basicamente, um lado
> fica na forma
> x^2+1-a_ i, e o outro como (x-a_ i)^2. Supóndo que as raízes são, em
> alguma ordem, iguais, dá pra chegar em algum lugar.
>
>
>
>
> Em 01/07/11, Ralph Teixeira escreveu:
> > O raciocínio do Dirichlet mostra que basta achar UM polinômio (não
> > constante) que tenha esta propriedade. Afinal, como ele mostrou, se p(x)
> > serve, então q(x)=(p(x))^2 também serve.
> >
> > Mas seja lá quem for o polinômio mágico, eu sei que ou ele é um polinômio
> > par ou ele é ímpar. Afinal, escreva p(x)=P(x)+I(x) onde P(x) tem apenas
> os
> > termos de grau par e I(x) tem apenas os de grau ímpar.
> >
> > Ora, p(x)^2=(P^2+I^2)+2PI. Note que P^2+I^2 é um polinômio par e 2PI é
> > ímpar.
> >
> > Mas a condição manda que p^2=p(x^2+1), que é uma função par. Então o
> termo
> > 2PI não pode existir, isto é, P=0 ou I=0. Assim, p(x) é par ou ímpar. E
> > x^2-x+1 não é um nem outro, então não funcionou...
> >
> > Então precisamos ainda mostrar que existe UM tal polinômio!
> >
> > Abraço,
> >   Ralph
> >
> > P.S.: Tem certeza que o enunciado é esse mesmo? Não seria, sei lá,
> > p(x^2+1)=(p(x))^2+1 ao invés?
> > 2011/7/1 Johann Dirichlet 
> >
> >> Em 01/07/11, Johann Dirichlet escreveu:
> >> > Em 30/06/11, marcone augusto araújo
> >> > borges escreveu:
> >> >>
> >> >> 1) Se p é inteiro primo ímpar,mostre que o numerador da fração
> >> >> 1+1/2+1/3+...1/(p-1) é um múltiplo de p.
> >> >
> >> > 1) Teorema de Wolstenholme, se não me engano...
> >> >
> >> > Bora lá, usar o velho truque das pontas de Gauss:
> >> > 1/k+1/(p-k)=p/(k(p-k));
> >> > assim sendo, temos um monte de frações p/(alguma coisa). Esta coisa
> >> > não será múltipla de p em momento nenhum, logo nada aniquila este
> >> > fator p.
> >> >
> >> >>
> >> >> 2) Mostre que existem infinitos polinômios p(x) com coeficientes
> reais
> >> >> tais
> >> >> que p(x^2+1) = [p(x)]^2.
> >>
> >> É mais mole do que eu pensei!
> >>
> >> 1 - Se P e Q são soluções da equação acima, P*Q também será. Óbvio!
> >> 2 - Um polinômio possível é x^2-x+1.
> >> Como sei? Simples:
> >>
> >> Se L é um zero de P, então L^2+1 também será.
> >> Se eu conseguir L=L^2+1, terei uma solução pronta!
> >> Basta abrir o polinomio sem medo.
> >>
> >>
> >> P.S.: saber todas as soluções me parece mais desgastante. Aplicando a
> >> transformação T(L)=L^2+1 um numero finito de vezes, todos os
> >> polinômios dos pontos fixos são soluções. A treta é saber se não
> >> escapa nenhum (até porque muitos desses polinomios são fatoráveis, I
> >> think so).
> >>
> >> >>
> >> >> 3) Uma corda AB,de comprimento constante,desliza sobre uma
> >> >> semicircunferência determinada por um diâmetro d.
> >> >> Considere o triângulo cujos vértices são: o ponto médio da corda e as
> >> >> projeções ortogonais dos seus extremos A e B
> >&

Re: [obm-l] Problemas(ajuda)

2011-07-01 Por tôpico Ralph Teixeira
Melhorando aos poucos, ainda usando as ideias do Dirichlet: p(x) não pode
ser ímpar. Se fosse, 0 seria raiz. Mas então 0^2+1=1 seria raiz, e 1^2+1=2
seria raiz, e 2^2+1=5 seria raiz... e p(x) não pode ter infinitas raízes.
Então estamos à procura de um polinômio **par** p(x) tal que
p(x^2+1)=[p(x)]^2.

Aliás, esse raciocínio mostra que esse p(x) não pode ter nenhuma raiz real
-- se tiver uma raiz real x, terá infinitas, já que x^2+1>x para todo x
real.

(Por enquanto, fico com a terrível impressão de que tal polinômio não
existe... Alguém achou o dito cujo?)
2011/7/1 Ralph Teixeira 

> O raciocínio do Dirichlet mostra que basta achar UM polinômio (não
> constante) que tenha esta propriedade. Afinal, como ele mostrou, se p(x)
> serve, então q(x)=(p(x))^2 também serve.
>
> Mas seja lá quem for o polinômio mágico, eu sei que ou ele é um polinômio
> par ou ele é ímpar. Afinal, escreva p(x)=P(x)+I(x) onde P(x) tem apenas os
> termos de grau par e I(x) tem apenas os de grau ímpar.
>
> Ora, p(x)^2=(P^2+I^2)+2PI. Note que P^2+I^2 é um polinômio par e 2PI é
> ímpar.
>
> Mas a condição manda que p^2=p(x^2+1), que é uma função par. Então o termo
> 2PI não pode existir, isto é, P=0 ou I=0. Assim, p(x) é par ou ímpar. E
> x^2-x+1 não é um nem outro, então não funcionou...
>
> Então precisamos ainda mostrar que existe UM tal polinômio!
>
> Abraço,
>   Ralph
>
> P.S.: Tem certeza que o enunciado é esse mesmo? Não seria, sei lá,
> p(x^2+1)=(p(x))^2+1 ao invés?
> 2011/7/1 Johann Dirichlet 
>
>> Em 01/07/11, Johann Dirichlet escreveu:
>> > Em 30/06/11, marcone augusto araújo
>> > borges escreveu:
>> >>
>> >> 1) Se p é inteiro primo ímpar,mostre que o numerador da fração
>> >> 1+1/2+1/3+...1/(p-1) é um múltiplo de p.
>> >
>> > 1) Teorema de Wolstenholme, se não me engano...
>> >
>> > Bora lá, usar o velho truque das pontas de Gauss:
>> 1/k+1/(p-k)=p/(k(p-k));
>> > assim sendo, temos um monte de frações p/(alguma coisa). Esta coisa
>> > não será múltipla de p em momento nenhum, logo nada aniquila este
>> > fator p.
>> >
>> >>
>> >> 2) Mostre que existem infinitos polinômios p(x) com coeficientes reais
>> >> tais
>> >> que p(x^2+1) = [p(x)]^2.
>>
>> É mais mole do que eu pensei!
>>
>> 1 - Se P e Q são soluções da equação acima, P*Q também será. Óbvio!
>> 2 - Um polinômio possível é x^2-x+1.
>> Como sei? Simples:
>>
>> Se L é um zero de P, então L^2+1 também será.
>> Se eu conseguir L=L^2+1, terei uma solução pronta!
>> Basta abrir o polinomio sem medo.
>>
>>
>> P.S.: saber todas as soluções me parece mais desgastante. Aplicando a
>> transformação T(L)=L^2+1 um numero finito de vezes, todos os
>> polinômios dos pontos fixos são soluções. A treta é saber se não
>> escapa nenhum (até porque muitos desses polinomios são fatoráveis, I
>> think so).
>>
>> >>
>> >> 3) Uma corda AB,de comprimento constante,desliza sobre uma
>> >> semicircunferência determinada por um diâmetro d.
>> >> Considere o triângulo cujos vértices são: o ponto médio da corda e as
>> >> projeções ortogonais dos seus extremos A e B
>> >> sobre o diâmetro d.Mostre que ,durante o deslizamento da corda,esse
>> >> triângulo é sempre isósceles e nunca muda de formato(i.é.,os ângulos do
>> >> triângulo são constantes)
>> >
>> > Faz um desenho!
>> > Diâmetro r;centro O, raio 1; corda AB, tamanho d, médio M; AB
>> > projetado em r dá XY.
>> >
>> > O triangulo AOB é obviamente isósceles.
>> > Os quadrilateros XOMA e YOMB são inscritíveis de diâmetros OA e OB
>> > respectivamente (angulos de 90 graus).
>> >
>> > Temos OXM=OAM=OBM=OYM, logo XMY é isosceles. E o angulo OBA depende
>> > unicamente de d.
>> >
>> > P.S.: duvido que os triangulos sejam todos congruentes. O angulo XOM
>> > define o tamanho de XM.
>> >
>> >>
>> >> Meus agradecimentos por qualquer esclarecimento.
>> >
>> >
>> > --
>> > /**/
>> > 神が祝福
>> >
>> > Torres
>> >
>>
>>
>> --
>> /**/
>> 神が祝福
>>
>> Torres
>>
>> =
>> Instru�ões para entrar na lista, sair da lista e usar a lista em
>> http://www.mat.puc-rio.br/~obmlistas/obm-l.html
>> =
>>
>
>


Re: [obm-l] Problemas(ajuda)

2011-07-01 Por tôpico Ralph Teixeira
O raciocínio do Dirichlet mostra que basta achar UM polinômio (não
constante) que tenha esta propriedade. Afinal, como ele mostrou, se p(x)
serve, então q(x)=(p(x))^2 também serve.

Mas seja lá quem for o polinômio mágico, eu sei que ou ele é um polinômio
par ou ele é ímpar. Afinal, escreva p(x)=P(x)+I(x) onde P(x) tem apenas os
termos de grau par e I(x) tem apenas os de grau ímpar.

Ora, p(x)^2=(P^2+I^2)+2PI. Note que P^2+I^2 é um polinômio par e 2PI é
ímpar.

Mas a condição manda que p^2=p(x^2+1), que é uma função par. Então o termo
2PI não pode existir, isto é, P=0 ou I=0. Assim, p(x) é par ou ímpar. E
x^2-x+1 não é um nem outro, então não funcionou...

Então precisamos ainda mostrar que existe UM tal polinômio!

Abraço,
  Ralph

P.S.: Tem certeza que o enunciado é esse mesmo? Não seria, sei lá,
p(x^2+1)=(p(x))^2+1 ao invés?
2011/7/1 Johann Dirichlet 

> Em 01/07/11, Johann Dirichlet escreveu:
> > Em 30/06/11, marcone augusto araújo
> > borges escreveu:
> >>
> >> 1) Se p é inteiro primo ímpar,mostre que o numerador da fração
> >> 1+1/2+1/3+...1/(p-1) é um múltiplo de p.
> >
> > 1) Teorema de Wolstenholme, se não me engano...
> >
> > Bora lá, usar o velho truque das pontas de Gauss: 1/k+1/(p-k)=p/(k(p-k));
> > assim sendo, temos um monte de frações p/(alguma coisa). Esta coisa
> > não será múltipla de p em momento nenhum, logo nada aniquila este
> > fator p.
> >
> >>
> >> 2) Mostre que existem infinitos polinômios p(x) com coeficientes reais
> >> tais
> >> que p(x^2+1) = [p(x)]^2.
>
> É mais mole do que eu pensei!
>
> 1 - Se P e Q são soluções da equação acima, P*Q também será. Óbvio!
> 2 - Um polinômio possível é x^2-x+1.
> Como sei? Simples:
>
> Se L é um zero de P, então L^2+1 também será.
> Se eu conseguir L=L^2+1, terei uma solução pronta!
> Basta abrir o polinomio sem medo.
>
>
> P.S.: saber todas as soluções me parece mais desgastante. Aplicando a
> transformação T(L)=L^2+1 um numero finito de vezes, todos os
> polinômios dos pontos fixos são soluções. A treta é saber se não
> escapa nenhum (até porque muitos desses polinomios são fatoráveis, I
> think so).
>
> >>
> >> 3) Uma corda AB,de comprimento constante,desliza sobre uma
> >> semicircunferência determinada por um diâmetro d.
> >> Considere o triângulo cujos vértices são: o ponto médio da corda e as
> >> projeções ortogonais dos seus extremos A e B
> >> sobre o diâmetro d.Mostre que ,durante o deslizamento da corda,esse
> >> triângulo é sempre isósceles e nunca muda de formato(i.é.,os ângulos do
> >> triângulo são constantes)
> >
> > Faz um desenho!
> > Diâmetro r;centro O, raio 1; corda AB, tamanho d, médio M; AB
> > projetado em r dá XY.
> >
> > O triangulo AOB é obviamente isósceles.
> > Os quadrilateros XOMA e YOMB são inscritíveis de diâmetros OA e OB
> > respectivamente (angulos de 90 graus).
> >
> > Temos OXM=OAM=OBM=OYM, logo XMY é isosceles. E o angulo OBA depende
> > unicamente de d.
> >
> > P.S.: duvido que os triangulos sejam todos congruentes. O angulo XOM
> > define o tamanho de XM.
> >
> >>
> >> Meus agradecimentos por qualquer esclarecimento.
> >
> >
> > --
> > /**/
> > 神が祝福
> >
> > Torres
> >
>
>
> --
> /**/
> 神が祝福
>
> Torres
>
> =
> Instru�ões para entrar na lista, sair da lista e usar a lista em
> http://www.mat.puc-rio.br/~obmlistas/obm-l.html
> =
>


[obm-l] Re: [obm-l] Números inteiros

2011-06-27 Por tôpico Ralph Teixeira
Eh fiquei tambem com a impressao que, em geral, ac eh bem maior que a+c em
modulo.

Vejamos como formalizar isto. Primeiro vou me livrar de uns casos pequenos
(que soh vi serem necessarios depois que terminei o problema :P):

CASO 0: Se um deles for 0 (digamos a=0)
Entao -2=b+c, que tem uma infinidade de solucoes. Assim, temos as inumeras
solucoes do tipo (0,n,-2-n) com n inteiro, e suas permutacoes.

CASO 1: Se um deles for 1 (digamos a=1).
Entao bc-2=b+c+1, isto eh, (b-1)(c-1)=4. Temos entao
{b-1,c-1}={2,2},{-2,-2},{1,4} ou {-1,-4}. Daqui vem as solucoes novas:
(1,3,3), (1,-1,-1), (1,2,5) -- e permutacoes.

CASO 2: Se um deles for -1 (digamos a=-1)
Entao -bc-2=-1+b+c
bc+b+c+1=0
(b+1)(c+1)=0
Entao b=-1 ou c=-1. Assim temos as solucoes do tipo (-1,-1,n) e permutacoes.

Acho que agora jah dah para fazer o caso geral, onde vou supor que todos
sao, em modulo, maiores que 2. Mas os sinais atrapalham, entao vou
subdividir em mais casos:

CASO 3: Todos positivos (digamos a>=b>=c>=2).
a(bc-1)=b+c+2 (como bc-1>0, a>=2 e c<=b)
2(bc-1)<=2b+2
bc-1<=b+1
b(c-1)<=2
Que nao dah muitas opcoes Como b>=c>=2, soh fica a opcao b=c=2!
Em suma, achamos apenas a resposta (2,2,2).

CASO 4: Dois positivos, um negativo (digamos a>=b>=2 mas c<=-2)
Entao troco (a,b,c) por (A,B,-C) para ficar com A,B,C positivos. Fica:
-ABC-2=A+B-C
ABC+A+B=C-2
Mas ABC+A+B>=4C+2+2, entao:
C-2>=4C+4
C<=-2 (impossivel)

CASO 5: Dois negativos, um positivo (digamos a>=2 e -2>=b>=c)
Troco (a,b,c) por (A,-B,-C). Fica:
ABC-2=A-B-C
ABC+B+C=A+2
Mas ABC+B+C>=4A+2+2, entao:
A+2>=4A+4
3A<=-2 (impossivel)

CASO 6: Todos negativos (digamos, 0>c>=b>=a)
Troco (a,b,c) por (-A,-B,-C) (com A>=B>=C)
-ABC-2=-A-B-C
A(BC-1)=B+C-2
Como BC-1>0, A>=2 e C<=B, vem:
2(BC-1)<=2B-2
BC-1<=B-1
B(C-1)<=0 (impossivel, pois B,C>=2)



Resumindo tudo, as solucoes sao:
(0,n,-2-n), (-1,-1,n), (1,3,3), (1,2,5), (2,2,2) e permutacoes.

Abraco,
 Ralph

2011/6/27 marcone augusto araújo borges 

>  Achar todas as soluções inteiras da equação abc - 2 = a + b + c .
>
>É fácil achar algumas soluções.Como (2,2,2) ou (3,3,1),por exemplo.
>Isolando b,obtemos b=(a+c+2)/(ac - 1),a impressão que dá é que em geral
> o módulo de ac é maior que o módulo de a+c,
> o módulo do denominador é maior que o módulo do numerador e b não é
> inteiro.
>Tentei uma maneira de restringir ao máximo os possíveis valores de a e
> c,mas...emperrei.
>Obrigado a quem puder ajudar.
>
>


[obm-l] Re: [obm-l] questão trigonometria complicada

2011-06-27 Por tôpico Ralph Teixeira
Ah, já vi errinho de sinal no meio do caminho, no sinal de ab+ac+bc e no de
abc. Corrigi abaixo, mas deve haver outros. De qualquer forma, a ideia ainda
vale.
2011/6/27 Ralph Teixeira 

> Hmmm, vejamos. Será que a gente arruma algum polinômio cujas raízes sejam
> as 3 parcelas da sua soma?
>
> Considere a "famosa" identidade
> trigonométrica sin7t=(8(cos2t)^3+4(cos2t)^2-4(cos2t)-1).sint
>
> (Desculpa, não pude resistir.)
>
> Note que t=kpi/7 (k=1,2,4) dá três raízes de sin7t, mas nenhum deles dá
> raiz de sint. Então estes valores de t devem anular o termo entre
> parênteses... Em outras palavras, se você considerar o polinômio
> P(x)=8x^3+4x^2-4x-1, você verá que suas raízes são exatamente cos(2pi/7),
> cos(4pi/7) e cos(8pi/7) -- exatamente porque é um polinômio do 3o grau,
> então se eu achei 3 raízes distintas, achei todas.
>
> (O argumento também vale para k=3,5,6, mas então obtemos
> cos(6pi/7)=cos(8pi/7), cos(10pi/7)=cos(4pi/7) e cos(12pi/7)=cos(2pi/7), que
> são aquelas raízes de novo)
>
> Em suma, o problema agora é: sejam a,b e c as raízes de
> P(x)=8x^3+4x^2-4x-1. Encontre a^(1/3)+b^(1/3)+c^(1/3).
>
> Vou escrever a^(1/3)=A, b^(1/3)=B e c^(1/3)=C. Mas, do polinômio sabemos
> que
> a+b+c=-1/2, isto é, A^3+B^3+C^3=-1/2
> ab+ac+bc=-1/2, isto é, A^3B^3+A^3C^3+B^3C^3=-1/2
> abc=1/8, isto é, ABC=1/2.
>
> Poxa, eu até consigo fazer o resto, mas é HORRENDO. Vamos lá.
>
> Agora, talvez você já tenha visto a identidade
> x^3+y^3+z^3-3xyz=(x+y+z)((x+y+z)^2-3(xy+xz+yz))
>
> Aplicando esta identidade com (x,y,z)=(A,B,C) temos:
> -1/2-3/2=-2=S(S^2-3D) (onde S=A+B+C e D=AB+AC+BC)
>
> Aplicando esta identidade com (x,y,z)=(AB,AC,BC), temos:
> -1/2-3(1/4)=-5/4=D(D^2-3SP)=D(D^2-3S/2) (onde P=ABC=1/2)
>
> Enfim, duas equações e duas incógnitas! Tire D da primeira e jogue na
> segunda -- fica horrendo, mas dá uma equação polinomial de grau 9 em S, com
> termos apenas em S^3, S^6 e S^9. Faça S^3=Z, resolva a equação cúbica em Z,
> S é a raiz cúbica de Z.
>
> Argh! Tá, fiquei sem vontade de terminar as contas, e devo ter errado algo
> no meio do caminho, mas saiu!
>
> Abraço,
>Ralph
>
>
> 2011/6/26 Jefferson Franca 
>
>> Boa tarde senhores. Será que alguém poderia me iluminar nesta questão:
>> Calcule o valor da soma (cos(2*pi/7)^1/3 + (cos(4*pi/7))^1/3 +
>> (cos(8*pi/7))^1/3 ?
>> abs
>>
>
>


[obm-l] Re: [obm-l] questão trigonometria complicada

2011-06-27 Por tôpico Ralph Teixeira
Hmmm, vejamos. Será que a gente arruma algum polinômio cujas raízes sejam as
3 parcelas da sua soma?

Considere a "famosa" identidade
trigonométrica sin7t=(8(cos2t)^3+4(cos2t)^2-4(cos2t)-1).sint

(Desculpa, não pude resistir.)

Note que t=kpi/7 (k=1,2,4) dá três raízes de sin7t, mas nenhum deles dá raiz
de sint. Então estes valores de t devem anular o termo entre parênteses...
Em outras palavras, se você considerar o polinômio P(x)=8x^3+4x^2-4x-1, você
verá que suas raízes são exatamente cos(2pi/7), cos(4pi/7) e cos(8pi/7) --
exatamente porque é um polinômio do 3o grau, então se eu achei 3 raízes
distintas, achei todas.

(O argumento também vale para k=3,5,6, mas então obtemos
cos(6pi/7)=cos(8pi/7), cos(10pi/7)=cos(4pi/7) e cos(12pi/7)=cos(2pi/7), que
são aquelas raízes de novo)

Em suma, o problema agora é: sejam a,b e c as raízes de P(x)=8x^3+4x^2-4x-1.
Encontre a^(1/3)+b^(1/3)+c^(1/3).

Vou escrever a^(1/3)=A, b^(1/3)=B e c^(1/3)=C. Mas, do polinômio sabemos que
a+b+c=-1/2, isto é, A^3+B^3+C^3=-1/2
ab+ac+bc=1/2, isto é, A^3B^3+A^3C^3+B^3C^3=1/2
abc=-1/8, isto é, ABC=-1/2.

Poxa, eu até consigo fazer o resto, mas é HORRENDO. Vamos lá.

Agora, talvez você já tenha visto a identidade
x^3+y^3+z^3-3xyz=(x+y+z)((x+y+z)^2-3(xy+xz+yz))

Aplicando esta identidade com (x,y,z)=(A,B,C) temos:
-1/2+3/2=1=S(S^2-3D) (onde S=A+B+C e D=AB+AC+BC)

Aplicando esta identidade com (x,y,z)=(AB,AC,BC), temos:
1/2-3(1/4)=-1/4=D(D^2-3SP)=D(D^2+3S/2) (onde P=ABC=-1/2)

Enfim, duas equações e duas incógnitas! Tire D da primeira e jogue na
segunda -- fica horrendo, mas dá uma equação polinomial de grau 9 em S, com
termos apenas em S^3, S^6 e S^9. Faça S^3=Z, resolva a equação cúbica em Z,
S é a raiz cúbica de Z.

Argh! Tá, fiquei sem vontade de terminar as contas, e devo ter errado algo
no meio do caminho, mas saiu!

Abraço,
   Ralph


2011/6/26 Jefferson Franca 

> Boa tarde senhores. Será que alguém poderia me iluminar nesta questão:
> Calcule o valor da soma (cos(2*pi/7)^1/3 + (cos(4*pi/7))^1/3 +
> (cos(8*pi/7))^1/3 ?
> abs
>


[obm-l] Re: [obm-l] Números inteiros

2011-06-21 Por tôpico Ralph Teixeira
Ah, errinho bobo: eh 5d-1={4,8,12,0}, que nao afeta o resto.
2011/6/21 Ralph Teixeira 

> 1) Suponho n natural. Como 28n^2+1 eh impar e tem que ser quadrado
> perfeito, escrevo
>
> 28n^2+1=(2k-1)^2 (com k inteiro)
> 7n^2=k^2-k=k(k-1)
>
> (Note que a expressao toda eh 2+2(2k-1)=4k; entao nosso objetivo eh mostrar
> que k eh quadrado perfeito)
>
> Leminha: Como k e k-1 sao primos entre si, um deles eh um quadrado
> perfeito, o outro eh 7 vezes um quadrado perfeito.
> Provinha: Um dos fatores k e k-1 nao eh divisivel por 7, o outro eh. Seja
> 7A o divisivel por 7, e B o outro.
> Temos n^2=AB com A e B primos entre si. Entao A e B sao quadrados perfeitos
> (Se p eh um fator de A, entao p tem de ser fator de n. Mas entao p aparece
> do lado esquerdo um numero par de vezes (em n^2).
> Como A e B sao primos entre si, p nao aparece em B -- entao p aparece um
> numero par de vezes em A.
> Todo fator primo de A aparece um numero par de vezes em A? Entao, A eh um
> quadrado perfeito. Idem para B.)
>
> Caso 1: k=a^2, k-1=7b^2 -- entao a expressao eh k=a^2, acabou.
> Caso 2: k=7a^2, k-1=b^2. Entao 7a^2-b^2=1, isto eh, 7a^2=b^2+1. Mas isto eh
> impossivel: b^2=(0 ou 1) mod 4, enquanto 7a^2=(0 ou 3) mod 4.
>
> 2) Este eh o Problema 1 da IMO 1986 (Polonia). Eu lembro... :)
> Um jeito de fazer eh olhar tudo mod 16. Os quadrados perfeitos mod 16 sao
> 0,1,4,9. Vou escrever tudo mod 16, e vou botar "=" ao inves de "pertence":
> 2d-1={0,1,4,9} implica em 2d={1,2,5,10}, isto eh, 2d={2,10}, e
> d={1,5,9,13}.
> Respectivamente, viria 5d-1={4,8,12,1}. Soh os dois das pontas podem ser
> quadrados perfeitos, isto eh, d={1,13}.
> Mas entao 13d-1={12,8}, e nenhum deles eh quadrado perfeito mod 16.
>
> 3) (x+1)(x^2+1)=2^y. Entao ambos x+1 e x^2+1 tem de ser potencias de 2.
> Como 2^y e x^2+1 sao positivos, x+1 tambem terah de ser positivo, isto eh,
> x eh um inteiro nao-negativo.
> CASO 1: x+1=1, dah x=0, entao y=0. (x,y)=(0,0) serve.
> CASO 2: x+1=2, dah x=1, entao y=2. (x,y)=(1,2) serve.
> CASO 3: x+1 eh divisivel por 4. Entao (x^2+1)=(x+1)(x-1)+2=2 (mod 4)...
> Assim, os unicos jeitos de x^2+1 ser potencia de 2 sao:
>
> -- x^2+1=1, isto eh, x=0, que jah foi.
> -- x^2+1=2, isto eh, x=1, que jah foi.
>
> Abraco, Ralph
>
>
> 2011/6/21 marcone augusto araújo borges 
>
>>  1) Prove que se 2+2raiz(28n^2 + 1) é um inteiro,então é um quadrado
>> perfeito.
>>
>> 2) Mostre que não existe um natural d tal que os nùmeros 2d - 1,5d - 1 e
>> 13d - 1 sejam quadrados perfeitos.
>>
>> 3) Encontre todas as soluções de 1 + x +x^2 + x^3 = 2^y em inteiros x e y
>>
>> Agradeço antecipadamente a quem puder ajudar.
>>
>
>


[obm-l] Re: [obm-l] Números inteiros

2011-06-21 Por tôpico Ralph Teixeira
1) Suponho n natural. Como 28n^2+1 eh impar e tem que ser quadrado perfeito,
escrevo

28n^2+1=(2k-1)^2 (com k inteiro)
7n^2=k^2-k=k(k-1)

(Note que a expressao toda eh 2+2(2k-1)=4k; entao nosso objetivo eh mostrar
que k eh quadrado perfeito)

Leminha: Como k e k-1 sao primos entre si, um deles eh um quadrado perfeito,
o outro eh 7 vezes um quadrado perfeito.
Provinha: Um dos fatores k e k-1 nao eh divisivel por 7, o outro eh. Seja 7A
o divisivel por 7, e B o outro.
Temos n^2=AB com A e B primos entre si. Entao A e B sao quadrados perfeitos
(Se p eh um fator de A, entao p tem de ser fator de n. Mas entao p aparece
do lado esquerdo um numero par de vezes (em n^2).
Como A e B sao primos entre si, p nao aparece em B -- entao p aparece um
numero par de vezes em A.
Todo fator primo de A aparece um numero par de vezes em A? Entao, A eh um
quadrado perfeito. Idem para B.)

Caso 1: k=a^2, k-1=7b^2 -- entao a expressao eh k=a^2, acabou.
Caso 2: k=7a^2, k-1=b^2. Entao 7a^2-b^2=1, isto eh, 7a^2=b^2+1. Mas isto eh
impossivel: b^2=(0 ou 1) mod 4, enquanto 7a^2=(0 ou 3) mod 4.

2) Este eh o Problema 1 da IMO 1986 (Polonia). Eu lembro... :)
Um jeito de fazer eh olhar tudo mod 16. Os quadrados perfeitos mod 16 sao
0,1,4,9. Vou escrever tudo mod 16, e vou botar "=" ao inves de "pertence":
2d-1={0,1,4,9} implica em 2d={1,2,5,10}, isto eh, 2d={2,10}, e d={1,5,9,13}.
Respectivamente, viria 5d-1={4,8,12,1}. Soh os dois das pontas podem ser
quadrados perfeitos, isto eh, d={1,13}.
Mas entao 13d-1={12,8}, e nenhum deles eh quadrado perfeito mod 16.

3) (x+1)(x^2+1)=2^y. Entao ambos x+1 e x^2+1 tem de ser potencias de 2.
Como 2^y e x^2+1 sao positivos, x+1 tambem terah de ser positivo, isto eh, x
eh um inteiro nao-negativo.
CASO 1: x+1=1, dah x=0, entao y=0. (x,y)=(0,0) serve.
CASO 2: x+1=2, dah x=1, entao y=2. (x,y)=(1,2) serve.
CASO 3: x+1 eh divisivel por 4. Entao (x^2+1)=(x+1)(x-1)+2=2 (mod 4)...
Assim, os unicos jeitos de x^2+1 ser potencia de 2 sao:

-- x^2+1=1, isto eh, x=0, que jah foi.
-- x^2+1=2, isto eh, x=1, que jah foi.

Abraco, Ralph


2011/6/21 marcone augusto araújo borges 

>  1) Prove que se 2+2raiz(28n^2 + 1) é um inteiro,então é um quadrado
> perfeito.
>
> 2) Mostre que não existe um natural d tal que os nùmeros 2d - 1,5d - 1 e
> 13d - 1 sejam quadrados perfeitos.
>
> 3) Encontre todas as soluções de 1 + x +x^2 + x^3 = 2^y em inteiros x e y
>
> Agradeço antecipadamente a quem puder ajudar.
>


Re: [obm-l] Probleminha....

2011-06-15 Por tôpico Ralph Teixeira
Oi, Paulo.

Sim, a Fundação Getulio Vargas encomendou que escrevêssemos um livro-texto
de Matemática para o Ensino Médio. São 3 volumes, em princípio um para cada
ano do Ensino Médio (nosso "modelo" imediato foi o Colégio Santo Inácio, já
que 3 dos autores, incluindo o Miguel, trabalham lá; eles já estão usando o
livro por lá).

Os Volumes 1 e 2 já foram publicados pela Editora do Brasil; o Volume 3
ainda está em processo de diagramação e revisão (por isso a gente não
divulgou muito ainda, a coleção não está completa, e ainda não foi apreciada
pelo MEC). Além de contar com a experiência incrível de anos de didática do
Miguel Jorge (que é o autor principal, aprendi um monte de coisas bacanas
com ele), a gente tentou dar um pouco mais de ênfase em lógica e
demonstrações do que o livro "usual" de Ensino Médio -- mas procurando
evitar formalismo excessivo...  Em outras palavras, na hora de botar ou não
uma demonstração de um fato, a gente pensou:
(A) É factível nível Ensino Médio?
(B) É interessante?
(C) Ajuda a entender o fato?
(D) É bonita pra caramba?
Se (A) e ((B) ou (C) ou (D)), a demonstração entra.

(Viu, lógica matemática! Capítulo 1 do livro 1! :) :) :) )

A gente também trabalhou bastante para o livro ficar bonito e organizado
(mas sem ficar botando fotos a cada página ou bonequinhos falando com
balõezinhos, que o Miguel não gosta :) :)). Tem uma diagramação levemente
colorida e bem simpática, vários exemplos bem bacanas, e toneladas de
exercícios resolvidos e propostos. Deu um trabalho de cão (e a gente ainda
vai ter que acertar vários detalhes para a 2a edição), mas acho que ficou
muito legal.

Bom, chega de propaganda. Na livraria FGV, eles me dizem ter apenas 2
exemplares de cada um dos dois volumes (a quase R$100 cada, são livros BEM
grossos), mas eles podem encomendar mais -- ligue para lá e pergunte para
não perder a viagem. Depois, mande para a gente os erros que você encontrar
(são 117, obviamente todos deixados de propósito, a gente nunca erraria nada
:P ).

Abraço,
  Ralph

2011/6/15 Paulo Barclay Ribeiro 

> oi Ralph,
>
> Vi na internet um livro chamado Aprender matematica e o seu nome estava
> nele junto com outros autores , acho que era o prof miguel jorge.Minhas
> perguntas:
> 1) Você é ,realmenteum dos autores?
> 2) Miguel Jorge é o mesmo que escreveu conjuntamente com o Morgado e
> EWagner o livro geometria 1?
> 3) Em caso afirmativo ,onde posso adquirir o livro?Ele tá disponível na
> FGV?
>
> Um abraço
> Paulo
>  --
> *De:* Ralph Teixeira 
> *Para:* obm-l@mat.puc-rio.br
> *Enviadas:* Segunda-feira, 6 de Junho de 2011 22:57:34
> *Assunto:* Re: [obm-l] Probleminha
>
> Que tal assim:
>
> Multiplicando tudo por (x-a)(x-b)(x-c), vemos que a equacao dada
> **implica**:
>
> (x-b)(x-c)+(x-a)(x-c)+(x-a)(x-b)=0
>
> Chame o lado esquerdo de f(x). Note que f(x) eh quadratica (coeficiente de
> x^2 eh 3, nao eh 0, entao quadratica mesmo). Tambem:
> f(a)=(a-b)(a-c)>0
> f(b)=(b-a)(b-c)<0
> f(c)=(c-a)(c-b)>0
> Assim, f(x) tem duas raizes reais, uma em (a,b), outra em (b,c). Como f eh
> quadratica, estas sao todas as raizes.
>
> Enfim, note que a, b e c nao sao raizes de f(x). Assim, a equacao f(x)=0 eh
> de fato EQUIVALENTE aa original (basta dividi-la por (x-a)(x-b)(x-c), o que
> eh permitido jah que x=a, x=b e x=c nao prestam).
>
> Abraco,
>  Ralph
>
>
> 2011/6/6 ruy de oliveira souza 
>
>> E ai rapaziada? Acho que não estou entendendo bem esse enunciado:
>> " Considere a,b e c números reais tais que a> 1/(x-a)  + 1/(x-b)  + 1/(x-c)=0 , possui exatamente duas raízes, x1 e x2 ,
>> que satisfazem a condição a> Agradeço antecipadamente quem resolver. Fiz de um modo baseado em
>> "provas indiretas". Não estou muito satisfeito.
>>
>
>


[obm-l] Re: [obm-l] dúvida sobre séries

2011-06-06 Por tôpico Ralph Teixeira
Cuidado: nao confunda o TERMO GERAL de uma serie com a SERIE em si...

Na serie SOMATORIO(a_n), o termo geral eh a_n. Mas a serie consiste em SOMAR
todos esses a_n.

A SEQUENCIA 1/n converge para 0 quando n vai para infinito. 1/n eh o "termo
geral" da serie SOMATORIO(1/n) -- mas nao eh a SERIE.

A SERIE eh o SOMATORIO (infinito). Ver se a serie converge significa ver se
a seguinte de somas parciais converge quando o numero de termos vai para
infinito:

s_1=1
s_2=1+1/2
s_3=1+1/2+1/3
s_4=1+1/2+1/3+1/4
...
s_n=1+1/2+1/3+...+1/n
...

Note que s_n eh uma sequencia crescente. Com 4 termos, jah passou de 2.
Certamente A SERIE SOMATORIO(1/n) NAO converge para 0.

Ajudou? Agora falta provar que a serie de fato diverge, que eh outra
estoria... Primeiro veja se voce sacou a diferenca entre SEQUENCIA e SERIE.

Abraco,
 Ralph

2011/6/6 claudinei 

> Pessoal definitivamente nao consigo entender pq a série [somatória de 1/n]
> com n indo de 1 ao infinito, divergepq nao converge para zero
>
> alguém por favor poderia me explicar???
>
>
>


Re: [obm-l] Probleminha....

2011-06-06 Por tôpico Ralph Teixeira
Que tal assim:

Multiplicando tudo por (x-a)(x-b)(x-c), vemos que a equacao dada
**implica**:

(x-b)(x-c)+(x-a)(x-c)+(x-a)(x-b)=0

Chame o lado esquerdo de f(x). Note que f(x) eh quadratica (coeficiente de
x^2 eh 3, nao eh 0, entao quadratica mesmo). Tambem:
f(a)=(a-b)(a-c)>0
f(b)=(b-a)(b-c)<0
f(c)=(c-a)(c-b)>0
Assim, f(x) tem duas raizes reais, uma em (a,b), outra em (b,c). Como f eh
quadratica, estas sao todas as raizes.

Enfim, note que a, b e c nao sao raizes de f(x). Assim, a equacao f(x)=0 eh
de fato EQUIVALENTE aa original (basta dividi-la por (x-a)(x-b)(x-c), o que
eh permitido jah que x=a, x=b e x=c nao prestam).

Abraco,
 Ralph


2011/6/6 ruy de oliveira souza 

> E ai rapaziada? Acho que não estou entendendo bem esse enunciado:
> " Considere a,b e c números reais tais que a 1/(x-a)  + 1/(x-b)  + 1/(x-c)=0 , possui exatamente duas raízes, x1 e x2 ,
> que satisfazem a condição a Agradeço antecipadamente quem resolver. Fiz de um modo baseado em
> "provas indiretas". Não estou muito satisfeito.
>


Re: [obm-l] Produto Vetorial

2011-06-02 Por tôpico Ralph Teixeira
Bom, notações que eu vou usar:

XY -- segmento XY ou vetor XY, espero que não dê confusão.
 -- o produto interno de v com w
||v|| -- o módulo (norma, tamanho) do vetor v
vxw -- o produto vetorial de v com w
[u,v,w] -- o produto misto de u, v e w
|z| -- o módulo do número z

Então vamos lá: sejam X em AB e Y em CD tais que XY é a distância entre AB e
CD (faça um figurisco). Note que XY é ortogonal tanto a AB quanto a CD...
Assim, XY=c.n onde n=ABxCD e c é uma constante a ser calculada.

Melhor: X é a projeção ortogonal de A sobre XY, e Y é a projeção ortogonal
de D sobre XY. Então, XY é a projeção ortogonal de AD sobre a direção de n,
ou seja, o tamanho de XY é ||AD||.|cos(theta)|= || / ||n|| (onde theta
é o ângulo entre AD e n).

Em suma: d= ||  /  ||ABxCD|| = | [AD,AB,CD] |   /||ABxCD||.

(Isto leva à fórmula do volume do tetraedro em função da distância entre
arestas opostas: V=(1/6).d.||AB||.||CD||.sin(alpha) onde alpha é o ângulo
entre AB e CD.)

---///---

Poxa, eu ando dando aula de série de Fourier, e por causa disto fiquei com
vontade de escrever logo o ***vetor*** XY... É só lembrar que a projeção
ortogonal de v sobre w é o vetor (/).w. Então

XY=( /)  .  ABxCD.

cuja norma é a expressão anterior para d.

---///---

Era isso?

Abraço,
 Ralph
2011/6/1 João Maldonado 

>  d -> 1/2sqrt(-a^6 + 2 a^4 b^2 - a^2 b^4 + 4 m^2 - 2 a^4 m^2 + 2 a^2
> b^2 m^2 - a^2 m^4 - m^4 n^2 - 2 m^2 n^4 - n^6 + 2 m^2 n^2 o^2 + 2 n^4 o^2 -
> n^2 o^4)
> Sendo  d a distância entre as arestas a e n
>
> Não sei o que  quer dizer vetor (aliás, sei a definição mas não sei
> calcular), calculei  d em função dos lados do tetradro
>
> []'s
> João
> --
> From: vitor__r...@hotmail.com
> To: obm-l@mat.puc-rio.br
> Subject: [obm-l] Produto Vetorial
> Date: Wed, 1 Jun 2011 14:15:30 +0300
>
> Exprima a distância entre duas arestas opostas AB e CD de um tetraedro ABCD
> em função de (AB),(DC),(AD).
> obs: (MN) quer dizer vetor MN
>


[obm-l] Re: [obm-l] Equação de variáveis inteiras

2011-05-30 Por tôpico Ralph Teixeira
2) Com este enunciado, não há triângulo nestas condições...

Tracei a bissetriz interna AX do ângulo A, fiz CX=x. Note que AXC é
isósceles, então AC=2xcosC, então BC=xcosC=2 no resto do problema.

Eu passei o x^2 pro lado de lá para fatorar:
(xy-7+x)(xy-7-x)=y^2.

(Agora, minha intuição me diz que, em geral, ambos xy-7+x e xy-7-x devem ser
BEM maiores que y, então isto vai restringir o problema... AH-HA!)

Note que xy-7-x=2). Assim, devemos ter xy-7-x=1, devemos ter y+1<=5, isto é, basta analisar y=2,3,4.

Abraço,
Ralph
2011/5/30 Pedro Júnior 

> Questões 02 e 03 da 2ª Parte da XXIV OCM - 2011 Nível 03, que ocorreu neste
> último sábado dia 28 de Maio:
>
> *02.* Um triângulo ABC é tal que o ângulo A=2C e AC = 2BC.. Mostre que
> este triângulo é retângulo.
> Usei a lei dos senos e lei dos cossenos mas não consegui concluir, favor
> quem tiver alguma ideia, contribuir...
>
> *03.* Determine todos os pares de inteiros não negativos que são soluções
> da equação (xy - 7)^2 = x^2 + y^2.
>
> Sem nenhuma estratégia descobrir que os pares (3,4); (4,3); (0,7); (7,0)
> satisfazem tal equação.
> Tentei enxergar o teo. de Pitágoras, fazendo x e y como catetos e xy - 7
> como hipotenusa.
> Há alguma resolução algébrica, alguma substituição que torne a equação com
> uma só incórnita?
>
> Desde já aradeço.
>
> --
>
> Pedro Jerônimo S. de O. Júnior
>
> Professor de Matemática
>
> Geo João Pessoa – PB
>
>


<    1   2   3   4   5   6   7   8   >